question 2

August 7, 2017 | Author: Aakash Shokeen | Category: Cloud, Troposphere, Atmosphere Of Earth, Wound, Rain
Share Embed Donate


Short Description

question 2...

Description

ATP Online - Meteorology

Page 1 of 213

Meteorology 1. At a certain position, the temperature on the 300 hPa chart is -48°C; according to the tropopause chart, the tropopause is at FL 330. What is the most likely temperature at FL 350 ? A. B. C. D.

-58°C. -56,5°C. -50°C. -54°C.

2. Which FL corresponds with the 200 hPa pressure level ? A. B. C. D.

FL 390. FL 100. FL 300. FL 50.

3. What is the approximate vertical interval which is equal to a pressure change of 1 hPa at an altitude of 5500 m ? A. B. C. D.

64 m (210 FT). 8 m (27 FT). 32 m (105 FT). 15 m (50 FT).

4. In order to calculate QFE from QNH, which of the following must be known ? A. B. C. D.

Temperature at the airfield. Elevation of the airfield. Elevation and the temperature at the airfield. Elevation of the airfield and the temperature at MSL.

5. What is the dry adiabatic lapse rate ? A. B. C. D.

3.0°C/1000 FT 3.5°C/1000 FT 2.0°C/1000 FT 1.5°C/1000 FT

6. A layer is conditionally unstable if the air A. B. C. D.

is unstable for saturated air as well as for dry air. is unstable for saturated air and stable for dry air. is stable for saturated air and unstable for dry air. becomes stable by lifting it.

7. A parcel of unsaturated air is lifted to just below the condensation level and then returned to its original level. What is the final temperature of the parcel of air? A. B. C. D.

Lower than the starting temperature. The same as the starting temperature. Higher than the starting temperature. It depends upon the QFE.

8. What is the effect of a strong low level inversion ? A. B. C. D.

It promotes extensive vertical movement of air. It results in good visual conditions near the surface. It promotes vertical wind shear. It prevents vertical wind shear.

9. The temperature at FL 140 is -12°C. What will the temperature be at FL 110 if the ICAO standard lapse rate is applied ? A. B. C. D.

-6°C. -9°C. -18°C. -15°C.

10. If atmospheric conditions exist such that the temperature deviation is ISA +10°C in the lower troposphere up to 18000 FT, what is the actual layer thickness between FL 60 and FL 120 ? A. B. C. D.

5760 FT. 6240 FT. 6000 FT. 5900 FT.

ATP Online - Meteorology

Page 2 of 213

11. Which of the following is a common cause of ground or surface temperature inversion ? A. B. C. D.

Terrestrial radiation on a clear night with no or very light winds. Heating of the air by subsidence Warm air being lifted rapidly aloft, in the vicinity of mountainous terrain. The movement of warm air under cold air.

12. How does the height of the tropopause normally vary with latitude in the northern hemisphere ? A. B. C. D.

It increases from south to north. It remains constant throughout the year. It remains constant from north to south. It decreases from south to north.

13. What, approximately, is the average height of the tropopause over the equator ? A. B. C. D.

11 km 40 km 8 km 16 km

14. In which layer is most of the atmospheric humidity concentrated ? A. B. C. D.

Stratosphere. Tropopause. Stratopause. Troposphere.

15. At FL 180, the air temperature is -35°C.The air density at this level is: A. B. C. D.

Less than the density of the ISA at FL 180. Unable to be determined without knowing the QNH. Equal to the density of the ISA at FL 180. Greater than the density of the ISA at FL 180.

16. Under what condition does pressure altitude have the same value as density altitude ? A. B. C. D.

When the altimeter setting is 1013.2 hPa. At sea level when the temperature is 0°C. At standard temperature. When the altimeter has no position error.

17. In the troposphere the decrease in pressure per 100 m increase in height is A. B. C. D.

larger in the higher layers than in the lower layers in the order of 27 hPa per 100 m in the friction layer larger in the lower layers than in the higher layers always larger in a high pressure area than in a low pressure area

18. In temperate latitudes what weather conditions may be expected over land during the summer in the centre of a stationary high pressure zone ? A. B. C. D.

Calm winds, haze. NS. CB, TS. TS, SH.

19. At what time of the year, are the paths of north Atlantic lows moving from west to east generally at their most southerly position? A. B. C. D.

Autumn. Winter. Summer. Spring.

20. What is the relationship between visibility and RVR in homogeneous fog? A. B. C. D.

The visibility is generally less than the RVR. The visibility generally is the same as the RVR. There is no specific relationship between the two. The visibility generally is greater than the RVR.

21. The wind indicator for a weather observation receives the measured value from an anemometer. Where is this instrument placed? A. 1 m above the runway. B. On a mast 6-10 m above the runway. C. On the roof of the weather station.

ATP Online - Meteorology

Page 3 of 213

D. Close to the station about 2 m above the ground.

22. You are flying with an outside air temperature of -12°C and a TAS of 250 kt at FL 150 through 8 oktas NS. What type and degree of icing is most probable? A. B. C. D.

Over flat terrain, away from fronts, moderate to severe mixed ice. Over flat terrain, moderate hoar frost. In clouds pushed up against the mountains, moderate to severe mixed ice. In clouds pushed up against the mountains, moderate to severe clear ice.

23. You intend to carry out a VFR flight over the Alps, on a fine and hot summer day. What is the best time of day to conduct this flight? A. B. C. D.

Mid-day. Early evening. Morning. Afternoon.

24. At what time of the year are tornadoes most likely to occur in North America? A. B. C. D.

Summer, autumn. Autumn, winter. Spring, summer. Winter.

25. In Central Europe when is the greatest likelihood for thunderstorms due to warm updrafts? A. B. C. D.

Around midnight. Early morning. Mid - afternoon. Late morning.

26. During which stage of thunderstorm development are rotor winds characterized by roll clouds most likely to occur ? A. B. C. D.

Cumulus stage and mature stage. mature stage. Dissipating stage. Cumulus stage.

27. Where is a squall line most likely to be expected? A. B. C. D.

At the surface position of a warm front. Behind a cold front. In front of a cold front occlusion at higher levels. In front of an active cold front.

28. What are squall lines? A. B. C. D.

Unusual intensive cold fronts. The paths of tropical revolving storms. Bands of intensive thunderstorms. The surface weather associated with upper air troughs.

29. Which weather phenomena are typical for the northern side of the Alps with strong winds from the south (Foehn)? A. B. C. D.

Icing, huge mass of clouds. Continuous precipitation, severe turbulence. Decrease in temperature, moderate to severe icing. Good visibility, turbulence.

30. What weather is prevalent in easterly waves? A. B. C. D.

Frontal weather. Clear skies. Thunderstorms and rain. Continuous rain.

31. What winds and air mass characteristics are mainly associated with the winter monsoon in the monsoon regions of the Indian subcontinent? A. B. C. D.

South-westerly winds carrying warm and humid air. North-easterly winds bringing dry and hazy air. North-westerly winds bringing dry and hazy air. South-easterly winds carrying warm and humid air.

ATP Online - Meteorology

Page 4 of 213

32. When is the RVR reported at most airports? A. B. C. D.

When the RVR decreases below 2500 m. When the RVR decreases below 2000 m. When the visibility decreases below 2000 m. When the visibility decreases below 1500 m.

33. How is the direction and speed of upper winds described in forecasts ? A. B. C. D.

The direction is relative to magnetic north and the speed is in miles per hour. The direction is relative to magnetic north and the speed is in knots. The direction is relative to true north and the speed is in knots. The direction is relative to true north and the speed is in miles per hour.

34. What positions are connected by contour lines on a weather chart? A. B. C. D.

Positions with the same wind velocity. Positions with the same thickness between two constant pressure levels. Positions with the same air density. Positions with the same height in a chart of constant pressure.

35. In which meteorological forecast chart is information about CAT regions found? A. B. C. D.

300 hPa chart. Significant weather chart. 500 hPa chart. 24 hour surface forecast.

36. Which of the following is typical for the passage of an active cold front in the summer ? A. B. C. D.

Mainly layered clouds Mainly towering clouds Rapid increase in temperature once the front has passed Rapid drop in pressure once the front has passed

37. Examining the pictures, on which one of the tracks (dashed lines) is this cross-section to be expected? A. B. C. D.

Track B-C Track A-D Track A-E Track B-D

38. Refer to the diagram. Assuming the usual direction of movement, to which position will the polar frontal wave have moved ? A. B. C. D.

Position 3 Position 2 Position 1 Position 4

ATP Online - Meteorology

Page 5 of 213

39. In a situation with a weak pressure gradient and no thunderstorms around, what will the indication of the aneroid altimeter of an aircraft parked on the ground do over a period of about ten minutes? A. B. C. D.

Increase rapidly. Show strong fluctuations. Apparently nothing, because any changes would be small. Decrease rapidly.

40. Under the weather conditions depicted, which of the following statements is likely to apply? A. B. C. D.

Thunderstorms may occur in the summer months over Central Europe. Radiation fog is unlikely in Central Europe in the winter. Severe gradient wind likely over Central Europe. Moderate to strong Foehn in the Alps.

41. In which of these temperature bands is ice most likely to form on the aircraft's surface? A. B. C. D.

0°C to -10°C -20°C to -35°C +10°C to 0°C -35°C to -50°C

42. Which of the following statements is true regarding moderate to severe airframe icing? A. It always occurs in altostratus cloud B. It may occur in the uppermost levels of a cumulonimbus capillatus formation

ATP Online - Meteorology

Page 6 of 213

C. It will occur in clear-sky conditions D. It is likely to occur in nimbostratus cloud

43. Which of the following statements is true regarding moderate to severe airframe icing? A. B. C. D.

It always occurs in altostratus cloud It will occur in the anvil part of the cumulonimbus cloud It will not occur in clear-sky conditions It is unlikely to occur in nimbostratus cloud

44. In which of these cloud types can icing be virtually ruled out? A. B. C. D.

CU NS SC CI

45. In which of these cloud types can icing be virtually ruled out? A. B. C. D.

CU SC NS CI

46. Which of the following conditions is most likely to cause airframe icing ? A. B. C. D.

+FZRA SHSN PE GR

47. The most likely reason for radiation fog to dissipate or become low stratus is: A. B. C. D.

increasing surface wind speed. a low level temperature inversion. an increasingly stable atmosphere. surface cooling.

48. At the same latitude the geostrophic wind is greater than the gradient wind around a low pressure system with equal pressure gradient because the A. B. C. D.

coriolis force is added to the pressure gradient centrifugal force is added to the pressure gradient centrifugal force opposes the pressure gradient coriolis force opposes to the centrifugal force

49. At the same latitude the geostrophic wind is less than the gradient wind around an anticyclone with equal pressure gradient because the A. B. C. D.

coriolis effect opposes the centrifugal force centrifugal force opposes the pressure gradient effect of coriolis is added to friction centrifugal force is added to the pressure gradient

50. An aircraft is flying in the southern hemisphere at low altitude (less than 2000 feet) and going directly away from a centre of low pressure. What direction, relative to the aircraft, does the wind come from ? A. B. C. D.

From the right and slightly on the tail From the left and slightly on the nose From the left and slightly on the tail From the right and slightly on the nose

51. What are the differences between radiation fog and advection fog ? A. B. C. D.

Radiation fog forms due to night cooling and advection fog due to daytime cooling. Radiation fog forms due to surface cooling at night in a light wind. Advection fog forms when warm humid air flows over a cold surface. Radiation fog is formed by surface cooling in a calm wind. Advection fog is formed by evaporation over the sea. Radiation fog forms only on the ground, advection fog only on the sea.

52. Depressions associated with developing frontal waves can be assumed to move in the direction of the 3000 feet wind A. B. C. D.

at the apex of the wave in the warm sector behind the cold front ahead of the warm front

ATP Online - Meteorology

Page 7 of 213

53. What types of cloud will you see flying at FL 50 towards a warm front? A. B. C. D.

At some 500 km from the front, groups of CB, later at some 250 km thickening AS At some 500 km AS, later CS and at some 80 km before the front CB At some 800 km CS, later AS, and at some 300 km NS until the front Mainly clear skies. At some 100 km from the front NS begins

54. A non-occluded frontal wave passes the airport. What form of precipitation do you expect ? A. Continuous rain or snow during several hours until the warm front arrives. The precipitation stops for several hours or becomes intermittent light within the warm sector. On the arrival of the cold front, showers will occur. B. Showers during some 2 hours until the warm front arrives. Drizzle in the warm sector within 12 hours. Rain or snow on the passage of the cold front. C. Rain or snow for about 12 hours until the warm front arrives. Within the warm sector rainfall increases. On the arrival of the cold front precipitation ceases. D. Continuous rain or snow while the frontal wave passes for a period of some 24 hours.

55. What characteristic is associated with a temperature inversion ? A. B. C. D.

Stability Clear ice Area of active storms Instability

56. What pressure is defined as QFE? A. B. C. D.

The pressure of the altimeter The pressure reduced to sea level using actual temperatures The pressure at field elevation The pressure reduced to sea level using ISA temperatures

57. In the lower layers of the atmosphere due to friction the wind changes direction towards the low pressure area because: A. B. C. D.

wind speed decreases and therefore coriolis force decreases turbulence is formed and pressure decreases turbulence is formed and pressure increases the pressure gradient increases

58. The most frequent wind direction in a valley caused by thermal effects is toward the: A. B. C. D.

mountain during daylight hours. valley during daylight hours. valley during daylight as much as at night. mountain at night.

59. The isobars drawn on a surface weather chart represent lines of equal pressure A. B. C. D.

reduced to sea level at height of observatory at flight level at a determined density altitude

60. What conditions are most likely to lead to the formation of hill fog? A. B. C. D.

High relative humidity and an unstable air mass Humid stable air mass, wind blowing towards the hills. Precipitation which is lifted by the action of moderate winds striking the range Clear skies, calm or light winds, with relatively low humidity

61. What is the approximate composition of the dry air by volume in the troposphere ? A. B. C. D.

21 % oxygen, 78 % nitrogen, and the rest other gasses 10 % oxygen, 89 % nitrogen, and the rest other gasses 88 % oxygen, 9 % nitrogen, and the rest other gasses 50 % oxygen, 40 % nitrogen, and the rest other gasses

62. How does temperature vary with increasing altitude in the ICAO standard atmosphere below the tropopause? A. B. C. D.

Remains constant At first it increases and higher up it decreases Decreases Increases

63. Which cloud type may indicate the presence of severe turbulence ? A. Altocumulus lenticularis B. Nimbostratus C. Stratocumulus

ATP Online - Meteorology

Page 8 of 213

D. Cirrocumulus

64. What is the boundary layer between troposphere and stratosphere called? A. B. C. D.

Ionosphere. Atmosphere. Stratosphere. Tropopause.

65. Which constant pressure altitude chart is standard for FL50? A. B. C. D.

500 hPa. 700 hPa. 850 hPa. 300 hPa.

66. An outside air temperature of -35°C is measured while cruising at FL 200. What is the temperature deviation from the ISA at this level? A. B. C. D.

10°C colder than ISA. 10°C warmer than ISA. 5°C colder than ISA. 5°C warmer than ISA.

67. Convective activity over land in mid-latitudes is greatest in A. B. C. D.

summer in the afternoon. winter in the afternoon. winter during the night and early morning. summer during the night and early morning.

68. Absolute instability is said to exist whenever the environmental lapse rate A. B. C. D.

is between the dry and saturated adiabatic lapse rate exceeds the dry adiabatic lapse rate is less than the saturated adiabatic lapse rate is less than the dry adiabatic lapse rate

69. Which of the following is a possible consequence of subsidence? A. B. C. D.

An inversion over a large area with haze, mist. Wide spread NS and AS clouds and intense precipitation. CB-clouds and thunderstorms over a large area. Clear air turbulence at higher altitudes .

70. The polar front is the boundary between: A. B. C. D.

maritime polar air and continental polar air. arctic air and polar air. polar air and tropical air. arctic air and tropical air.

71. What does dewpoint mean? A. B. C. D.

The temperature at which ice melts. The temperature at which the relative humidity and saturation vapour pressure are the same. The temperature to which a mass of air must be cooled in order to reach saturation. The freezing level (danger of icing).

72. Which of the following is the definition of relative humidity ? A. B. C. D.

Ratio between the actual mixing ratio and the saturation mixing ratio X 100 Ratio between water vapour pressure and atmospheric pressure X 100 Ratio between water vapour (g) and air (kg) X 100 Ratio between air temperature and dewpoint temperature X 100

73. The relative humidity of a sample air mass is 50%. How is the relative humidity of this air mass influenced by changes of the amount of water vapour in it? A. B. C. D.

It decreases with increasing water vapour. It is not influenced by changing water vapour. It increases with increasing water vapour. It is only influenced by temperature.

74. Relative humidity

ATP Online - Meteorology A. B. C. D.

Page 9 of 213

does not change when water vapour is added provided the temperature of the air remains constant. is not affected when air is ascending or descending. changes when water vapour is added, even though the temperature remains constant. is not affected by temperature changes of the air.

75. How, if at all, is the relative humidity of an unsaturated air mass influenced by temperature changes? A. B. C. D.

It increases with increasing temperature. It is not influenced by temperature changes. It decreases with increasing temperature. It is only influenced by the amount of water vapour.

76. How does relative humidity and the dewpoint in an unsaturated air mass change with varying temperature? A. B. C. D.

When temperature increases, the relative humidity decreases, and the dewpoint remains constant. When temperature increases, the relative humidity increases, and the dewpoint decreases. When temperature decreases, the relative humidity and the dewpoint remain constant. When temperature decreases, the relative humidity decreases, and the dewpoint increases.

77. When a mass of unsaturated air descends, what effect will it have on relative humidity? A. B. C. D.

It increases up to 100%, then remains stable. It decreases. It increases. It remains constant.

78. During the late afternoon an air temperature of +12°C and a dew point of +5°C were measured. What temperature change must at least occur during the night in order to induce saturation? A. B. C. D.

It must decrease to +7°C. It must decrease by 5°C. It must decrease to +6°C. It must decrease to +5°C.

79. Which of the following changes of state is known as sublimation (in meteorology)? A. B. C. D.

Solid direct to vapour Liquid direct to vapour Liquid direct to solid Solid direct to liquid

80. Clouds, fog or dew will always be formed when: A. B. C. D.

temperature and dew point are nearly equal. relative humidity reaches 98%. water vapour is present. water vapour condenses.

81. Which of the following are favourable conditions for the formation of freezing rain? A. B. C. D.

An isothermal layer aloft with a temperature just above 0°C through which rain is falling. Water droplets falling from cold air aloft with a temperature below 0°C into air with a temperature above 0°C. Warm air aloft from which rain is falling into air with a temperature below 0°C. Cold air aloft from which hail is falling into air that is warm.

82. Which form of precipitation from clouds containing only water is most likely to fall in mid-latitudes? A. B. C. D.

Hail. Heavy rain with large drops. Moderate rain with large drops. Drizzle.

83. Which of the following zones is most likely to encounter little or no precipitation? A. B. C. D.

Occlusions. The north side of the alps with a prevailing Foehn from the north. The north side of the alps with a prevailing Foehn from the south. Frontal zones.

84. Which of the following is a cause of stratus forming over flat land? A. B. C. D.

Unstable air. Radiation during the night from the earth surface in moderate wind. The release of latent heat. Convection during the day.

ATP Online - Meteorology

Page 10 of 213

85. Which of the following processes within a layer of air may lead to the building of CU and CB clouds? A. B. C. D.

Subsidence. Convection. Frontal lifting within stable layers. Radiation.

86. What are the characteristics of cumuliform clouds? A. B. C. D.

Small water droplets, stability, no turbulence and extensive areas of rain. Large water droplets, stability, no turbulence, showers and mainly rime ice. Large water droplets, instability, turbulence, showers and mainly clear ice. Small water droplets, instability, turbulence, extensive areas of rain and rime ice.

87. Which of the following types of clouds are evidence of unstable air conditions? A. B. C. D.

CU, CB. ST, CS. CI, SC. SC, NS.

88. Which of the following clouds are classified as medium level clouds in temperate regions ? A. B. C. D.

CI, CC. SC, NS AS, AC. CS, ST.

89. Which one of the displayed cloud forms is representative of altocumulus castellanus? A. B. C. D.

4. 3. 1. 2.

90. What is the main composition of clouds classified as "high level clouds"? A. B. C. D.

Water vapour. Water droplets. Supercooled water droplets. Ice crystals.

91. A plain in Western Europe with an average elevation of 500 m (1600 FT) above sea level is covered with a uniform AC layer of cloud during the summer months. At what height above the ground is the base of this cloud to be expected? A. B. C. D.

1500 - 7000 FT above the terrain. 100 - 1500 FT above the terrain. 15000 - 25000 FT above the terrain. 7000 - 15000 FT above the terrain.

92. Which of the following clouds may extend into more than one level? A. B. C. D.

Altocumulus. Nimbostratus. Stratus. Cirrus.

93. Which one of the following types of cloud is most likely to produce heavy precipitation ? A. B. C. D.

ST. CS. NS. SC.

94. What characteristics will the surface winds have in an area where the isobars on the weather map are very close together? A. Strong and flowing parallel to the isobars. B. Strong and flowing somewhat across the isobars. C. Moderate and flowing parallel to the isobars.

ATP Online - Meteorology

Page 11 of 213

D. Very weak but gusty and flowing somewhat across the isobars.

95. Which forces are balanced with geostrophic winds? A. B. C. D.

Pressure gradient force, centrifugal force, friction force. Pressure gradient force, Coriolis force. Pressure gradient force, Coriolis force, centrifugal force. Friction force, pressure gradient force, Coriolis force.

96. How does moderate turbulence affect an aircraft? A. B. C. D.

Rapid and somewhat rhythmic bumpiness is experienced without appreciable changes in altitude or attitude. Changes in altitude or attitude occur but the aircraft remains in positive control at all times. Continued flight in this environment will result in structural damage. Large, abrupt changes in altitude or attitude occur but the aircraft may only be out of control momentarily.

97. Which degree of aircraft turbulence is determined by the following ICAO description? "There may be moderate changes in aircraft attitude and/or altitude but the aircraft remains in positive control at all times. Usually, small variations in air speed. Changes in accelerometer readings of 0.5 to 1.0 g at the aircraft's center of gravity. Occupants feel strain against seat belts. Loose objects move about. Food service and walking are difficult." A. B. C. D.

Severe. Moderate. Light. Violent.

98. For an aircraft what are the meteorological dangers associated with a Harmattan wind? A. B. C. D.

Hail. Dust and poor visibility. Thunderstorms. Sand up to FL 150.

99. What is the strong relatively cold katabatic wind, blowing down the northern Adriatic coast, mainly during the winter and spring called? A. B. C. D.

Mistral. Ghibli. Scirocco. Bora.

100. In a land- and sea-breeze circulation the land-breeze blows: A. B. C. D.

during the night and is stronger than the sea-breeze. during the night and is weaker than the sea-breeze. during the day and is stronger than the sea-breeze. during the day and is weaker than the sea-breeze.

101. A high pressure area (slack pressure gradient) covers part of the Mediterranean Sea and coastal region during the summer. What surface wind direction is likely at an airport at the coast on a sunny afternoon? A. B. C. D.

Sea to land. Parallel to the coastline. Land to sea. Variable.

102. A katabatic wind blows A. B. C. D.

up the slope during the night. down the slope during the night. up the slope during the day. down the slope during the day.

103. In the northern hemisphere at 5000 ft/AGL a south-westerly wind is blowing. The surface wind at the same location is a southerly wind. What is the primary reason of difference between these two wind directions? A. B. C. D.

A strong pressure gradient at higher altitudes. Friction between the wind and the surface. Stronger Coriolis force at the surface. The influence of warm air at the lower altitude.

104. Friction between the air and the ground results in the northern hemisphere in: A. veering of the wind and increase of wind speed at the surface. B. veering of the wind and decrease of wind speed at the surface.

ATP Online - Meteorology

Page 12 of 213

C. backing of the wind and decrease of wind speed at the surface. D. backing of the wind and increase of wind speed at the surface.

105. Whilst flying at FL 180 on the northern hemisphere an aircraft experiences right drift. What effect, if any, will this have on the aircraft's true altitude ? A. B. C. D.

It decreases It remains constant. It increases. Without knowing the pressure change this question cannot be answered.

106. What is the most likely temperature at the tropical tropopause? A. B. C. D.

-35°C. -25°C. -55°C. -75°C.

107. Which weather condition lowers true altitude as compared to pressure altitude to a position where flight over mountains could be dangerous? A. B. C. D.

Warm high. Cold high. Cold low. Warm depression.

108. In the southern hemisphere what wind effect would you expect when flying from a high pressure area towards a low pressure area at FL 100? A. B. C. D.

Wind from the left. Wind from the right. Headwind with no drift. Tailwind with no drift.

109. What degree of turbulence, if any, is likely to be encountered while flying through a cold front in the summer over Central Europe at FL 100? A. B. C. D.

Light turbulence in CB cloud. Moderate turbulence in NS cloud. Severe turbulence in CB cloud. Light turbulence in ST cloud.

110. You are flying at 2 500 FT/AGL, the wind is 180°, and intend to land at an airport, at sea level directly below. From approximately which direction would you expect the surface wind (mid-latitude, northern hemisphere)? A. B. C. D.

South-southeast. South-southwest. South. Southwest.

111. What are the typical differences with regard to the temperature and humidity between an air mass with its origin in the Azores and an air mass with its origin over northern Russia ? A. B. C. D.

The air of the Azores is warmer and dryer than the North-Russian air. The air of the Azores is warmer and more humid than the North-Russian air. The North-Russian air is colder and more humid than the air of the Azores. The North-Russian air is warmer and dryer than the air of the Azores.

112. Where is the source of continental tropical air that affects Europe in summer? A. B. C. D.

The southern Balkan region and the Near East. Southern Italy. The Azores region. Southern France.

113. Where does polar continental air originate? A. B. C. D.

Siberian landmass. Polar ice cap. The region of the British Isles. The region of the Azores.

114. In which air mass are extremely low temperatures encountered? A. Continental polar air.

ATP Online - Meteorology

Page 13 of 213

B. Maritime arctic air. C. Maritime polar air. D. Continental tropical air.

115. With what type of clouds are showers most likely associated? A. B. C. D.

Nimbostratus. Cumulonimbus. Stratus. Stratocumulus.

116. At what time of day, or night, is radiation fog most likely to occur? A. B. C. D.

Shortly after midnight. Late evening. At sunset. Shortly after sunrise.

117. What is the average vertical extent of radiation fog? A. B. C. D.

10 000 FT. 500 FT. 5 000 FT. 2 000 FT.

118. Which of the following weather conditions favour the formation of radiation fog? A. B. C. D.

Light wind, extensive cloud, moist air. Strong wind, little or no cloud, moist air. Light wind, little or no cloud, moist air. Light wind, extensive cloud, dry air.

119. What type of fronts are most likely to be present during the winter in Central Europe when temperatures close to the ground are below 0°C, and freezing rain starts to fall? A. B. C. D.

Cold occlusions. High level cold fronts. Warm fronts, warm occlusions. Cold fronts.

120. Which of the following conditions are you most likely to encounter when approaching an active warm front at medium to low level ? A. B. C. D.

Low cloud base and poor visibility. Extreme turbulence and severe lightning striking the ground. High cloud base, good surface visibility, and isolated thunderstorms. Severe thunderstorms at low altitude.

121. During a cross-country flight at FL 50, you observe the following sequence of clouds: Nimbostratus, Altostratus, Cirrostratus, Cirrus. Which of the following are you most likely to encounter ? A. B. C. D.

A strong downdraught. Decreasing temperatures. Increasing temperatures. Strong, gusty winds.

122. In case of fronts, what cloud formation is most likely to occur at low levels when a warm air mass overrides a cold air mass? A. B. C. D.

Cumulus. Cumulonimbus. Nimbostratus. Altostratus.

123. The approximate inclined plane of a warm front is: A. B. C. D.

1/300 1/500 1/50 1/150

124. What type of low pressure area is associated with a surface front? A. B. C. D.

A cold air pool. A low on lee side of a mountain. Polar front low. Heat low.

ATP Online - Meteorology

Page 14 of 213

125. In which approximate direction does the centre of a non-occluded frontal depression move? A. B. C. D.

In the direction of the warm sector isobars. In the direction of the isobars ahead of the warm front. In the direction of the isobars behind the cold front. In the direction of the sharpest pressure increase.

126. Where is the coldest air to be found, in an occlusion with cold front characteristics? A. B. C. D.

Ahead of the front. Behind the front. At the surface position of the front. At the junction of the occlusion.

127. What type of front / occlusion usually moves the fastest? A. B. C. D.

Cold occlusion. Warm front. Cold front. Warm occlusion.

128. Over Central Europe what type of cloud cover is typical of the warm sector of a depression during winter? A. B. C. D.

CI, CS. ST. Fair weather CU. CU, CB.

129. What weather conditions are prevalent during the summer, over the North Sea, approximately 300 km behind a quickly moving cold front? A. B. C. D.

Rain covering a large area, 8 octas NS. Cloud cover mostly scattered, isolated showers. Advection fog. 8 octas CS, AS without precipitation.

130. What is the surface visibility most likely to be in a warm sector of maritime tropical air during a summer afternoon in western Europe? A. B. C. D.

Moderate (several km). Very poor (less than 1 km). Good (greater than 10 km). Very good (greater than 50 km).

131. Which one of the following statements regarding the intertropical convergence zone (ITCZ) is correct? A. B. C. D.

Thunderstorms seldom occur within the area of the ITCZ. The ITCZ is always associated with a strong jet stream. Frequent and widespread thunderstorms are to be expected within the area of the ITCZ. The ITCZ does not change its position during the course of the year.

132. In which of the following bands of latitude is the intertropical convergence zone most likely to be encountered in January, between Dakar and Rio de Janeiro? A. B. C. D.

3° - 8°S. 7° - 12°N. 8° - 12°S. 0° - 7°N.

133. What weather conditions are indications of the summer monsoon in India? A. B. C. D.

Thunderstorms, showers of heavy rain. Fog. Sandstorms. Stratus clouds and drizzle.

134. After passing at right angles through a very active cold front in the direction towards the cold air, what will you encounter at FL 50, in the northern hemisphere immediately after a marked change in temperature? A. B. C. D.

A backing in the wind direction. A veering in the wind direction. A decrease in tailwind. An increase in tailwind.

135. When are the rainy seasons in equatorial Africa?

ATP Online - Meteorology A. B. C. D.

Page 15 of 213

March to May and August to October. December to February and July to October. April to July and December to February. March to May and October to November.

136. Which of the following best describes the intertropical convergence zone ? A. B. C. D.

The zone where cold fronts form in the tropics. The zone where the Harmattan meets the north-easterly trade winds over Africa. The zone where the trade winds of the northern hemisphere meet those of the southern hemisphere. The zone where the west winds meet the subtropical high pressure belt.

137. What is the likely track for a hurricane in the Caribbean area? A. B. C. D.

West deep into the USA. East then south. West in the earlier stages and later turning north east. West in the earlier stages and later turning south east.

138. During which seasons are hurricanes most likely to appear in the northern hemisphere? A. B. C. D.

Winter. Summer and autumn. Winter and spring. All seasons.

139. What is encountered during the summer, over land, in the centre of a cold air pool? A. B. C. D.

Strong westerly winds. Sky clear (SKC). Fine weather CU. Showers and thunderstorms.

140. How do you recognize a cold air pool? A. B. C. D.

A cold air pool may only be recognized on the surface chart as a low pressure area. A cold air pool may only be recognized on the surface chart as a high pressure area. As a high pressure area aloft (e.g. on the 500 hPa chart). As a low pressure area aloft (e.g. on the 500 hPa chart).

141. What type of air movement is associated with the centre line of a trough? A. B. C. D.

Divergence with lifting. Convergence with descending air. Divergence with descending air. Convergence with lifting.

142. What is the correct term for the descending air flow in a large high pressure area? A. B. C. D.

Convection. Subsidence. Advection. Convergence.

143. In mid-latitudes, what surface weather is associated with a stationary high pressure region over land in the winter? A. B. C. D.

Thunderstorms. The possibility of snow showers. A tendency for fog and low ST. NS with continuous rain.

144. On which of the following aviation weather charts can a pilot most easily find a jetstream? A. B. C. D.

Significant weather chart. Wind / temperature chart. Upper air chart. Surface chart.

145. Why are indications about the height of the tropopause not essential for flight documentation in the tropics? A. B. C. D.

The meteorological services are unable to provide such a chart. The tropopause is generally well above the flight level actually flown. The temperatures of the tropical tropopause are always very low and therefore not important. The tropopause is always at the same height.

ATP Online - Meteorology

Page 16 of 213

146. The QNH of an airport at sea level is 983 hPa and the temperature deviation from ISA is -15°C below FL 100. What is the true altitude of FL 100? A. B. C. D.

9740 ft 8640 ft 10160 ft 11460 ft

147. In the TAF for Athens, during the summer, for the time of your landing you note: TEMPO TS. What is the maximum time this deterioration in weather can last in anyone instance ? A. B. C. D.

20 minutes. 60 minutes. 120 minutes. 10 minutes.

148. How are well separated CB clouds described on the Significant Weather Chart? A. B. C. D.

FRQ CB. OCNL CB. ISOL CB. EMBD CB.

149. The cold front is indicated with a number at position: A. B. C. D.

4 1 3 2

150. Of the four radio soundings, select the one that indicates ground fog: A. B. C. D.

2 1 3 4

ATP Online - Meteorology

151. What does the symbol indicate on a significant weather chart? A. B. C. D.

The lower limit of the tropopause. The upper limit of significant weather at FL 400. The center of a tropopause "high", where the tropopause is at FL 400. The center of a high pressure area at 400 hPa.

152. Which typical weather situation is shown on the weather chart ? (Spacing of the isobars: 5 hPa) A. B. C. D.

West wind condition. Warm south wind condition (Foehn). Flat pressure pattern. Cutting wind.

153. Which one of the tracks (dashed lines) is represented by the cross-section shown on the left ? A. B. C. D.

Track D-A Track B-C Track B-A Track C-A

Page 17 of 213

ATP Online - Meteorology

154. Which cross-section of air mass and cloud presentation is applicable to the straight line A-B? A. B. C. D.

3 2 4 1

155. Refer to the diagram. Assuming the usual direction of movement, where will this polar frontal wave have moved ? A. B. C. D.

Position 1 Position 3 Position 2 Position 4

156. What is the classification of the airmass affecting position "Q" at 0600 UTC? A. B. C. D.

Tropical maritime.; Polar maritime.; Polar continental.; Tropical continental.;

Page 18 of 213

ATP Online - Meteorology

Page 19 of 213

157. What information is required to calculate a lowest usable flight level from a given minimum safe altitude? A. B. C. D.

Highest value of QNH and the highest negative temperature deviation from ISA. Lowest value of QNH and the highest negative temperature deviation from ISA. Highest value of QNH and the highest positive temperature deviation from ISA Lowest value of QNH and the lowest negative temperature deviation from ISA

158. What is the relationship, if any, between QFE and QNH at an airport situated 50 FT below sea level? A. B. C. D.

QFE equals QNH. QFE is smaller than QNH. No clear relationship exists. QFE is greater than QNH.

159. You plan a flight over a mountain range at a true altitude of 15000 FT/AMSL. The air is on an average 15°C colder than ISA, the pressure at sea level is 1003 hPa. What approximate indication should the altimeter (setting 1013.2 hPa) read? A. B. C. D.

15690 FT. 16230 FT. 13830 FT. 14370 FT.

160. During a flight at FL 100 from Marseille (QNH 1012 hPa) to Palma de Mallorca (QNH 1015 hPa), an aircraft remains at a constant true altitude. The reason for this is that: A. B. C. D.

the altimeters are erroneous, and need to be tested. the air at Marseille is warmer than that at Palma de Mallorca. one of the two QNH values may be incorrect. the air at Marseille is colder than that at Palma de Mallorca.

161. An aircraft lands at an airport (airport elevation 1240 FT, QNH 1008 hPa). The altimeter is set to 1013 hPa. The altimeter will indicate: A. B. C. D.

1280 FT. 1200 FT. 1105 FT. 1375 FT.

162. At what time of the year are typhoons most likely to occur over the southern islands of Japan? A. B. C. D.

January to May. May to July. July to November. September to January.

163. What units are used to report vertical wind shear? A. B. C. D.

m/sec. m/100 FT. kt. kt/100 FT.

164. Which of the following weather reports is a warning of conditions that could be potentially hazardous to aircraft in flight ? A. B. C. D.

ATIS. SPECI. TAF. SIGMET.

ATP Online - Meteorology

Page 20 of 213

165. In which of the following circumstances is a SIGMET issued ? A. B. C. D.

Clear ice on the runways of an aerodrome. Fog or a thunderstorm at an aerodrome. Severe mountain waves. A sudden change in the weather conditions contained in the METAR.

166. After landing at an aerodrome (aerodrome elevation 1715 FT), the altimeter indicates an altitude of 1310 FT. The altimeter is set to the pressure value of 1013 hPa. What is the QNH at this aerodrome? A. B. C. D.

998 hPa. 1028 hPa. 1013 hPa. 1015 hPa.

167. You intend to overfly a mountain range. The recommended minimum flight altitude is, according to the aviation chart, 15000 FT/AMSL. The air mass that you will fly through is on average 15°C warmer than the standard atmosphere. The altimeter is set to QNH (1023 hPa). At what altimeter reading will you effectively be at the recommended minimum flight altitude? A. B. C. D.

15900 FT. 14370 FT. 13830 FT. 14100 FT.

168. You are flying at FL 130, and your true altitude is 12000 FT. What is the temperature deviation from that of the standard atmosphere at FL 130 (QNH 1013,2 hPa) ? A. B. C. D.

ISA -20°C ISA +/-0°C ISA +12°C ISA +20°C

169. Rime ice forms through the freezing onto aircraft surfaces of A. B. C. D.

snow. large supercooled water drops. small supercooled water drops. water vapour.

170. How does a pilot react to heavy freezing rain at 2000 FT/AGL, when he is unable to deice, nor land? A. B. C. D.

He turns back before the aircraft loses manoeuvrability. He descends to the warm air layer below. He ascends to the cold air layer above. He continues to fly at the same altitude.

171. What name is given to the jet stream lying across India (A) ? A. B. C. D.

Arctic jet stream. Polar front jet stream. Tropical jet stream. Sub-tropical jet stream.

ATP Online - Meteorology

Page 21 of 213

172. An aircraft is flying through the polar front jet stream from south to north, beneath the core. How would the OAT change, in the northern hemisphere, during this portion of the flight? A. B. C. D.

It decreases. It remains constant. It increases. It first increases, then decreases.

173. What is the average height of the arctic jet stream core? A. B. C. D.

30000 FT. 20000 FT . 50000 FT. 40000 FT.

174. What is the approximate ratio between height and width for a jet stream cross section? A. B. C. D.

1/1000 1/100 1/10 1/1

175. Which jet stream blows all year round, over the northern hemisphere? A. B. C. D.

The subtropical jet stream. The polar night jet stream. The arctic jet stream. The equatorial jet stream.

176. What is the average height of the jet core within a polar front jet stream? A. B. C. D.

50000 FT. 30000 FT. 40000 FT. 20000 FT.

177. An aircraft is flying from south to north, above the polar front jet stream, at FL 400 in the southern hemisphere. What change, if any, in temperature will be experienced ? A. B. C. D.

It decreases and then increases. It stays the same. It decreases. It increases.

178. A wind speed of 350 kt within a jet stream core should be world-wide regarded as: A. not unusual in polar regions. B. not possible.

ATP Online - Meteorology

Page 22 of 213

C. a common occurrence. D. possible but a very rare phenomenon.

179. Does the following report make sense? METAR LSZH 182320Z VRB02KT 5000 MIFG 02/02 Q1015 NOSIG A. B. C. D.

The report is nonsense, because it is impossible to observe a visibility of 5 km if shallow fog is reported. The report is not possible, because, with a temperature of 2°C and a dew point of 2°C there must be uniform fog. The report would never be seen, because shallow fog is not reported when the visibility is more than 2 km. The report is possible, because shallow fog is defined as a thin layer of fog below eye level.

180. You receive the following METAR: LSGG 120750Z 00000KT 0300 R05/0700N FG VV001 M02/M02 Q1014 NOSIG = What will be the RVR at 0900 UTC? A. B. C. D.

300 m. 700 m. 900 m. The RVR is unknown, because the "NOSIG" does not refer to RVR.

181. An aircraft over Western Europe is crossing a jet stream 2500 FT below its core at right angles. While crossing, the outside temperature is increasing. The prevailing wind is A. B. C. D.

from the left. tailwind. headwind. from the right

182. Which of the following cloud types can project up into the stratosphere? A. B. C. D.

Cumulonimbus Cirrostratus Altocumulus Altostratus

183. Which one of the following statements applies to the tropopause? A. B. C. D.

It is, by definition, a temperature inversion It separates the troposphere from the stratosphere It is, by definition, an isothermal layer It indicates a strong temperature lapse rate

184. The 0° isotherm is forecast to be at FL 50. At what FL would you expect a temperature of -6° C? A. B. C. D.

FL 20 FL 110 FL 100 FL 80

185. What positions are connected by isobars on the surface weather chart? A. B. C. D.

Positions with the same wind velocity at a given level Positions with the same relative pressure heights Positions with the same air pressure at a given level Positions with the same temperature at a given level

186. A vertical spacing of 1000 FT is the standard required separation between two FL. Under conditions of cold air advection (ISA -15°C), what would the true vertical separation be? A. B. C. D.

Without QNH information, it can not be determined More than 1000 FT It remains 1000 FT Less than 1000 FT

187. At which pressure and temperature conditions may you safely assume that the minimum usable flight level at least lies at the same height, as the minimum safe altitude? A. B. C. D.

At a temperature greater than or equal to that of the ISA and where the QNH is greater than or equal to 1013.25 hPa In a cold low pressure region In a very cold area with a QNH of 1015 hPa At a temperature less than or equal to that of the ISA and where the QNH is less than 1013.25 hPa

188. What is the main cause for the formation of a polar front jet stream? A. B. C. D.

Strong winds in the upper atmosphere The pressure difference, close to the ground, between a high over the Azores and a low over Iceland The north-south horizontal temperature gradient at the polar front The varied elevations of the tropopause in the polar front region

ATP Online - Meteorology

Page 23 of 213

189. Which jet stream is connected with a surface front system? A. B. C. D.

The easterly jet stream The equatorial jet stream The polar front jet stream The subtropical jet stream

190. At approximately what flight level is the subtropical jet stream found? A. B. C. D.

FL 200 FL 500 FL 400 FL 300

191. A parcel of rising air stays unsaturated. Which of the following statements is correct? A. B. C. D.

Relative humidity decreases, mixing ratio remains unchanged Relative humidity and mixing ratio remain unchanged Relative humidity increases, temperature decreases Mixing ratio increases, temperature decreases

192. In which of the following changes of state is latent heat released ? A. B. C. D.

Solid to liquid Solid to gas Gas to liquid Liquid to gas

193. How are high level condensation trails formed that are to be found occasionally behind jet aircraft ? A. B. C. D.

Through water vapour released during fuel combustion Through a decrease in pressure, and the associated adiabatic drop in temperature at the wing tips while flying through relatively warm but humid air In conditions of low humidity, through the particles of soot contained in the exhaust gases Only through unburnt fuel in the exhaust gases

194. What process in an air mass leads to the creation of wide spread NS and AS cloud coverage? A. B. C. D.

Sinking Lifting Convection process Radiation

195. Which of the following cloud is classified as low level cloud ? A. B. C. D.

AS CS CC ST

196. In which of the following conditions is moderate to severe airframe icing most likely to be encountered? A. B. C. D.

In clear air above the freezing level In Nimbostratus cloud Below the freezing level in clear air Within cloud of any type

197. What flying conditions are normally encountered when flying in cirrus clouds? A. B. C. D.

Average horizontal visibility less than 500 m; nil icing. Average horizontal visibility more than 1000 m; light to moderate rime ice. Average horizontal visibility more than 1000 m; nil icing. Average horizontal visibility less than 500 m; light to moderate icing.

198. Which of the following is most likely to lead to the dissipation of radiation fog ? A. B. C. D.

A marked decrease in wind velocity close to the ground A marked increase in wind velocity near the ground A build up of a high pressure area resulting in adiabatic warming associated with a sinking air mass Ground cooling caused by radiation during the night

199. Which of the following conditions is most likely to lead to the formation of steam fog (arctic smoke)? A. The sea is warmed by strong radiation from the sun B. The coastal region of the sea cools at night

ATP Online - Meteorology

Page 24 of 213

C. Cold air moving over warm water D. Warm air moving over cold water

200. How does freezing rain develop? A. B. C. D.

Rain falls through a layer where temperatures are below 0°C Through melting of ice crystals Through melting of snow grains Snow falls through a layer where temperatures are above 0°C

201. What type of cloud can produce hail showers? A. B. C. D.

AC CB NS CS

202. In which of the following regions does maritime polar air originate? A. B. C. D.

Baltic Sea Black Sea Region of British Isles East of Greenland

203. In which of the following situations can freezing rain be encountered ? A. B. C. D.

Ahead of a cold front in the summer Behind a warm front in the summer Ahead of a warm front in the winter Ahead of a cold front in the winter

204. How do air masses move at a warm front ? A. B. C. D.

Warm air undercuts a cold air mass Cold air overrides a warm air mass Cold air undercuts a warm air mass Warm air overrides a cold air mass

205. What type of precipitation would you expect at an active unstable cold front? A. B. C. D.

Freezing rain Drizzle Showers associated with thunderstorms Light to moderate continuous rain

206. What is the relative movement of the two air masses along a cold front ? A. B. C. D.

Warm air pushes over a cold air mass Cold air pushes under a warm air mass Warm air pushes under a cold air mass Cold air slides over a warm air mass

207. What cloud cover is typical for a wide warm sector of a polar front depression over Central Europe in the summer ? A. B. C. D.

Fair weather CU Sky clear BKN CU and CB ST with drizzle

208. Which of the following describes a warm occlusion? A. B. C. D.

The warmer air mass is ahead of the original warm front The air mass ahead of the front is drier than the air mass behind the front The coldest air mass is ahead of the original warm front The air mass behind the front is more unstable than the air mass ahead of the front

209. When do cold occlusions occur most frequently in Europe? A. B. C. D.

Summer Winter and spring Winter Autumn and winter

210. In which main direction does a polar front depression move?

ATP Online - Meteorology A. B. C. D.

Along the front towards the east Across the front towards the south Across the front towards the north Along the front towards the west

211. What change in pressure, will occur at point A, during the next hour? A. B. C. D.

Irregular fluctuations Approximately constant pressure A drop in pressure A rise in pressure

212. What is the most likely cause of a lack of clouds at higher levels in a stationary high? A. B. C. D.

Sinking air Divergence at higher levels Instability Rising air

213. On which coast of North America, is the danger of tropical revolving storms the greatest? A. B. C. D.

N coast W coast SE coast NE coast

214. What is characteristic of the pamperos? A. B. C. D.

A marked advance of cold air in South America A marked advance of cold arctic air in North America Katabatic winds in the Atlas Mountains Foehn conditions in the Spanish Pyrenees

215. Where, during a flight from Marseille to Dakar, in July, may the ITCZ be encountered? A. B. C. D.

At the latitudes of Algeria Near the Canary Islands In the vicinity of Dakar At the latitudes of Gibraltar

216. Which wind systems converge on the ITCZ, when it lies at the equator? A. B. C. D.

SE trade winds and NE trade winds NW monsoon and SW trade winds SW monsoon and NW monsoon SW monsoon and NW trade winds

217. From which direction do the trade winds blow, in the southern hemisphere? A. SW B. N

Page 25 of 213

ATP Online - Meteorology

Page 26 of 213

C. NE D. SE

218. What weather conditions in the region of the Alps would you expect with Foehn from south? A. B. C. D.

Clouds, on the southern sides of passes in the Alps Heavy clear air turbulence on the southern side of the Alps Heavy airframe icing conditions on the northern side of the Alps Strong north winds on the southern side of the Alps

219. At what temperatures would you expect the heaviest ice accretion to occur in a CB? A. B. C. D.

Close to the freezing level Between -2°C and -15°C Between -20°C and -30°C Between -30°C and -40°C

220. Clear ice forms on an aircraft by the freezing of: A. B. C. D.

large supercooled water drops snow small supercooled water drops water vapour

221. What weather conditions would you expect at a squall line? A. B. C. D.

Continuous heavy rain Fog Strong whirlwinds reaching up to higher levels Thunderstorms

222. What is the approximate maximum diameter of a the area affected by damaging winds at the surface caused by a microburst ? A. B. C. D.

20 km 50 km 400 m 4 km

223. In general, the visibility during rainfall compared to during drizzle is A. B. C. D.

less greater the same in rain - below 1 km, in drizzle - more than 2 km

224. Which of the following symbols represents a severe squall line? A. B. C. D.

Symbol 4) Symbol 2) Symbol 1) Symbol 3)

225. What is a trend forecast? A. B. C. D.

A landing forecast appended to METAR/SPECI, valid for 2 hours A routine report A route forecast valid for 24 hours An aerodrome forecast valid for 9 hours

226. What does the abbreviation "BKN" mean?

ATP Online - Meteorology A. B. C. D.

Page 27 of 213

Nil significant cloud cover 3-5 Eights of the sky is cloud covered 3-4 Eights of the sky is cloud covered 5-7 Eights of the sky is cloud covered

227. What does the abbreviation "nosig" mean? A. B. C. D.

No weather related problems No significant changes No report received Not signed by the meteorologist

228. In which weather report would you expect to find information about icing conditions on the runway? A. B. C. D.

SIGMET GAFOR TAF METAR

229. In which of the following 1850 UTC METAR reports, is the probability of fog formation, in the coming night, the highest? A. B. C. D.

00000KT 9999 SCT300 21/01 Q1032 NOSIG = 22004KT 6000 -RA SCT012 OVC030 17/14 Q1009 NOSIG = VRB01KT 8000 SCT250 11/10 Q1028 BECMG 3000 BR = VRB02KT 2500 HZ SCT120 14/M08 Q1035 NOSIG =

230. Which of the following extracts of weather reports could be, in accordance with the regulations, abbreviated to "CAVOK"? (MSA minus airport elevation equals: LSZB 10000 FT, LSZH 8000 FT, LSGG 12000 FT, LFSB 6000 FT) A. B. C. D.

LSGG 22003KT 9999 SCT120 BKN280 09/08 Q1026 BECMG 5000 BR = LFSB 00000KT 9000 SCT080 22/15 Q1022 NOSIG = LSZH 26024G52KT 9999 BKN060 17/14 Q1012 RETS TEMPO 5000 TSRA = LSZB 30004KT 9999 SCT090 10/09 Q1006 NOSIG =

231. Within a short interval, several flight crews report that they have experienced strong clear air turbulence in a certain airspace. What is the consequence of these reports? A. B. C. D.

The competent aviation weather office will issue a SIGMET The competent aviation weather office will issue a storm warning The airspace in question, will be temporarily closed The competent aviation weather office will issue a SPECI

232. At which airport, is the following weather development taking place? TAF 060600Z 060716 25006KT 8000 BKN240 BECMG 0710 OVC200 BECMG 1013 23010KT 8000 OVC100 BECMG 1316 23014KT 6000 RA SCT030 OVC050 = A. B. C. D.

LFPO EDDL LOWW LEMD

233. What weather conditions are expected at Paris airport (LFPO) around 0550 UTC? A. B. C. D.

20004KT 8000 SCT110 SCT250 22/08 Q1016 NOSIG = 26012KT 9999 SCT025 SCT040 14/09 Q1018 TEMPO 5000 SHRA = 22020G36KT 1500 TSGR SCT004 BKN007 BKN025CB 18/13 Q1009 BECMG NSW = 23014KT 3000 +RA SCT008 OVC025 15/13 Q1004 NOSIG =

ATP Online - Meteorology

234. Over Madrid, what intensity of turbulence and icing is forecast at FL 200 ? A. B. C. D.

Moderate turbulence, moderate icing Severe turbulence, severe icing Moderate turbulence, light icing Severe turbulence, moderate icing

235. Which airport, at 1200 UTC, has the lowest probability of precipitation? A. B. C. D.

LSZH EFHK ENFB ESSA

236. Over Paris at what flight level would you expect to find the tropopause according to the map? A. FL 280 B. FL 300

Page 28 of 213

ATP Online - Meteorology

Page 29 of 213

C. FL 150 D. FL 330

237. What is the optimum flight level between Rome and Paris according to the significant weather chart? A. B. C. D.

FL 360 FL 340 FL 160 FL 220

238. Flight Zurich to Rome, ETD 1600 UTC, ETA 1800 UTC. At what flight level would you first expect to encounter clear air turbulence on the climb out from Zurich? A. B. C. D.

FL 140 FL 220 FL 320 FL 160

ATP Online - Meteorology

Page 30 of 213

239. What is the approximate height of the tropopause between Munich and Helsinki? A. B. C. D.

FL 300; FL 340; FL 390; FL 280;

240. Flight from Bordeaux to Amsterdam, ETA 2100 UTC. What lowest cloud base is forecast for arrival at Amsterdam?; TAF EHAM 281500Z 281601 14010KT 6000 -RA SCT025 BECMG 1618 12015G25KT SCT008 BKN013 TEMPO 1823 3000 RA BKN005 OVC010 BECMG 2301 25020KT 8000 NSW A. B. C. D.

500 FT 800 FT 250 FT 500 m

241. You are flying from Munich to Amsterdam. Which of the following flight levels would you choose in order to avoid turbulence and icing? A. B. C. D.

FL 180 FL 260 FL 320 FL 140

ATP Online - Meteorology

242. In what height range and at what intensity could you encounter turbulence in CAT area n°2? A. B. C. D.

From FL 220 to FL 400, moderate From FL 250 to FL 320, moderate From FL 240 to FL 370, light From below FL 130 to FL 270, light

243. At what flight level is the jet stream core that is situated over northern Scandinavia ? A. B. C. D.

FL 280 FL 330 FL 360 FL 300

Page 31 of 213

ATP Online - Meteorology

Page 32 of 213

244. At which position is the development of thunderstorms most likely, and what is the maximum height of the CB clouds at 00 UTC? A. B. C. D.

Position A, FL 200. Position D, FL 290. Position B, FL 270. Position C, FL 200.

245. At what approximate flight level is the tropopause over Frankfurt? A. B. C. D.

FL 330 FL 300 FL 240 FL 350

ATP Online - Meteorology

246. Select from the map the wind for the route Zurich - London at FL 280. A. B. C. D.

040 / 60 160 / 90 220 / 60 250 / 80

247. The temperature at FL 330 overhead London will be A. B. C. D.

-39°C -57°C -45°C -33°C

Page 33 of 213

ATP Online - Meteorology

248. The front labelled "Z" is a: A. B. C. D.

Cold occlusion Warm front Cold front Warm occlusion

249. What name is given to the jet stream lying over North Africa (B) ? A. B. C. D.

Sub-tropical jet stream Equatorial jet stream Arctic jet stream Polar front jet stream

Page 34 of 213

ATP Online - Meteorology

Page 35 of 213

250. Flight Lisbon to Bordeaux, ETA 1800 UTC. What type of precipitation is forecast on the approach to Bordeaux ?; TAF LFBD 281400Z 281524 26015KT 9999 SHRA BKN020 TEMPO 1620 26020G30KT; 8000 +SHRA BKN015CB PROB30 TSRA = A. B. C. D.

Moderate snow showers Continuous moderate rain Heavy rain showers Light drizzle and fog

251. Flight Lisbon to Bordeaux, ETA 1800 UTC. At ETA Bordeaux what is the lowest quoted visibility forecast ?; TAF LFBD 281400Z 281524 26015KT 9999 SHRA BKN020 TEMPO 1620 26020G30KT; 8000 +SHRA BKN015CB PROB30 TSRA = A. B. C. D.

8 NM 10 or more km 10 NM 8 km

252. What wind is forecast at FL 390 over Paris ? A. B. C. D.

210/40 030/40 190/40 240/20

253. What is the average temperature at FL 160 between Oslo and Paris ? A. B. C. D.

-15°C -19°C -25°C -23°C

ATP Online - Meteorology

254. What is the temperature deviation in degrees Celsius, from the ICAO Standard Atmosphere overhead Frankfurt ? A. B. C. D.

ISA +13°C ISA -2°C ISA +2°C ISA -13°C

255. What is the speed of the front located over France ? A. B. C. D.

30 kt 25 kt 10 kt 15 kt

Page 36 of 213

ATP Online - Meteorology

Page 37 of 213

256. Flight Munich to London. What is the direction and maximum speed of the jet stream affecting the route between Munich and London ? A. B. C. D.

050° / 120 kt 050° / 120 km/h 220° / 120 kt 230° / 120 m/sec

ATP Online - Meteorology

Page 38 of 213

257. Flight from Bordeaux to Amsterdam, ETA 2100 UTC. At ETA Amsterdam what surface wind is forecast ?; TAF EHAM 281500Z 281601 14010KT 6000 -RA SCT025 BECMG 1618 12015G25KT; SCT008 BKN013 TEMPO 1823 3000 RA BKN005 OVC010 BECMG 2301 25020KT 8000; NSW BKN020 = A. B. C. D.

250° / 20 kt 120° / 15 kt gusts 25 kt 300° / 15 kt maximum wind 25 kt 140° / 10 kt

258. Flight from Bordeaux to Amsterdam, ETA 2100 UTC. What is the minimum visibility forecast for ETA Amsterdam ?; TAF EHAM 281500Z 281601 14010KT 6000 -RA SCT025 BECMG 1618 12015G25KT; SCT008 BKN013 TEMPO 1823 3000 RA BKN005 OVC010 BECMG 2301 25020KT 8000; NSW BKN020 = A. B. C. D.

6 km 3 km 5 NM 5 km

259. Flight Shannon to London. What amount and type of cloud is forecast for the eastern sector of the route between Shannon and London at FL 220 ? A. B. C. D.

Scattered castellanus Individual cumulonimbus Scattered towering cumulus Overcast nimbo layered cumulonimbus

260. Over Amsterdam, what amount and general type of cloud would you expect at FL 160 ? A. Isolated cumulonimbus only B. 5 to 7 oktas towering cumuliform cloud and with moderate turbulence

ATP Online - Meteorology C. 4 oktas broken cumulus D. Mainly 5 to 8 oktas of stratiform cloud in layers

261. To what extent is Munich covered by clouds ? A. B. C. D.

1 to 4 oktas 5 to 8 oktas 5 to 7 oktas 3 to 5 oktas

262. What OAT would you expect at FL 200 over Geneva ? A. B. C. D.

-16°C -20°C -24°C -28°C

Page 39 of 213

ATP Online - Meteorology

Page 40 of 213

263. 8/8 stratus base 200 FT/AGL is observed at sunrise at an aerodrome in the north of France; the QNH is 1028 hPa and there is a variable wind of 3 kt. What change in these clouds is likely at 1200 UTC in summer and winter? A. B. C. D.

Winter: BKN, base 2500 FT/AGL; summer: BKN, base 3500 FT/AGL. Winter: OVC, base 500 FT/AGL; summer: SCT, base 3000 FT/AGL. Winter: clear sky; summer: BKN CB, base 1500 FT/AGL. Winter: SCT, base 3000 FT/AGL; summer: OVC, base 500 FT/AGL.

264. The lowest visibility forecast at ETA Zurich 1430 UTC is:; LSZH 261019 20018G30KT 9999 -RA SCT050 BKN080 TEMPO 23012KT 6000 -DZ BKN015 BKN030 BECMG 1518 23020G35KT 4000 RA OVC010=; A. B. C. D.

6 NM. 10 km. 4 km. 6 km.

265. The lowest cloud base forecast at ETA Zurich (1200 UTC) is:; LSZH 061019 20018G30KT 9999 -RA SCT050 BKN080 TEMPO 23012KT 6000 -DZ BKN015 BKN030 BECMG 1518 23020G35KT 4000 RA OVC010=; A. B. C. D.

1000 FT. 5000 FT. 1500 m. 1500 FT.

266. The following weather message EDDM 241200Z 241322 VRB03KT 1500 BR OVC004 BECMG 1517 00000KT 0500 FG VV002 TEMPO 2022 0400 FG VV001 is a: A. B. C. D.

24 hour TAF. 9 hour TAF. METAR. SPECI.

267. In METAR messages, the pressure group represents the A. B. C. D.

QNH rounded up to the nearest hPa. QNH rounded down to the nearest hPa. QFE rounded down to the nearest hPa. QFE rounded to the nearest hPa.

268. An altimeter adjusted to 1013 hPa indicates an altitude of 3600 FT. Should this altimeter be adjusted to the local QNH value of 991 hPa, the altitude indicated would be A. B. C. D.

3006 FT. 4194 FT. 4278 FT. 2922 FT.

269. In Geneva, the local QNH is 994 hPa. The elevation of Geneva is 1411 FT. The QFE adjustment in Geneva is A. B. C. D.

961 hPa. 967 hPa. 942 hPa. 948 hPa.

270. An aircraft is flying at FL 80. The local QNH is 1000 hPa. After the second altimeter has been adjusted to the local QNH, the reading will be approximately A. 8000 FT.

ATP Online - Meteorology

Page 41 of 213

B. 8350 FT. C. 7650 FT. D. 8600 FT.

271. The barometric compensator of an altimter is locked on reference 1013.2 hPa. The aircraft has to land on a point with an elevation of 290 feet where the QNH is 1023 hPa. The reading on the altimeter on the ground will be: A. B. C. D.

20 FT. 11 FT. 560 FT. -10 FT.

272. A microburst phenomenon can arise in the A. B. C. D.

updraught of a cumulonimbus at the mature stage. updraught of a cumulonimbus at the growth stage. downdraught of a cumulonimbus at the formation stage. downdraught of a cumulonimbus at the mature stage.

273. While approaching your destination aerodrome you receive the following message: RVR runway 23: 400m This information indicates the A. B. C. D.

length of runway which a pilot in an aircraft on the ground would see, on the threshold of runway 23. visibility on runway 23. portion of runway which a pilot on the threshold of any of the runways would see, with runway 23 in service. minimum visibility at this aerodrome, with runway 23 being the one in service.

274. What is the meaning of the abbreviation "SCT" ? A. B. C. D.

1 - 2 oktas 1 - 4 oktas 5 - 7 oktas 3 - 4 oktas

275. An airborne weather radar installation makes it possible to detect the location of A. B. C. D.

stratocumulus and its vertical development cumulonimbus, but provided that cloud of this type is accompanied by falls of hail zones of precipitation, particularly liquid-state precipitation, and also their intensity all clouds

276. The upper wind and temperature chart of 250 hPa corresponds, in a standard atmosphere, to about A. B. C. D.

32 000 FT 34 000 FT 39 000 FT 30 000 FT

277. Marseille Information gives you the following meteorological information for Ajaccio and Calvi for 1600 UTC: Ajaccio: wind 360°/2 kt, visibility 2000 m, rain, BKN stratocumulus at 1000 FT, OVC altostratus at 8000 FT, QNH 1023 hPa. Calvi: wind 040°/2 kt, visibility 3000 m, mist, FEW stratus at 500 FT, SCT stratocumulus at 2000 FT, OVC altostratus at 9000 FT, QNH 1023 hPa. The ceilings (more than 4 oktas) are therefore: A. B. C. D.

8000 FT at Ajaccio and 9000 FT at Calvi 1000 FT at Ajaccio and 500 FT at Calvi 1000 FT at Ajaccio and 9000 FT at Calvi 1000 FT at Ajaccio and 2000 FT at Calvi

278. Extensive cloud and precipitation is often associated with a non frontal thermal depression because of: A. B. C. D.

surface divergence and upper level convergence causing widespread descent of air in the depression surface divergence and upper level convergence causing widespread ascent of air in the depression surface convergence and upper level divergence causing widespread descent of air in the depression surface convergence and upper level divergence causing widespread ascent of air in the depression

279. The equatorial easterly jet is a jet stream that occurs: A. B. C. D.

only in the winter of the northern hemisphere at approx. 30 000 FT during the whole year in the northern hemisphere only in the summer of the northern hemisphere at approx. 45 000 FT during the whole year in the southern hemisphere

280. All pilots encountering Clear Air Turbulence are requested to report it. You experience CAT which causes passengers and crew to feel definite strain against their seat belt or shoulders straps. Unsecured objects are dislodged. Food service and walking are difficult. This should be reported as:

ATP Online - Meteorology A. B. C. D.

Page 42 of 213

Moderate severe extreme light

281. On average, going from the equator to the north pole, the altitude of the tropopause A. B. C. D.

decreases and its temperature increases increases and its temperature decreases decreases and its temperature decreases increases and its temperature increases

282. The Harmattan is a A. B. C. D.

warm southerly dust-bearing wind affecting the coast of North Africa. NE wind affecting north-west Africa during November to April reducing visibility in rising dust. localised depression giving squally winds. SW monsoonal wind causing extensive areas of advection fog along the West African coast south of 15°N.

283. To which aerodrome is the following TAF most applicable ? TAF 231019 24014KT 6000 SCT030 BKN100 TEMPO 1113 25020G38KT 2500 +TSRA SCT008 BKN025CB BECMG 1315 28012KT 9999 SCT025 TEMPO 5000 SHRA BKN020 BECMG 1719 27008KT 9999 SCT030 A. B. C. D.

EKCH LEMD LOWW LFPG

284. What name is given to the low level wind system between the subtropical high pressure belt and the equatorial trough of low pressure (ITCZ) ? A. B. C. D.

Doldrums. Westerly winds. Monsoon. Trade winds.

285. In which zone of a polar front jet stream is the strongest CAT to be expected ? A. B. C. D.

About 12000 FT above the core. On the polar air side of the core. On the tropical air side of the core. Exactly in the centre of the core.

286. What is the minimum speed for a wind to be classified as a jet stream? A. B. C. D.

70 kt. 50 kt. 60 kt. 100 kt.

287. A wind sounding in the region of a polar front jet stream gives the following windprofile (Northern hemisphere).; 900hPa 220/20kt ; 800hPa 220/25kt ; 700hPa 230/35kt ; 500hPa 260/60kt ; 400hPa 280/85kt; 300hPa 300/100kt; 250hPa 310/120kt ; 200hPa 310/80kt; Which system is the jet stream associated with? A. B. C. D.

With a warm front. With a cold front. With a ITCZ. With an easterly wave.

288. Where is the most dangerous zone in a tropical revolving storm?

ATP Online - Meteorology A. B. C. D.

Page 43 of 213

About 600 km away from the eye. In the centre of the eye. In the wall of clouds around the eye. Anywhere in the eye.

289. In the weather briefing room during the pre-flight phase of a passenger flight from Zurich to Rome, you examine the following weather reports of pressing importance at the time:; EINN SHANNON 2808 sigmet 2 valid 0800/1100 loc sev turb fcst einn fir blw fl 050 south of 53n wkn =; LIMM MILANO 2809 sigmet 2 valid 0900/1500 mod sev cat btn fl 250 and fl 430 fcst limm fir stnr nc =; EGLL LONDON 2808 sigmet nr01 valid 0800/1200 for london fir isol cb embd in lyr cloud fcst tops fl 300 btn 52n and 54n east of 002e sev ice sev turb ts also fcst mov e wkn =; Which decision is correct? A. B. C. D.

You show no further interest in these reports, since they do not concern the route to be flown. Owing to these reports and taking into account the presence of heavy thunderstorms at planned FL 310 you select a higher flight level (FL 370). Because of the expected turbulence you select a flight level below FL 250. You cancel the flight since the expected dangerous weather conditions along the route would demand too much of the passengers.

290. In the month of August you prepare a flight (cruising level FL 370) from Bombay (19°N - 73°E) to Bangkok (13°N - 100°E). What wind conditions can you expect? A. B. C. D.

Light winds diagonal to the route. Strong northerly winds. Tailwinds. Headwinds.

291. What is the most significant difference between an equatorial jet stream and all the other jet streams ? A. B. C. D.

Wind direction. Wind speed. Vertical dimension. Horizontal dimension.

292. Which of the following types of jet streams can be observed all year round? A. B. C. D.

Equatorial jet stream / arctic jet stream. Equatorial jet stream / polar front jet stream. Arctic jet stream / subtropical jet stream. Subtropical jet stream / polar front jet stream.

293. Tropical revolving storms do not occur in the southeast Pacific and the south Atlantic mainly because A. B. C. D.

there is no coriolis force present. the southeast trade winds cross over into the northern hemisphere. of the low water temperature. of the strong southeast wind.

294. Refer to the following TAF extract: BECMG 1821 2000 BR BKN004 BECMG 2124 0500 FG VV001 What visibility is forecast for 2400 UTC? A. B. C. D.

Between 0 m and 1000 m. 500 m. 2000 m. Between 500 m and 2000 m.

295. Refer to the following TAF extract: BECMG 1821 2000 BR BKN004 BECMG 2124 0500 FG VV001 What does the abbreviation "VV001" mean? A. B. C. D.

RVR less than 100 m. RVR greater than 100 m. Vertical visibility 100 FT. Vertical visibility 100 m.

296. What is the main energy source of a tropical revolving storm? A. B. C. D.

Temperature difference between equatorial low pressure trough and subtropical high pressure belt. Latent heat released from condensing water vapour. Cold air advancing from temperate latitudes. The equatorial jet stream.

297. Which of the following meteorological phenomenon indicates upper level instability which may lead to thunderstorm development ? A. B. C. D.

Red cirrus. AC castellanus. AC lenticularis. Halo.

ATP Online - Meteorology

Page 44 of 213

298. Isolated thunderstorms of a local nature are generally caused by A. B. C. D.

frontal occlusion. frontal lifting (cold front). thermal triggering. frontal lifting (warm front).

299. Which thunderstorms move forward the fastest? A. B. C. D.

Frontal thunderstorms. Thunderstorms formed by lifting processes. Orographic thunderstorms. Thermal thunderstorms.

300. What does the term METAR signify? A. B. C. D.

A METAR signifies the actual weather report at an aerodrome and is generally issued in half-hourly intervals. A METAR is a landing forecast added to the actual weather report as a brief prognostic report. A METAR is a flight forecast, issued by the meteorological station several times daily. A METAR is a warning of dangerous meteorological conditions within a FIR.

301. How long from the time of observation is a TREND in a METAR valid? A. B. C. D.

1 hour. 2 hours. 30 minutes. 9 hours.

302. A zone of strong convection currents is encountered during a flight. In spite of moderate gust you decide to continue the flight. What are your precautionary measures? A. B. C. D.

Decrease the speed / try to descend below the zone of convective currents. Increase the speed / try to descend below the zone of convective currents. Increase the speed / try to climb above the zone of convective currents if aircraft performance parameters allow. Decrease the speed / try to climb above the zone of convective currents if aircraft performance parameters allow.

303. At which altitude, at temperate latitudes, may hail be expected in connection with a CB? A. B. C. D.

From the ground up to about FL 200. From the ground up to a maximum of FL 450. From the ground up to about FL 100. From the base of the clouds up to about FL 200.

304. What is a SPECI? A. B. C. D.

An aerodrome forecast issued every 9 hours. A routine aerodrome weather report issued every 3 hours. A warning of meteorological dangers at an aerodrome, issued only when required. A special aerodrome weather report, issued when a significant change of the weather conditions have been observed.

305. Refer to the following TAF extract: BECMG 1821 2000 BR BKN004 BECMG 2124 0500 FG VV001 What does the "BECMG" data indicate for the 18 to 21 hour time frame? A. B. C. D.

The new conditions are achieved between 1800 and 2100 UTC Many short term changes in the original weather. Many long term changes in the original weather. A quick change to new conditions between 1800 and 1900 UTC.

306. Refer to the following TAF extract: BECMG 1821 2000 BR BKN004 BECMG 2124 0500 FG VV001 What does the abbreviation "BKN004" mean? A. B. C. D.

5 - 7 oktas, ceiling 400 FT. 1 - 4 oktas, ceiling 400 m. 1 - 4 oktas, ceiling 400 FT. 4 - 8 oktas, ceiling 400 m.

307. Refer to the following TAF extract; BECMG 1821 2000 BR BKN004 PROB30 TEMPO 2124 0500 FG VV001 What does the abbreviation "PROB30" mean? A. B. C. D.

Conditions will last for at least 30 minutes. The cloud ceiling should lift to 3000 FT. Probability of 30%. Change expected in less than 30 minutes.

308. Where, as a general rule, is the core of the polar front jet stream to be found?

ATP Online - Meteorology A. B. C. D.

Page 45 of 213

Just above the warm-air tropopause. Just below the cold-air tropopause. In the polar air mass. In the tropical air mass.

309. How long does a typical microburst last? A. B. C. D.

About 30 minutes. 1 to 2 hours. 1 to 5 minutes. Less than 1 minute.

310. You cross a jet stream in horizontal flight at approximately right angles. While crossing, in spite of a strong wind of 120 kt, you notice the temperature barely changes. Which of the following statements is correct ? A. B. C. D.

This phenomenon does not surprise you at all, since normally no large temperature differences are possible at these heights. Since the result of such readings seems impossible, you will have the instruments tested after landing. You assume the front associated with the jet stream to be very weak with practically no temperature difference between the two air masses. This phenomenon is absolutely normal as you are crossing the jet core.

311. In which month does the humid monsoon in India start? A. B. C. D.

In October. In December. In March. In June.

312. At about what geographical latitude as average is assumed for the zone of prevailing westerlies? A. B. C. D.

80°N. 10°N. 50°N. 30°N.

313. During July flights from Bangkok (13°N - 100°E) to Karachi (25°N - 67°E) experience an average tailwind component of 22 kt. In January the same flights, also operating at FL 370, have an average headwind of 50 kt. What is the reason for this difference? A. B. C. D.

The wind components correspond to the seasonal change of the regional wind system. The flights during the summer encountered, by chance, very unusual, favourable conditions. The flights happen to be in the area of the polar front jet stream. The flights in January encountered, by chance, very unusual, adverse wind conditions.

314. What jet streams are likely to be crossed during a flight from Stockholm to Rio de Janeiro (23°S) at FL 350 in July ? A. B. C. D.

A subtropical jet stream followed by a polar front jet stream. One subtropical jet stream. A polar front jet stream followed by a subtropical jet stream and later, a second polar front jet stream. A polar front jet stream followed by one or two subtropical jet streams.

315. While crossing a jet stream at right angles in Western Europe (3000 FT below its core) and OAT is decreasing, what would be the prevailing wind? A. B. C. D.

Crosswind from the left A headwind. Crosswind from the right A tailwind.

316. What is normally the most effective measure to reduce or avoid CAT effects? A. B. C. D.

Change of course. Increase of speed. Change of flight level. Decrease of speed.

317. Which area of a polar front jet stream in the northern hemisphere has the highest probability of turbulence? A. B. C. D.

Looking downstream, the area to the right of the core. Looking downstream, the area to the left of the core. Above the core in the boundary between warm and cold air. In the core of the jet stream.

318. Atmospheric soundings give the following temperature profile: 3000 FT +15°C 6000 FT +8°C 10000 FT +1°C 14000 FT -6°C 18000 FT 14°C 24000 FT -26°C At which of the following flight levels is the risk for aircraft icing, in cloud, greatest? A. FL 150 B. FL 80

ATP Online - Meteorology

Page 46 of 213

C. FL 180 D. FL 220

319. On the approach, the surface temperature is given as -5°C. The freezing level is at 3000 FT/AGL. At 4000 FT/AGL, there is a solid cloud layer from which rain is falling. According to the weather briefing, the clouds are due to an inversion caused by warm air sliding up and over an inclined front. Would you expect icing? A. B. C. D.

Yes, but only between 3000 and 4000 FT/AGL. No, flights clear of cloud experience no icing. No, absolutely no icing will occur. Yes, between ground level and 3000 FT/AGL.

320. In an air mass with no clouds the surface temperature is 15°C and the temperature at 1000 m/AGL is 13°C. This layer of air is: A. B. C. D.

a layer of heavy turbulence conditionally unstable stable unstable

321. Half the mass of the atmosphere is found in the first A. B. C. D.

11 km 8 km 3 km 5 km

322. The average thickness of the troposphere varies with A. B. C. D.

rotation of the earth the wind latitude longitude

323. According to ISA the temperature in the lower part of the stratosphere A. B. C. D.

is constant with altitude decreases with altitude increases with altitude first increases with altitude but decreases at higher altitudes

324. How are the air masses distributed in a cold occlusion ? A. B. C. D.

The coldest air mass behind and the less cold air in front of the occlusion; the warm air mass is above ground level. The coldest air behind and the warm air in front of the occlusion; the less cold air mass is above ground level. The coldest air in front of and the warm air behind the occlusion; the less cold air is above ground level. The coldest air in front of and the less cold air is behind the occlusion; the warm air mass is above ground level.

325. If a saturated air mass descends down a slope its temperature increases at A. B. C. D.

the same rate as if the air mass were dry. a lower rate than in dry air, as evaporation absorbs heat. a higher rate than in dry air, as it gives up latent evaporation heat. a lower rate than in dry air, as condensation gives out heat.

326. What characterizes a stationary front ? A. B. C. D.

The surface wind usually has its direction perpendicular to the front The surface wind usually has its direction parallel to the front The weather conditions that it originates is a combination between those of an intense cold front and those of a warm and very active front The warm air moves at approximately half the speed of the cold air

327. Which of the following conditions would cause the altimeter to indicate a lower altitude than that actually flown ? A. B. C. D.

Air temperature higher than standard Pressure altitude the same as indicated altitude. Atmospheric pressure lower than standard Air temperature lower than standard.

328. Which of the following conditions are most favourable to the formation of mountain waves ? A. B. C. D.

Either stable or unstable air at mountain top and a wind of at least 30 knots blowing parallel to the mountain ridge. Unstable air at mountain top altitude and a wind at least 20 knots blowing across the mountain ridge. Moist unstable air at mountain top and wind of less than 5 knots blowing across the mountain ridge. Stable air at mountain top altitude and a wind at least 20 knots blowing across the mountain ridge.

ATP Online - Meteorology

Page 47 of 213

329. An aircraft is approaching a cold front from the warm air mass side at FL 270 and experiencing moderate to severe turbulence. A jet stream is forecast to be at FL 310. The shortest way to get out of this turbulence is by: A. B. C. D.

Descending Turn right Climbing Maintain FL 270

330. The jetstream and associated clear air turbulence can sometimes be visually identified in flight by A. B. C. D.

dust or haze at high level a high-pressure centre at high level a constant outside air temperature long streaks of cirrus clouds.

331. During the winter months in mid-latitudes in the northern hemisphere, the polar front jet stream moves toward the A. B. C. D.

south and speed increases north and speed decreases south and speed decreases north and speed increases

332. What causes surface winds to flow across the isobars at an angle rather than parallel to the isobars ? A. B. C. D.

Greater density of the air at the surface Greater atmospheric pressure at the surface Surface friction Coriolis force

333. Which type of fog is likely to form when air having temperature of 15°C and dew point of 12°C blows at 10 knots over a sea surface having temperatures of 5°C ? A. B. C. D.

Frontal fog Radiation fog Steam fog Advection fog

334. In addition to a lifting action, what are two other requirements necessary for thunderstorm formation ? A. B. C. D.

Unstable conditions and high moisture content Stable conditions and low atmospheric pressure Stable conditions and high moisture content Unstable conditions and high atmospheric pressure

335. If you have to fly through a warm front when freezing level is at 10000 feet in the warm air and at 2000 feet in the cold air, at which altitude is the probability of freezing rain the lowest ? A. B. C. D.

9000 feet 3000 feet 12000 feet 5000 feet

336. From the following TAF you can assume that visibility at 2055Z in Birmingham (EGBB) will be :; EGBB 261812 28015G25KT 9999 SCT025 TEMPO 1822 29018G35KT 5000 SHRASN BKN010CB PROB30 TEMPO 1821 1500 TSGR BKN008CB BECMG 2124 26010KT A. B. C. D.

more than 10 km not less than 1,5 km but could be in excess of 10 km. a maximum 5 km. a minimum of 1,5 km and a maximum of 5 km.

337. What prevents air from flowing directly from high-pressure areas to low-pressure areas ? A. B. C. D.

Coriolis force Katabatic force Surface friction The pressure gradient force

338. Select the true statement concerning isobars and wind flow patterns around high- and low-pressure systems that are shown on a surface weather chart. A. B. C. D.

When the isobars are close together, the pressure gradient force is greater and wind velocities are stronger. Isobars connect contour lines of equal temperature. Surface winds flow perpendicular to the isobars. When the isobars are far apart, crest of standing waves may be marked by stationary lenticular clouds.

ATP Online - Meteorology

Page 48 of 213

339. The QFF at an airfield located 400 metres above sea level is 1016 hPa. The air temperature is 10°C lower than a standard atmosphere. What is the QNH? A. B. C. D.

It is not possible to give a definitive answer Less than 1016 hPa More than 1016 hPa 1016 hPa

340. The QNH at an airfield located 200 metres above sea level is 1009 hPa. The air temperature is 10°C lower than a standard atmosphere. What is the QFF? A. B. C. D.

More than 1009 hPa It is not possible to give a definitive answer 1009 hPa Less than 1009 hPa

341. The QNH at an airfield located 200 metres above sea level is 1022 hPa. The air temperature is not available. What is the QFF? A. B. C. D.

1022 hPa More than 1022 hPa It is not possible to give a definitive answer Less than 1022 hPa

342. The QNH at an airfield located at mean sea level is 1022 hPa. The air temperature is not available. What is the QFF? A. B. C. D.

1022 hPa Less than 1022 hPa It is not possible to give a definitive answer More than 1022 hPa

343. The QNH at an airfield in California located 69 metres below sea level is 1018 hPa. The air temperature is 10°C higher than a standard atmosphere. What is the QFF? A. B. C. D.

More than 1018 hPa It is not possible to give a definitive answer 1018 hPa Less than 1018 hPa

344. The QFF at an airfield in California located 69 metres below sea level is 1030 hPa. The air temperature is 10°C lower than a standard atmosphere. What is the QNH? A. B. C. D.

Less than 1030 hPa More than 1030 hPa 1030 hPa It is not possible to give a definitive answer

345. If the QFE at Locarno (200 metres above sea level) is 980 hPa, what is the approximate QNH ? A. B. C. D.

1000 hPa 1010 hPa 1005 hPa 1015 hPa

346. If the QFE at Locarno (200 metres above sea level) is 1000 hPa, what is the approximate QNH? A. B. C. D.

1025 hPa 985 hPa 990 hPa 1035 hPa

347. If the QNH at Locarno (200 metres above sea level) is 1015 hPa, what is the approximate QFE? A. B. C. D.

995 hPa 1005 hPa 1000 hPa 990 hPa

348. If the QNH at Locarno (200 metres above sea level) is 1025 hPa, what is the approximate QFE? A. B. C. D.

1000 hPa 1025 hPa 995 hPa 1005 hPa

349. If you are flying at FL 300 in an air mass that is 15°C warmer than a standard atmosphere, what is the outside temperature likely to

ATP Online - Meteorology

Page 49 of 213

be? A. B. C. D.

-15°C -60°C -30°C -45°C

350. If you are flying at FL 100 in an air mass that is 10°C warmer than a standard atmosphere, what is the outside temperature likely to be? A. B. C. D.

+5°C +15°C -15°C -10°C

351. If you are flying at FL 120 and the outside temperature is -2°C, at what altitude will the "freezing level" be? A. B. C. D.

FL 110 FL 90 FL 130 FL 150

352. An aircraft flying at FL 100 from Marseille (QNH 1012 hPa) to Palma de Mallorca (QNH 1006 hPa) experiences no change to true altitude. The reason for this is that: A. B. C. D.

the altimeters are erroneous, and need to be tested the air at Palma de Mallorca is colder than that at Marseille one of the two QNH values may be incorrect the air at Palma de Mallorca is warmer than that at Marseille

353. During a flight over the sea at FL 100 from Marseille (QNH 1012 hPa) to Palma de Mallorca (QNH 1012 hPa), the true altitude is constantly increasing. What action, if any, should be taken ? A. B. C. D.

Recheck the QNH because one of the QNH values must be wrong None, the reason for the change is that the air around Palma is warmer than the air around Marseille Have your altimeter checked, because its readings are obviously wrong Compensate by heading further to the left

354. During a flight over the sea at FL 100 from Marseille (QNH 1016 hPa) to Palma de Mallorca (QNH 1016 hPa), the true altitude is constantly decreasing. What is the probable reason for this ? A. B. C. D.

The air at Palma de Mallorca is warmer than that at Marseille The air at Marseille is warmer than that at Palma de Mallorca One of the QNH values must be wrong The altimeter is faulty

355. During a flight over the sea at FL 135, the true altitude is 13500 feet; local QNH is 1019 hPa. What information, if any, can be gained about the air mass in which the aircraft is flying? A. B. C. D.

Its average temperature is the same as ISA There is insufficient information to make any assumption It is colder than ISA It is warmer than ISA

356. An aircraft is flying over the sea at FL 90; the true altitude is 9100 feet; local QNH is unknown. What assumption, if any, can be made about the air mass in which the aircraft is flying ? A. B. C. D.

It is colder than ISA There is insufficient information to make any assumption Its average temperature is the same as ISA It is warmer than ISA

357. An aircraft is flying over the sea at FL 120, with a true altitude of 12000 feet; local QNH is 1013 hPa. What assumption, if any, can be made about the air mass in which the aircraft is flying ? A. B. C. D.

There is insufficient information to come to any conclusion It is warmer than ISA Its average temperature is the same as ISA It is colder than ISA

358. An aircraft is flying over the sea at FL 100, with a true altitude of 10000 feet; local QNH is 1003 hPa. What assumption, if any, can be made about the air mass in which the aircraft is flying ? A. B. C. D.

It is warmer than ISA Its average temperature is about ISA There is insufficient information to come to any conclusion It is colder than ISA

ATP Online - Meteorology

Page 50 of 213

359. What type of fog is most likely to form over flat land during a clear night, with calm or light wind conditions ? A. B. C. D.

Steam. Orographic. Radiation. Advection.

360. In which environment is aircraft structural ice most likely to have the highest rate of accretion ? A. B. C. D.

Freezing rain. Snow. Stratus clouds. Cirrus clouds.

361. The presence of ice pellets at the surface is evidence that A. B. C. D.

there are thunderstorms in the area freezing rain occurs at a higher altitude a cold front has passed a warm front has passed

362. Updraughts in a thunderstorm are prevailing during the A. B. C. D.

period in which precipitation is falling. initial stage. mature stage. dissipating stage.

363. Thunderstorms reach their greatest intensity during the A. B. C. D.

dissipating stage. mature stage. period in which precipitation is not falling. cumulus stage.

364. The most dangerous form of airframe icing is A. B. C. D.

rime ice. hoar frost. clear ice. dry ice.

365. The degree of clear air turbulence experienced by an aircraft is proportional to the A. B. C. D.

intensity of vertical and horizontal wind shear height of the aircraft stability of the air intensity of the solar radiation

366. In an intense trough of low pressure over Iceland during wintertime the weather likely to be experienced is: A. B. C. D.

light wind, good visibility and a high cloud ceiling strong wind shear, convection and snow showers strong wind associated with an almost clear sky strong wind with subsidence at low levels

367. Which layer of the atmosphere contains more than 90 per cent of all water vapour? A. B. C. D.

Troposphere Lower stratosphere Ozone layer Upper stratosphere

368. The temperature at FL 80 is +6°C. What will the temperature be at FL 130 if the ICAO standard lapse rate is applied ? A. B. C. D.

-4°C +2°C -6°C 0°C

369. The temperature at FL 110 is -5°C. What will the temperature be at FL 50 if the ICAO standard lapse rate is applied ? A. +7°C B. -3°C

ATP Online - Meteorology

Page 51 of 213

C. 0°C D. +3°C

370. The temperature at FL 160 is -22°C. What will the temperature be at FL 90 if the ICAO standard lapse rate is applied ? A. B. C. D.

0°C -4°C +4°C -8°C

371. A temperature of +15°C is recorded at an altitude of 500 metres above sea level. If the vertical temperature gradient is that of a standard atmosphere, what will the temperature be at the summit of a mountain, 2500 metres above sea level? A. B. C. D.

+4°C 0°C +2°C -2°C

372. An air temperature of -15°C at the 700 hPa level over central Europe in summer is A. B. C. D.

High 20°C below standard Within +/-5°C of ISA Low

373. An air temperature of -30°C at the 300 hPa level over central Europe in summer is A. B. C. D.

Very low Within +/-5°C of ISA High Low

374. An air temperature of -55°C at the 200 hPa level over central Europe in summer is A. B. C. D.

Within +/-5°C of ISA High Very high Low

375. What is the technical term for an increase in temperature with altitude? A. B. C. D.

Inversion Adiabatic Advection Subsidence

376. The station pressure used in surface weather charts is A. B. C. D.

QFF QFE QNH QNE

377. Which FL corresponds with the 300 hPa pressure level ? A. B. C. D.

FL 300 FL 50 FL 390 FL 100

378. Which FL corresponds with the 500 hPa pressure level ? A. B. C. D.

FL 390 FL 180 FL 100 FL 160

379. Which FL corresponds with the 700 hPa pressure level ? A. B. C. D.

FL 180 FL 300 FL 100 FL 390

ATP Online - Meteorology

Page 52 of 213

380. Which FL corresponds with the 850 hPa pressure level ? A. B. C. D.

FL 50 FL 300 FL100 FL 390

381. The QFF at an airfield located 400 metres above sea level is 1016 hPa. The air temperature is 10°C higher than a standard atmosphere. What is the QNH? A. B. C. D.

More than 1016 hPa Less than 1016 hPa 1016 hPa It is not possible to give a definitive answer

382. An aircraft is descending to land under IFR. If the local QNH is 1009 hPa, what will happen to the altitude reading when the altimeter is reset at the transition level ? A. B. C. D.

It will not be affected It will remain the same It will increase It will decrease

383. During the climb after take-off, the altimeter setting is adjusted at the transition altitude. If the local QNH is 1023 hPa, what will happen to the altimeter reading during the resetting procedure ? A. B. C. D.

It will remain the same It is not possible to give a definitive answer It will decrease It will increase

384. During the climb after take-off, the altimeter setting is adjusted at the transition altitude. If the local QNH is 966 hPa, what will happen to the altimeter reading during the resetting procedure? A. B. C. D.

It will remain the same It will decrease It will increase It is not possible to give a definitive answer

385. An aircraft is flying from Point A to Point B on the upper level contour chart. The altimeter setting is 1013.2 hPa. Which of these statements is correct? A. B. C. D.

Wind speed at Paris is higher than at B The true altitude will be higher at A than at B The true altitude will be higher at B than at A Wind speed at A is higher than at B

386. An aircraft is flying from Point A to Point B on the upper level contour chart. The altimeter setting is 1013,2 hPa. Which of these statements is correct? A. B. C. D.

Wind speed at A is higher than at B Wind speed at A and at B is the same The true altitude will be higher at B than at A The true altitude will be higher at A than at B

ATP Online - Meteorology

Page 53 of 213

387. An aircraft is flying from Point A to Point B on the upper level contour chart. The altimeter setting is 1013,2 hPa. Which of these statements is correct? A. B. C. D.

The true altitude will be higher at A than at B Wind speed at B is higher than at A Wind speed at Madrid is higher than at A The true altitude will be higher at B than at A

388. An aircraft is flying from Point A to Point B on the upper level contour chart. The altimeter setting is 1013,2 hPa. Which of these statements is correct? A. B. C. D.

The true altitude will be higher at A than at B Wind speed at A and at B is the same Wind speed at B is higher than at A The true altitude will be higher at B than at A

389. You are flying at FL 200. Outside air temperature is -40°C, and the pressure at sea level is 1033 hPa. What is the true altitude? A. B. C. D.

18290 ft 20630 ft 19310 ft 21770 ft

ATP Online - Meteorology

Page 54 of 213

390. You are flying at FL 160. Outside air temperature is -27°C, and the pressure at sea level is 1003 hPa. What is the true altitude? A. B. C. D.

15620 ft 16360 ft 15100 ft 16920 ft

391. You are planning to fly across a mountain range. The chart recommends a minimum altitude of 12000 feet above mean sea level. The air mass you will be flying through is an average 10°C warmer than ISA. Your altimeter is set to 1023 hPa (QNH of a nearby airport at nearly sea level). What altitude will the altimeter show when you have reached the recommended minimum altitude?; A. B. C. D.

11520 feet 11790 feet 11250 feet 12210 feet

392. An aircraft lands at an airport (airport elevation 540 FT, QNH 993 hPa) with the altimeter set to 1013 hPa. What will it indicate ? A. B. C. D.

700 FT 380 FT 0 FT 1080 FT

393. After landing at an aerodrome (QNH 993 hPa) it is noticed that the altimeter is still set to 1013,2 hPa and that it reads 1200 feet. What is the elevation of the aerodrome above mean sea level ? A. B. C. D.

660 feet. 1740 feet. 1200 feet. 2280 feet.

394. What is the approximate speed of a 25-knot wind, expressed in kilometres per hour? A. B. C. D.

45 km/h 60 km/h 35 km/h 55 km/h

395. What is the approximate speed of a 90 km/h wind, expressed in knots? A. B. C. D.

55 kt 60 kt 70 kt 50 kt

396. What is the approximate speed of a 40-knot wind, expressed in m/sec? A. B. C. D.

30 m/sec 25 m/sec 20 m/sec 15 m/sec

397. What is the wind speed given in a METAR report based on? A. B. C. D.

The actual speed at the time of recording The average speed of the previous 10 minutes The average speed of the previous 30 minutes The strongest gust in the previous hour

398. Where are you likely to find the strongest winds close to the ground? A. B. C. D.

Where there is little variation in pressure over a large area during the winter months At the centre of a high-pressure system At the centre of a low-pressure system In the transition zone between two air masses

399. An aircraft flying in the southern hemisphere at 2000 feet, has to turn to the right in order to allow for drift. In which direction, relative to the aircraft, is the centre of low pressure ? A. B. C. D.

To the left. To the right. In front. Behind.

400. Between which latitudes are you most likely to find the subtropical high-pressure belt ?

ATP Online - Meteorology A. B. C. D.

Page 55 of 213

10° - 15°. 35° - 55°. 55° - 75°. 25° - 35°.

401. Between which latitudes are you most likely to find the region of travelling low pressure systems during summer? A. B. C. D.

15° - 25° 45° - 70° 10° - 15° 25° - 45°

402. Where, in central Europe, are the highest wind speeds to be found ? A. B. C. D.

At about 5500 metres altitude In the stratosphere Close to the ground Just below the tropopause

403. If Paris reports a wind of 19015KT on the METAR, what wind velocity would you expect to encounter at a height of 2000 feet above the ground ? A. B. C. D.

22030KT 25025KT 22010KT 16020KT

404. If Paris reports a wind of 08010KT on the METAR, what wind velocity would you expect to encounter at a height of 2000 feet above the ground ? A. B. C. D.

08015KT 05020KT 11020KT 08005KT

405. If Paris reports a wind of 16020KT on the METAR, what wind velocity would you expect to encounter at a height of 2000 feet above the ground? A. B. C. D.

16030KT 17015KT 19040KT 14020KT

406. If Paris reports a wind of 30012KT on the METAR, what wind velocity would you expect to encounter at a height of 2000 feet above the ground ? A. B. C. D.

23030KT 33025KT 27020KT 30025KT

407. Which of the following is true of a land breeze? A. B. C. D.

It blows by day It blows from water to land It blows only at noon It blows from land to water

408. An aircraft is approaching under visual flight rules an airfield whose runway is parallel to the coast. When downwind over the sea, the airfield is on the right. What wind effect should be anticipated on final approach and landing during a sunny afternoon? A. B. C. D.

Headwind Tailwind Crosswind from the right Crosswind from the left

409. An aircraft is approaching under visual flight rules an airfield (northern hemisphere) whose runway is parallel to the coast. When downwind over the sea, the airfield is on the left. What wind effect should be anticipated on final approach and landing during a sunny afternoon ? A. B. C. D.

Crosswind from the right Headwind Crosswind from the left Tailwind

ATP Online - Meteorology

Page 56 of 213

410. Which of the following statements is true of the dew point of an air mass? A. B. C. D.

It can only be equal to, or lower, than the temperature of the air mass It can be higher than the temperature of the air mass It can be used together with the air pressure to estimate the air mass's relative humidity It can be used to estimate the air mass's relative humidity even if the air temperature is unknown

411. Which types of clouds are typical evidence of stable air conditions? A. B. C. D.

CU, CB NS, CU CB, CC ST, AS

412. Which one of the displayed cloud forms is representative of altocumulus lenticularis? A. B. C. D.

4 1 3 2

413. Which one of the displayed cloud forms is representative of a cumulonimbus capillatus? A. B. C. D.

4 1 2 3

414. Which of the following cloud genera may extend over the low, middle and high stages? A. B. C. D.

CI ST AC CB

415. A plain in Western Europe with an average height of 500 m (1600 FT) above sea level is covered with a uniform SC layer of cloud during the summer months. At what height above the ground is the base of this cloud to be expected? A. B. C. D.

7000 - 15000 FT above ground 100 - 1500 FT above ground 15000 - 25000 FT above ground 1500 - 7000 FT above ground

416. A plain in Western Europe with an average height of 500 m (1600 FT) above sea level is covered with a uniform CC layer of cloud during the summer months. At what height above the ground is the base of this cloud to be expected? A. B. C. D.

7000 - 15000 FT above the terrain 100 - 1500 FT above the terrain 1500 - 7000 FT above the terrain 15000 - 35000 FT above the terrain

417. Which of the following cloud types is found at high levels? A. B. C. D.

CI SC CU AS

418. Which of the following cloud types is a medium level cloud ? A. ST

ATP Online - Meteorology

Page 57 of 213

B. SC C. AS D. CS

419. Under which of these conditions is radiation fog most likely to form? A. B. C. D.

Strong surface winds Very dry air Little or no cloud Very low temperatures

420. Which of the following is most likely to lead to the formation of radiation fog? A. B. C. D.

Dry, warm air passing over warm ground Cold air passing over warm ground Heat loss from the ground on clear nights The passage of fronts

421. With which of the following types of cloud is "+RA" precipitation most commonly associated? A. B. C. D.

SC ST NS AC

422. With what type of cloud is "GR" precipitation most commonly associated? A. B. C. D.

AS CC CB ST

423. With what type of cloud is "DZ" precipitation most commonly associated? A. B. C. D.

ST CU CB CC

424. Which of the following cloud types is least likely to produce precipitation ? A. B. C. D.

NS CB AS CI

425. With what type of cloud is heavy precipitation unlikely during the summer months ? A. B. C. D.

SC, AS CB, ST NS, CC AS, NS

426. With what type of cloud is "+TSRA" precipitation most commonly associated? A. B. C. D.

CB AS NS SC

427. Read this description: "After such a fine day, the ring around the moon was a bad sign yesterday evening for the weather today. And, sure enough, it is pouring down outside. The clouds are making an oppressively low ceiling of uniform grey; but at least it has become a little bit warmer." Which of these weather phenomena is being described? A. B. C. D.

Weather at the back of a cold front A cold front A blizzard A warm front

428. What will be the effect on the reading of an altimeter of an aircraft parked on the ground during the period following the passage of an active cold front ? A. B. C. D.

It will have decreased. It will remain unchanged. It will show a small increase or decrease. It will have increased.

ATP Online - Meteorology

Page 58 of 213

429. What will be the effect on the reading of an altimeter of an aircraft parked on the ground shortly before an active cold front passes? A. B. C. D.

It will be increasing. It will fluctuate up and down by about +/- 50 feet. It will remain unchanged. It will be decreasing.

430. What will be the effect on the reading of an altimeter of an aircraft parked on the ground as an active cold front is passing? A. B. C. D.

It will remain unchanged. It will first decrease then increase. It will fluctuate up and down by about +/- 50 feet. It will first increase then decrease.

431. Continuous freezing rain is observed at an airfield. Which of the four diagrams is most likely to reflect temperatures above the airfield concerned? A. B. C. D.

Diagram 3 Diagram 2 Diagram 1 Diagram 4

432. At what degree of icing should ICAO's "Change of course and/or altitude desirable" recommendation be followed? A. B. C. D.

Severe Extreme Moderate Light

433. At what degree of icing should ICAO's "Change course and/or altitude immediately" instruction be followed? A. B. C. D.

Light Severe Extreme Moderate

434. At what degree of icing can ICAO's "No change of course and altitude necessary" recommendation be followed? A. Extreme

ATP Online - Meteorology B. Light C. Severe D. Moderate

435. When will the surface wind in a METAR record a gust factor ? A. B. C. D.

When gusts are at least 10 knots above the mean wind speed With gusts of at least 25 knots When gusts are at least 15 knots above the mean wind speed With gusts of at least 35 knots

436. Of the four radio soundings, select the one that indicates low stratus: A. B. C. D.

4 1 3 2

437. What are the images of satellites provided daily by the Weather Service used for? A. B. C. D.

To help provide 14-day forecasts To measure wind currents on the ground To locate precipitation zones To locate fronts in areas with few observation stations

438. Which of the following causes echoes on meteorological radar screens? A. B. C. D.

Hail Water vapour Any cloud Fog

439. Which constant pressure altitude chart is standard for FL 100? A. B. C. D.

500 hPa 700 hPa 300 hPa 850 hPa

440. Which constant pressure altitude chart is standard for FL 180? A. B. C. D.

700 hPa 500 hPa 200 hPa 300 hPa

441. Which constant pressure altitude chart is standard for FL 300? A. 300 hPa

Page 59 of 213

ATP Online - Meteorology

Page 60 of 213

B. 700 hPa C. 500 hPa D. 200 hPa

442. Which constant pressure altitude chart is standard for FL 390? A. B. C. D.

200 hPa 300 hPa 500 hPa 700 hPa

443. If you are planning a flight at FL 170, which of these upper wind and temperature charts would be nearest your flight level ? A. B. C. D.

700 hPa 500 hPa 300 hPa 850 hPa

444. If you are planning a flight at FL 290, which of these upper wind and temperature charts would be nearest your flight level ? A. B. C. D.

500 hPa 850 hPa 700 hPa 300 hPa

445. When planning a flight at FL 60, which upper wind and temperature chart would be nearest your flight level ? A. B. C. D.

300 hPa 700 hPa 500 hPa 850 hPa

446. When planning a flight at FL 110, which upper wind and temperature chart would be nearest your flight level ? A. B. C. D.

300 hPa 700 hPa 850 hPa 500 hPa

447. Which of the following best describes Zone A? A. B. C. D.

Ridge of high pressure Trough of low pressure Depression Col

448. Which of the following best describes Zone B? A. B. C. D.

Ridge of high pressure Depression Trough of low pressure col

ATP Online - Meteorology

449. Which of the following best describes Zone C? A. B. C. D.

ridge of high pressure Trough of low pressure Col Depression

450. Which of the following best describes Zone D? A. B. C. D.

Ridge of high pressure Trough of low pressure Anticyclone Depression

451. According to ICAO, which symbol indicates severe icing ? A. B. C. D.

Symbol c) Symbol a) Symbol b) Symbol d)

Page 61 of 213

ATP Online - Meteorology

452. According to ICAO, which symbol indicates a tropical revolving storm? A. B. C. D.

Symbol c) Symbol d) Symbol a) Symbol b)

453. According to ICAO, which symbol indicates widespread haze ? A. B. C. D.

Symbol d) Symbol c) Symbol a) Symbol b)

Page 62 of 213

ATP Online - Meteorology

454. According to ICAO, which symbol indicates a severe line squall ? A. B. C. D.

Symbol d) Symbol c) Symbol a) Symbol b)

455. What does the term SIGMET signify? A. B. C. D.

A SIGMET is an actual weather report at an aerodrome and is generally issued at half-hourly intervals A SIGMET is a brief landing forecast added to the actual weather report A SIGMET is a warning of dangerous meteorological conditions A SIGMET is a flight forecast, issued by the meteorological station several times daily

456. What does the term TREND signify? A. B. C. D.

It is a warning of dangerous meteorological conditions It is a flight forecast, issued by the meteorological station several times daily It is a landing forecast added to the actual weather report It is the actual weather report at an aerodrome and is generally issued at half-hourly intervals

Page 63 of 213

ATP Online - Meteorology

Page 64 of 213

457. Which of the following phenomena should be described as precipitation at the time they are observed? A. B. C. D.

SN HZ BCFG FZFG

458. Which of the following phenomena should be described as precipitation at the time they are observed? A. B. C. D.

+SHSN; BR; MIFG; VA;

459. Which of the following phenomena should be described as precipitation at the time they are observed? A. B. C. D.

SQ; TS; SA; DZ;

460. Which of the following meteorological phenomena can rapidly change the braking action of a runway? A. B. C. D.

+FZRA FG MIFG HZ

461. Which of the following phenomena can produce a risk of aquaplaning? A. B. C. D.

+RA SA BCFG FG

462. Which of these four METAR reports suggests that rain is most likely in the next few hours? A. B. C. D.

16002KT 0100 FG SCT300 06/06 Q1022 BECMG 1000 = 23015KT 8000 BKN030 OVC070 17/14 Q1009 BECMG 4000 = 05016G33KT 8000 OVC015 08/06 Q1028 NOSIG = 34004KT 9999 SCT040 SCT100 m05/m08 Q1014 NOSIG =

463. Which of these four METAR reports suggests that a thunderstorm is likely in the next few hours? A. B. C. D.

201350Z 16004KT 8000 SCT110 OVC220 02/M02 Q1008 NOSIG = 201350Z 21005KT 9999 SCT040CB SCT100 26/18 Q1016 TEMPO 24018G30KT TS = 201350Z 34003KT 0800 SN VV002 M02/M04 Q1014 NOSIG = 201350Z 04012KT 3000 BR OVC012 04/03 Q1022 BECMG 6000 =

464. In which of the following METAR reports is the probability of fog formation in the coming night the highest? A. B. C. D.

201850Z 15003KT 6000 SCT120 05/04 Q1032 BECMG 1600 BR= 201850Z 21003KT 8000 SCT250 12/m08 Q1028 NOSIG= 201850Z 25010KT 4000 RA BKN012 OVC030 12/10 Q1006 TEMPO 1500= 201850Z 06018G30KT 5000 -RA OVC010 04/01 Q1024 NOSIG=

465. Which of these statements best describes the weather most likely to be experienced at 1500 UTC?; TAF LSZH 211322 22018G35KT 9999 SCT012 BKN030 BECMG 1315 25025G45KT TEMPO 1720 4000 +SHRA BKN025TCU BECMG 2022 25015KT T1815Z T1618Z = A. B. C. D.

Severe rainshowers, meteorological visibility 4000 metres, temperature 15°C, gusts up to 35 knots. Visibility 10 kilometres or more, main cloudbase 3000 feet, wind 250°, temperature 18°C. Visibility 4000 metres, gusts up to 25 knots, temperature 18°C. Visibility 10 kilometres or more, main cloudbase 1200 feet, gusts up to 45 knots.

466. TAF LSZH 250716 00000KT 0100 FG VV001 BECMG 0810 0800 VV002 BECMG 1012 23005KT 2500 BKN005 TEMPO 1316 6000 SCT007 =; Which of these statements best describes the weather that can be expected at 1200 UTC? A. B. C. D.

Visibility 800 metres, wind from 230°, cloudbase 500 feet Visibility 800 metres, vertical visibility 200 feet, calm Visibility 6 kilometres, cloudbase 500 feet, windspeed 5 knots Visibility 2,5 kilometres, cloudbase 500 feet, windspeed 5 knots

467. Which of the following weather reports could be, in accordance with the regulations, abbreviated to "CAVOK"? A. 29010KT 9999 SCT045TCU 16/12 Q1015 RESHRA NOSIG = B. 15003KT 9999 BKN100 17/11 Q1024 NOSIG =

ATP Online - Meteorology

Page 65 of 213

C. 24009KT 6000 RA SCT010 OVC030 12/11 Q1007 TEMPO 4000 = D. 04012G26KT 9999 BKN030 11/07 Q1024 NOSIG =

468. Which of the following weather reports could be, in accordance with the regulations, abbreviated to "CAVOK"? A. B. C. D.

04012G26KT 9999 BKN030 11/07 Q1024 NOSIG = 29010KT 9999 SCT045TCU 16/12 Q1015 RESHRA NOSIG = 15003KT 9999 BKN100 17/11 Q1024 NOSIG = 24009KT 6000 RA SCT010 OVC030 12/11 Q1007 TEMPO 4000 =

469. Which of the following statements is an interpretation of the METAR ? 25020G38KT 1200 +TSGR BKN006 BKN015CB 23/18 Q1016 BECMG NSW = A. B. C. D.

Gusts of 38 knots, thunderstorm with heavy hail, dew point 18°C Broken, cloud base 600 feet and 1500 feet, temperature 18°C Wind 250°, thunderstorm with moderate hail, QNH 1016 hPa Mean wind speed 20-38 knots, visibility 1200 metres, temperature 23°C

470. Which of the following statements is an interpretation of the METAR ? 00000KT 0200 R14/0800U R16/P1500U FZFG VV001 m03/m03 Q1022 BECMG 0800 = A. B. C. D.

Visibility 200 feet, RVR for runway 16 more than 1500 metres, vertical visibility 100 feet, fog with hoar frost RVR for runway 14 800 metres, vertical visibility 100 feet, calm, visibility improving to 800 metres in the next 2 hours Visibility 200 metres, RVR for runway 16 1500 metres, temperature -3°C, vertical visibility 100 metres Visibility for runway 14 800 metres, fog with hoar frost, RVR for runway 16 more than 1500 metres

471. Which of the following statements is an interpretation of the SIGMET ? LSAS SIGMET 2 VALID 030700/031100 LSZHSWITZERLAND FIR/UIR MOD TO SEV CAT FCST N OF ALPS BTN FL 260/380 STNR INTSF= A. B. C. D.

Moderate to severe clear air turbulence of constant intensity to be expected north of the Alps Moderate to severe clear air turbulence to be expected north of the Alps. Intensity increasing. Danger zone between FL 260 and FL 380 Severe turbulence observed below FL 260 north of the Alps. Pilots advised to cross this area above FL 380 Zone of moderate to severe turbulence moving towards the area north of the Alps. Intensity increasing. Pilots advised to cross this area above FL 260

472. Which of the following statements is an interpretation of the SIGMET ? LGGG SIGMET 3 VALID 121420/121820 LGAT- ATHINAI FIR EMBD TS OBS AND FCST IN W PART MOV E NC = A. B. C. D.

The thunderstorms in the Athens FIR are increasing in intensity, but are stationary above the western part of the Athens FIR Thunderstorms must be expected in the western part of the Athens FIR. The thunderstorm zone is moving east. Intensity is constant Athens Airport is closed due to thunderstorms. The thunderstorm zone should be east of Athens by 1820 UTC Thunderstorms have formed in the eastern part of the Athens FIR and are slowly moving west

473. Compare the following TAF and VOLMET reports for Nice: TAF 240600Z 240716 VRB02KT CAVOK = 0920Z 13012KT 8000 SCT040CB BKN100 20/18 Q1015 TEMPO TS = What can be concluded from the differences between the two reports ? A. B. C. D.

That the weather at Nice is clearly more volatile than the TAF could have predicted earlier in the morning That the weather conditions at 0920 were actually predicted in the TAF That the VOLMET speaker has got his locations mixed up, because there is no way the latest VOLMET report could be so different from the TAF That the weather in Nice after 0920 is also likely to be as predicted in the TAF

474. At which airport is the following weather development taking place? TAF 231200Z 231322 24014G32KT 4000 +TSRA SCT005 BKN015 BKN020CB BECMG 1416 29012KT 9999 BKN030TCU SCT100 TEMPO 1619 8000 SHRA BKN025TCU BECMG 1922 27012KT 9999 SCT030 OVC220 = A. B. C. D.

EKCH LSZH ESSA EINN

ATP Online - Meteorology

475. Which of the following weather conditions would be expected at Athens Airport (LGAT) at around 1450 UTC? A. B. C. D.

21002KT 6000 BR SCT040 29/16 Q1026 NOSIG = 26014KT 8000 BKN090 17/12 Q1009 BECMG 4000 = 16002KT 0200 R33L/0600N FG VV001 12/12 Q1031 BECMG 0800 = 23018G35KT 9999 SCT035 10/04 Q0988 NOSIG =

476. Which airport is most likely to have fog in the coming night? A. B. C. D.

LSZH ESSA EKCH ENGM

Page 66 of 213

ATP Online - Meteorology

477. Select from the map the average wind for the route Zurich - Rome at FL110. A. B. C. D.

250/20 230/10 040/10 200/30

478. Select from the map the average temperature for the route Zurich - Rome at FL 110. A. B. C. D.

-12°C -6°C -9°C +5°C

Page 67 of 213

ATP Online - Meteorology

479. Select from the map the average wind for the route Athens - Geneva at FL 160. A. B. C. D.

240/40 260/40 210/25 050/35

480. Select from the map the average temperature for the route Athens - Geneva at FL 150. A. B. C. D.

-27°C -14°C -21°C -11°C

Page 68 of 213

ATP Online - Meteorology

481. What is the deviation of the temperature at FL 140 above Copenhagen compared to ISA? A. B. C. D.

8°C colder than ISA 4°C warmer than ISA 8°C warmer than ISA 12°C colder than ISA

Page 69 of 213

ATP Online - Meteorology

482. Select from the map the average wind for the route Zurich - Hamburg at FL 240. A. B. C. D.

200/15 020/20 260/25 230/20

483. Select from the map the average temperature for the route Zurich - Lisboa at FL 200. A. B. C. D.

-33°C -49°C -41°C -30°C

Page 70 of 213

ATP Online - Meteorology

484. Select from the map the average wind for the route Shannon - Lisboa at FL 290. A. B. C. D.

030/70 340/90 190/75 360/80

485. Select from the map the average temperature for the route Geneva -Stockholm at FL 260. A. B. C. D.

-55°C -47°C -51°C -63°C

Page 71 of 213

ATP Online - Meteorology

486. On which of the following routes can you expect icing to occur, on the basis of the chart? A. B. C. D.

Rome - Frankfurt Hamburg - Oslo Tunis - Rome Copenhagen - Helsinki

487. Looking at the chart, at what altitude above Frankfurt would you expect the tropopause to be located? A. B. C. D.

FL 390 FL 410 FL 360 FL 330

Page 72 of 213

ATP Online - Meteorology

488. If you are flying from Zurich to London at FL 240, what conditions can you expect at cruising altitude? A. B. C. D.

Flight largely in cloud; no turbulence CAT for the first half of the flight Moderate or severe turbulence and icing Prolonged severe turbulence and icing throughout the flight

489. Judging by the chart, on which of these routes can you expect to encounter moderate CAT at FL 300? A. B. C. D.

Zurich - Athens London - Zurich Zurich - Stockholm Madrid - Bordeaux

Page 73 of 213

ATP Online - Meteorology

490. If you are flying from Zurich to Shannon at FL 340, where will your cruising altitude be? A. B. C. D.

In the stratosphere for part of time Constantly in the stratosphere Constantly in the troposphere First in the troposphere and later in the stratosphere

491. Which of these statements is true? A. B. C. D.

Turbulence is likely to be encountered at FL 410 over Madrid; Freezing level above Madrid is higher than FL 120; Scattered thunderstorms can be expected over France; The front to the north of London is moving south;

Page 74 of 213

ATP Online - Meteorology

Page 75 of 213

492. On which of these routes would you not need to worry about icing at FL 170? A. B. C. D.

London - Stockholm Madrid - Vienna Zurich - Athens Paris - Lisbon

493. This chart shows the weather conditions on the ground at 0600 UTC on May 23. Which of the following reports reflects weather development at Geneva Airport? A. TAF LSGG 230600Z 230716 VRB03KT 6000 SCT020 BECMG 0811 23005KT 9999 SCT025TCU PROB 40 TEMPO 1216 34012G30KT 3000 TSRA BKN020CB = B. TAF LSGG 230600Z 230716 05014KT 5000 BR OVC015 BECMG 0810 8000 BKN018 BECMG 1013 05015G30KT 9999 SCT025 =

ATP Online - Meteorology

Page 76 of 213

C. TAF LSGG 230600Z 230716 20016KT 8000 -RA BKN030 OVC070 BECMG 0810 5000 RA BKN020 OVC050 TEMPO 1012 3000 +RA BKN010 OVC030 BECMG 1215 25014KT 8000 SCT030 BKN090 = D. TAF LSGG 230600Z 230716 26012KT 9999 SCT030 BKN080 TEMPO 1013 25020G35KT 3000 TSRA BKN030CB BECMG 1316 VRB02KT 3000 BCFG SCT100 =

494. In Zurich during a summer day the following weather observations were taken: 160450Z 23015KT 3000 +RA SCT008 SCT020 OVC030 13/12 Q1010 NOSIG = 160650Z 25008KT 6000 SCT040 BKN090 18/14 Q1010 RERA NOSIG = 160850Z 25006KT 8000 SCT040 SCT100 19/15 Q1009 NOSIG 161050Z 24008KT 9999 SCT040 SCT100 21/15 Q1008 NOSIG =; 161250Z 23012KT CAVOK 23/16 Q1005 NOSIG = 161450Z 23016KT 9999 SCT040 BKN090 24/17 Q1003 BECMG 25020G40KT TS =161650Z 24018G35KT 3000 +TSRA SCT006 BKN015CB 18/16 Q1002 NOSIG = 161850Z 28012KT 9999 SCT030 SCT100 13/11 Q1005 NOSIG = What do you conclude based on these observations? A. B. C. D.

A cold front passed the station early in the morning and a warm front during late afternoon A trough line passed the station early in the morning and a warm front during late afternoon Storm clouds due to warm air came close to and grazed the station A warm front passed the station early in the morning and a cold front during late afternoon

495. Which typical weather condition is shown by the design for the area of Central Europe ? A. B. C. D.

Uniform pressure pattern Westerly waves Easterly waves Cutting wind

496. Which typical weather condition is shown by the design for northern Italy? A. B. C. D.

High pressure Warm southerly wind Easterly wind Westerly wind

497. The attached chart shows the weather conditions on the ground at 1200 UTC on October 10. Which of the following reports reflects

ATP Online - Meteorology

Page 77 of 213

weather development at Zurich Airport? A. B. C. D.

TAF LSZH 101601 VRB02KT 8000 SCT280 BECMG 1618 00000KT 3500 MIFG BECMG 1820 1500 BCFG BECMG 2022 0100 FG VV001 = TAF LSZH 101601 23012KT 6000 RA BKN012 OVC030 TEMPO 2023 22025G40KT 1600 +SNRA BKN003 OVC015 = TAF LSZH 101601 05020G35KT 8000 BKN015 TEMPO 1720 05018KT 0300 +SHSN VV002 = TAF LSZH 101601 32008KT 9999 SCT030TCU TEMPO 2201 32020G32KT 3000 TSRA BKN020CB =

498. You have been flying for some time in dense layered cloud. The outside air temperature is -25°C. Which of the following statements is true? A. B. C. D.

Severe airframe icing is unlikely under these conditions Severe airframe icing is quite likely under these conditions In a dense layered cloud icing is unlikely also at an outside air temperature of -5°C If you do not have weather radar on board there is no need to worry, as CB is unlikely to form in such cloud

499. Thunderstorms in exceptional circumstances can occur in a warm front if A. B. C. D.

the warm air is unstable. the cold air is unstable. the warm air is stable. the cold air is stable.

500. Low level wind shear is likely to be greatest A. B. C. D.

at the condensation level when there is strong surface friction. at the top of a marked surface-based inversion. at the top of the friction layer. at the condensation level when there is no night radiation.

501. On an aerodrome, when a warm front is approaching A. B. C. D.

QFE and QNH decrease. QFE decreases and QNH increases. QFE and QNH increase. QFE increases and QNH decreases.

502. In which of the areas below does the intertropical convergence zone (ITCZ) have a significant impact? A. B. C. D.

the Atlantic ocean, between latitudes 10°N and 30°N, depending on the time of year. western Africa, where it is situated between the 10°N and 30°N parallels, depending on the time of the year. western Africa between 10°N and 20°N and the northern coasts of the Arabian sea in July. western Africa, at a latitude of 25°N in July.

503. In which of the following situations is an aircraft most susceptible to icing ? A. B. C. D.

Level flight below a rain producing cloud when OAT is below zero degrees C. Level flight in snowfall below a nimbostratus layer. Flying in heavy drizzle. Flying in dense cirrus clouds.

504. Which of the following cloud types are most likely to produce light to moderate icing when they are not subject to orographic lifting and consist of supercooled cloud droplets? A. B. C. D.

Altocumulus and altostratus. Stratocumulus and cirrostratus Stratus and cumulonimbus Altostratus and cirrocumulus

505. A winter day in northern Europe with a thick layer of stratocumulus clouds and temperature close to zero degrees C at ground level, you can expect: A. Reduced visibility and light icing in clouds

ATP Online - Meteorology B. Turbulence due to a strong inversion, but no icing because clouds consist of ice crystals C. A high probability for icing in clouds. Severe icing may occur in the upper part due to accumulation of large droplets. D. Decreasing visibility due to snowfall below cloud base, but only light icing in clouds.

506. Which one of the following statements concerning the formation of aircraft icing is most correct ? A. B. C. D.

Probability of icing increases when dry snow starts to fall from a cloud. Greatest risk of icing conditions is experienced in cirrus clouds. Risk for icing increases when cloud temperature decreases well below minus 12 degrees C. A cloud consisting of both supercooled water droplets and ice crystals produces aircraft icing

507. The troposphere is the A. B. C. D.

part of the atmosphere below the tropopause boundary between the mesosphere and thermosphere part of the atmosphere above the stratosphere boundary between the stratosphere and the mesosphere

508. The tropopause is a level at which A. B. C. D.

vertical currents are strongest water vapour content is greatest temperature ceases to fall with increasing height pressure remains constant

509. The tropopause is lower A. B. C. D.

over the equator than over the South Pole over the North Pole than over the equator south of the equator than north of it in summer than winter in moderate latitudes

510. The temperature at 10000 FT in the ICAO Standard Atmosphere is: A. B. C. D.

-20°C -35°C -5°C 0°C

511. In meteorology, which constituent is considered the most important in the atmosphere's composition? A. B. C. D.

Nitrogen Water vapour Oxygen Hydrogen

512. The average height of the tropopause at 50°N is about A. B. C. D.

8 km 16 km 11 km 14 km

513. The height and the temperature of the tropopause are respectively in the order of A. B. C. D.

8 km and - 75°C over the poles 16 km and -40°C over the poles 16 km and -75°C over the equator 8 km and -40°C over the equator

514. An inversion is a layer of air in which the temperature A. B. C. D.

decreases with height less than 1°C/100m remains constant with height decreases with height more than 1°C/100m increases with height

515. An isothermal layer is a layer of air in which the temperature A. B. C. D.

increases with height at a constant rate decreases with height at a constant rate increases with height remains constant with height

Page 78 of 213

ATP Online - Meteorology 516. An inversion is a layer of air which is A. B. C. D.

absolutely stable absolutely unstable conditionally unstable conditionally stable

517. The environmental lapse rate in an actual atmosphere A. B. C. D.

has a fixed value of 0.65°C/100m varies with time has a fixed value of 2°C/1000 FT has a fixed value of 1°C/100m

518. The dry adiabatic lapse rate has a value of A. B. C. D.

2°C/1000FT 0.65°C/100m 0.5°C/100m 1°C/100m

519. The dry adiabatic lapse rate A. B. C. D.

has a constant fixed value is greater during the night than during the day is greater in summer than in winter has a variable value

520. An air mass is called stable when A. B. C. D.

the environmental lapse rate is high, with little vertical motion of air currents the vertical motion of rising air tends to become weaker and disappears the pressure in a given area is constant the temperature in a given air mass decreases rapidly with height

521. In the ICAO Standard Atmosphere the decrease in temperature with height below 11 km is A. B. C. D.

0.65°C per 100m 0.5°C per 100m 0.6°C per 100m 1°C per 100m

522. Which statement is correct regarding the ICAO Standard Atmosphere ? A. B. C. D.

At MSL temperature is 15°C and pressure is 1013.25hPa At MSL pressure is 1013.25 hPa and the decrease of temperature with height is 1°C per 100m At MSL temperature is 15°C and the decrease in temperature with height is 1°C per 100m At MSL temperature is 10°C and the decrease in temperature with height is 1°C per 100m

523. QNH is defined as A. B. C. D.

pressure at MSL in the standard atmosphere QFE reduced to MSL using the values of the actual atmosphere pressure at MSL in the actual atmosphere QFE reduced to MSL using the values of the standard atmosphere

524. Which of the following statements is true ? A. B. C. D.

QNH is always lower than QFE QNH is always equal to QFE QNH is always higher than QFE QNH can be equal to QFE

525. Which statement is true ? A. B. C. D.

QNH is lower than 1013.25 hPa at any time QNH can not be 1013.25 hPa QNH can be lower as well as higher than 1013.25 hPa QNH can be 1013.25 hPa only for a station at MSL

526. When the subscale is set to the QNH of an airfield the pressure altimeter indicates A. B. C. D.

zero while landing elevation while landing elevation while landing only if conditions are as in the ICAO Standard Atmosphere zero while landing only if conditions are as in the ICAO Standard Atmosphere

Page 79 of 213

ATP Online - Meteorology

Page 80 of 213

527. Advection is: A. B. C. D.

the same as subsidence the same as convection horizontal motion of air vertical motion of air

528. Subsidence is: A. B. C. D.

vertically downwards motion of air the same as convection horizontal motion of air vertically upwards motion of air

529. Which of the following processes is the most important one concerning the heating of the air in the troposphere?; The radiation of the sun heats A. B. C. D.

the air in the troposphere mainly by absorbtion the air in the troposphere only directly if no clouds are present the surface of the earth, which heats the air in the troposphere the water vapour in the air of the troposphere

530. The diurnal variation in temperature is largest when A. B. C. D.

the sky is overcast and the wind is weak the sky is clear and the wind is weak the sky is overcast and the wind is strong the sky is clear and the wind is strong

531. Rising air cools because A. B. C. D.

it expands surrounding air is cooler at higher levels it contracts it becomes more moist

532. A layer can be A. B. C. D.

unstable for unsaturated air and neutral for saturated air unstable for unsaturated air and conditionally unstable stable for saturated air and unstable for unsaturated air stable for unsaturated air and unstable for saturated air

533. In a layer of air the decrease in temperature per 100 metres increase in height is more than 1°C. This layer can be described as being A. B. C. D.

conditionally unstable absolutely unstable absolutely stable conditionally stable

534. Which statement is true for a conditionally unstable layer? A. B. C. D.

The layer is unstable for unsaturated air The environmental lapse rate is less than 1°C/100m The environmental lapse rate is less than 0.65°C/100m The wet adiabatic lapse rate is 0.65°C/100m

535. The stability in a layer is increasing if A. B. C. D.

warm and moist air is advected in the lower part cold and dry air is advected in the upper part warm air is advected in the lower part and cold air in the upper part warm air is advected in the upper part and cold air in the lower part

536. Which of the following statements concerning the lifting of a parcel of air is correct ? A. B. C. D.

Unsaturated parcels cool at a rate of 0.65°C per 100m Unsaturated parcels cool less rapidly than saturated parcels Unsaturated parcels cool more rapidly than saturated parcels Saturated parcels always cool at a rate of 0.65°C per 100m

537. When in the upper part of a layer warm air is advected the A. wind will back with increasing height in the northern hemisphere

ATP Online - Meteorology B. stability increases in the layer C. wind speed will always decrease with increasing height in the northern hemisphere D. stability decreases in the layer

538. The dewpoint temperature A. B. C. D.

is always higher than the air temperature is always lower than the air temperature can not be equal to the air temperature can be equal to the air temperature

539. Relative humidity depends on A. B. C. D.

moisture content and pressure of the air temperature of the air only moisture content of the air only moisture content and temperature of the air

540. The dewpoint temperature A. B. C. D.

is always higher than the air temperature can not be equal to the air temperature can be equal to the air temperature is always lower than the air temperature

541. When water evaporates into unsaturated air A. B. C. D.

heat is absorbed heat is released relative humidity is not changed relative humidity is decreased

542. A moist but unsaturated parcel of air becomes saturated by A. B. C. D.

lowering the parcel to a lower level moving the parcel to an area with higher pressure and equal temperature lifting the parcel to a higher level moving the parcel to an area with lower pressure and equal temperature

543. A sample of moist but unsaturated air may become saturated by A. B. C. D.

expanding it adiabatically raising the temperature lowering the pressure, keeping temperature constant compressing it adiabatically

544. Precipitation in the form of showers occurs from A. B. C. D.

stratified clouds convective clouds cirro-type clouds clouds containing only ice crystals

545. A super-cooled droplet is A. B. C. D.

a droplet still in liquid state at a temperature below 0°C a water droplet that is mainly frozen a small particle of water at a temperature below -50°C a water droplet that has been frozen during its descent

546. Steady precipitation, in contrast to showery precipitation falls from A. B. C. D.

stratiform clouds with little or no turbulence convective clouds with little or no turbulence stratiform clouds with severe turbulence convective clouds with moderate turbulence

547. The maximum amount of water vapour that the air can contain depends on the A. B. C. D.

stability of the air relative humidity air temperature dewpoint

Page 81 of 213

ATP Online - Meteorology

Page 82 of 213

548. Convective clouds are formed A. B. C. D.

in mid-latitudes only in summer during the day only in stable atmosphere in unstable atmosphere

549. Altostratus clouds are classified as A. B. C. D.

medium level clouds high level clouds low level clouds convective clouds

550. The most effective way to dissipate cloud is by A. B. C. D.

subsidence convection a decrease in temperature a decrease in pressure

551. A cumulonimbus cloud at mid-latitudes in summer contains A. B. C. D.

ice crystals, water droplets and supercooled water droplets ice crystals and water droplets but never supercooled water droplets only ice crystals only water droplets

552. Strongly developed cumulus clouds are an indication of A. B. C. D.

instability in the atmosphere the presence of warm air aloft the presence of a low level inversion poor surface visibility

553. Areas of sinking air are generally cloudless because as air sinks it A. B. C. D.

is heated by expansion reaches warmer layers loses water vapour is heated by compression

554. The main factor which contributes to the formation of very low clouds ahead of a warm front is the A. B. C. D.

saturation of the warm air by rain falling into it and evaporating reduction of outgoing radiation due to clouds saturation of the cold air by rain falling into it and evaporating warm air moving over a cold surface

555. Clouds, classified as being low level are considered to have bases from A. B. C. D.

1000 to 2000 FT 500 to 1000 FT 100 to 200 FT the surface to 6500 FT

556. Which of the following are medium level clouds ? A. B. C. D.

Cirrocumulus and cirrostratus Cumulonimbus Altostratus and altocumulus All convective clouds

557. In an unstable layer there are cumuliform clouds. The vertical extent of these clouds depends on the A. B. C. D.

air pressure at the surface wind direction thickness of the unstable layer pressure at different levels

558. A moist, stable layer of air is forced to rise against a mountain range and the stability does not change. Which of the following phenomena may develop? A. Showers and thunderstorms B. Areas of severe turbulence C. Stratified clouds

ATP Online - Meteorology

Page 83 of 213

D. Inversions

559. The height of the lifting condensation level is determined by A. B. C. D.

temperature at surface and air pressure wet adiabatic lapse rate and dewpoint at the surface wind and dewpoint at the surface temperature and dewpoint at the surface

560. Cumulus clouds are an indication for A. B. C. D.

stability up and downdrafts the approach of a warm front the approach of a cold front

561. What type of cloud is being described ? A generally grey cloud layer with fairly uniform base and uniform appearance, which may give drizzle or snow grains. When the sun is visible through the cloud, the outline is clearly discernible. Sometimes it appears in the form of ragged patches. A. B. C. D.

Altostratus Cirrostratus Stratus Nimbostratus

562. The presence of altocumulus castellanus indicates A. B. C. D.

strong convection at low height stability in the higher troposphere subsidence in a large part of the troposphere instability in the middle troposphere

563. The presence of altocumulus lenticularis is an indication of the A. B. C. D.

development of thermal lows risk of orographic thunderstorms presence of mountain waves presence of valley winds

564. When in summer calm and clear conditions exist, a station on the shore of a large body of water will experience wind A. B. C. D.

from the land in daytime and from the water at night continually from land to water from the water in daytime and from the land at night continually from water to the land

565. The greater the pressure gradient the A. B. C. D.

closer the isobars and the lower the temperatures further the isobars will be apart and the weaker the wind closer the isobars and the stronger the wind further the isobars will be apart and the higher the temperature

566. When isobars, for an area in the mid-latitudes on a weather map, are close together, the wind is most likely to be A. B. C. D.

changing direction rapidly light strong blowing perpendicular to the isobars

567. In the northern hemisphere a pilot flying at 1000 FT/AGL directly towards the centre of a low pressure area, will find the wind blowing from A. B. C. D.

about 45 degrees to the right of directly ahead right and behind left and behind directly ahead

568. In the northern hemisphere the wind at the surface blows A. B. C. D.

clockwise around, and away from the centre of, a low pressure area counter-clockwise around, and toward the centre of, a low pressure area counter-clockwise around, and away from the centre of, a high pressure area. from a low pressure area to a high pressure area

ATP Online - Meteorology

Page 84 of 213

569. Wind is caused by A. B. C. D.

the movements of fronts friction between the air and the ground the rotation of the earth horizontal pressure differences

570. During a descent from 2000 FT above the surface to the surface (no frontal passage, northern hemisphere), the wind normally A. B. C. D.

veers and decreases veers and increases backs and decreases backs and increases

571. The Foehn wind is a A. B. C. D.

warm katabatic wind warm anabatic wind cold katabatic wind cold anabatic wind

572. The sea breeze is a wind from the sea A. B. C. D.

occurring only in the lower layers of the atmosphere in daytime that reaches up to the tropopause in daytime blowing at night in mid-latitudes occurring only in mid-latitudes and in daytime

573. Low level vertical wind shear can be expected during the night A. B. C. D.

in unstable atmospheres and early morning only in summer and early morning only in winter in association with radiation inversions

574. In an area of converging air in low level A. B. C. D.

convective clouds can be dissolved stratified clouds can be dissolved clouds can be formed clouds can not be formed

575. You are flying from east to west in the northern hemisphere at the 500 hPa pressure surface. Which of the following statements is correct? A. B. C. D.

If the wind is from the south you are gaining altitude If you have a tail wind you are losing altitude If the wind is from the north you are gaining altitude If you have a head wind you are gaining altitude

576. Geostrophic wind is the wind when isobars are A. B. C. D.

straight lines and no friction is involved. straight lines and friction is involved. curved lines and friction is involved. curved lines and no friction is involved

577. Fair weather cumulus often is an indication of A. B. C. D.

turbulence at and below the cloud level smooth flying conditions below the cloud level a high risk of thunderstorms poor visibility at surface

578. What relationship exists between the wind at 3000 feet and the surface wind? A. B. C. D.

They are practically the same, except when eddies exist, caused by obstacles The wind at 3000 feet is parallel to the isohypses and the surface wind direction is across the isobars toward the low pressure and the surface wind is weaker. The surface wind is veered compared to the wind at 3000 feet and is usually weaker. They have the same direction, but the surface wind is weaker, caused by friction

579. The wind tends to follow the contour lines (isohypses) above the friction layer because A. the coriolis force tends to balance with the horizontal pressure gradient force B. the friction of the air with the earth's surface gives the airflow a diversion perpendicular to the gradient force. C. the coriolis force acts perpendicular on a line that connects high and low pressure system

ATP Online - Meteorology

Page 85 of 213

D. contour lines are lines that connect points with the same wind speed in the upper air

580. The wind speed in a system with curved isobars compared to a system with straight isobars is (other conditions being the same) A. B. C. D.

higher if curvature is anticyclonic always lower higher if curvature is cyclonic always higher

581. Divergence in the upper air that is greater than the convergence in the friction layer results, near the surface, in A. B. C. D.

rising pressure and likely formation of clouds rising pressure and likely dissipation of clouds falling pressure and likely dissipation of clouds falling pressure and likely formation of clouds

582. The geostrophic wind depends on A. B. C. D.

earth's rotation, geographic latitude, centripetal force geographic latitude, centripetal force, height density, earth's rotation, geographic latitude centripetal force, height, pressure gradient

583. In a mountain-valley wind circulation, the mountain wind blows A. B. C. D.

at night down from the mountains during the day up from the valley at night up from the valley during the day down from the mountains

584. Ahead of a warm front (northern hemisphere) the wind direction changes from the surface up to the tropopause. The effect of this change is that the wind A. B. C. D.

backs in the friction layer and veers above the friction layer veers in the friction layer and backs above the friction layer veers in the friction layer and veers above the friction layer backs in the friction layer and backs above the friction layer

585. The difference between geostrophic wind and gradient wind is caused by A. B. C. D.

friction slope of pressure surfaces horizontal temperature gradients curvature of isobars

586. The vertical extent of the friction layer depends primarily on A. B. C. D.

roughness of surface, temperature, local time temperature, local time, environmental lapse rate stability, wind speed, roughness of surface wind speed, roughness of surface, temperature

587. In the northern hemisphere, during periods of prolonged clear skies associated with anticyclonic conditions, the A. B. C. D.

angle between isobars and surface wind direction tends to be greatest in the early afternoon wind tends to back from early morning until early afternoon surface wind speed tends to be highest at night surface wind speed tends to be highest during the early afternoon

588. The Chinook is a A. B. C. D.

warm and dry wind that forms as air descends on the leeward side of the Rocky Mountains very cold wind with blowing snow downslope wind that occurs particularly at night as air cools along mountain slopes warm anabatic wind up the slopes of snowfields or glaciers

589. The geostrophic wind speed is directly proportional to the A. B. C. D.

sine of latitude curvature of isobars horizontal pressure gradient density of the air

590. A strong, dry and warm downslope wind, produced by prior enforced ascent of air over hills or mountains is known as a

ATP Online - Meteorology A. B. C. D.

Page 86 of 213

Bora Foehn Scirocco Mistral

591. A dry, sand- and dust-laden North Easterly wind that blows in winter over large parts of North West Africa is known as a A. B. C. D.

Harmattan Pampero Khamsin Scirocco

592. Geostrophic wind A. B. C. D.

veers with height if cold air is advected in the northern hemisphere is directly proportional to the density of the air is perpendicular to the horizontal pressure gradient force always increases with increasing height

593. In a low pressure system the convergence at the surface is caused by A. B. C. D.

centripetal forces frictional forces the inbalance of the horizontal gradient force and the Coriolis force the curvature of the isobars

594. An observer on the northern hemisphere is under influence of the wind system of a depression, which is moving from West to East. The centre of the depression passes to the South of the observer. For this observer the wind direction is A. B. C. D.

continuously veering continuously backing initially backing, then veering initially veering, then backing

595. When the temperature and dew point are less than one degree apart the weather conditions are most likely to be A. B. C. D.

clear and cool fog or low cloud high scattered clouds unlimited visibility

596. The morning following a clear, calm night when the temperature has dropped to the dewpoint, is likely to produce A. B. C. D.

a cold front advection fog good clear weather radiation fog

597. Advection fog can be formed when A. B. C. D.

cold moist air flows over warmer water warm moist air flows over a warmer surface cold moist air flows over a warmer surface warm moist air flows over a colder surface

598. Steaming fog (arctic sea smoke) occurs in air A. B. C. D.

that is absolutely stable that is stable with cold mass properties with warm mass properties

599. Frontal fog is most likely to occur A. B. C. D.

in summer in the early morning in winter in the early morning in advance of a warm front in rear of a warm front

600. Visibility is reduced by haze when A. B. C. D.

a light drizzle falls small water droplets are present a cold front just passed dust particles are trapped below an inversion

ATP Online - Meteorology

Page 87 of 213

601. Freezing fog exists if fog droplets A. B. C. D.

are freezing very rapidly freeze when temperature falls below zero are supercooled are frozen

602. Runway Visual Range (RVR) is A. B. C. D.

usually better than visibility reported when visibility is less than 2000m reported in AIRMET and METAR measured with ceilometers alongside the runway

603. Which of the following circumstances most favour the development of radiation fog? A. B. C. D.

Moist air over land during clear night with little wind Advection of very cold air over much warmer sea Warm moist air at the windward side of a mountain Maritime tropical air flowing over cold sea

604. In a warm front occlusion A. B. C. D.

the cold air is lifted the warm front overtakes the cold front the warm air is lifted the warm front becomes a front aloft

605. If the pressure surfaces bulge upwards in all levels then the pressure system is a A. B. C. D.

cold low cold high warm low warm high

606. A cold air pool A. occurs frequently in winter to the south of the Alps when this region is under the influence of cold north-westerly airstream B. normally disappears at night and occurs almost exclusively in summer C. develops usually in winter when very unstable maritime polar or maritime arctic air currents stream southwards along the eastern side of an extensive ridge of high pressure, in association with occluded systems D. is usually most evident in the circulation and temperature fields of the middle troposphere and may show little or no sign on a surface chart

607. The stable layer at some height in the low troposphere of an older high pressure area in the mid-latitudes is called A. B. C. D.

trade wind inversion subsidence inversion friction inversion radiation inversion

608. A blocking anticyclone in the northern hemisphere is A. B. C. D.

a warm anticyclone/quasi stationary/situated between 50°N and 70°N a cold anticyclone/steering depressions/situated over Scandinavia situated between 50°N and 70°N/a cold anticyclone/steering depressions quasi stationary/situated between 50°N and 70°N/a cold anticyclone

609. The transition from SW to NE monsoon in India occurs in A. B. C. D.

December, January, February July, August, September February, March, April September, October, November

610. Which of the following statements concerning the intertropical convergence zone is true? A. B. C. D.

It does not change its position over the oceans during the year It lies totally in the northern hemisphere in July and totally in the southern hemisphere in January There are frequent occurrences of CB It is an area of low pressure and low relative humidity

611. An easterly wave is a A. wave in a trade wind belt, moving from east to west, with severe convective activity in rear of its trough

ATP Online - Meteorology

Page 88 of 213

B. small scale wave disturbance in the tropics, moving from east to west, with severe convective activity ahead of its trough C. wave-like disturbance in the monsoon regime of India, moving from east to west, with severe convective activity ahead of its trough D. disturbance in the higher levels associated with the equatorial easterly jet, moving from east to west, with severe convective activity in rear of its trough

612. The prevailing surface wind in the area of the west coast of Africa north of the equator (gulf of Guinea) is a A. B. C. D.

NE monsoon in winter and SE tradewind in summer SW monsoon in summer and NE tradewind in winter SW monsoon in winter and NE monsoon in summer NE tradewind in summer and SE tradewind in winter

613. What information is given on a Significant Weather Chart? A. B. C. D.

The significant weather forecast for the time given on the chart The significant weather that is observed at the time given on the chart The significant weather forecast for a period 6 hours after the time given on the chart The significant weather in a period 3 hours before and 3 hours after the time given on the chart

614. The most dangerous low level wind shears are encountered A. B. C. D.

near valleys and at the windward side of mountains. when strong ground inversions are present and near thunderstorms in areas with layered clouds and wind speeds higher than 35 kt during any period when wind speed is greater than 35 kt and near valleys

615. Vertical wind shear is A. B. C. D.

a change of vertical wind speed with horizontal distance a horizontal shear of vertical wind a change of horizontal wind direction and/or speed with horizontal distance a change of horizontal wind direction and/or speed with height

616. The most hazardous type of cloud that may be encountered on a cross country flight is A. B. C. D.

Cumulonimbus stratocumulus cirrus cumulus

617. During the life cycle of a thunderstorm, which stage is characterized predominantly by downdrafts? A. B. C. D.

Mature stage Dissipating stage Tornado stage Initial stage

618. What feature is normally associated with the initial stage of a thunderstorm? A. B. C. D.

Continuous updraft Roll cloud Rain or hail at the surface Frequent lightning

619. Large hail stones A. B. C. D.

only occur in thunderstorms of mid-latitudes only occur in frontal thunderstorms are entirely composed of clear ice are typically associated with severe thunderstorms

620. Which of the statements is true concerning squall lines ? A. B. C. D.

For severe squall lines a TAF is issued For severe squall lines a SIGMET is issued Severe squall lines only occur in the tropics Severe squall lines always move from northwest to southeast

621. A gust front is A. B. C. D.

formed by the cold air outflow from a thunderstorm another name for a cold front normally encountered directly below a thunderstorm characterized by heavy lightning

ATP Online - Meteorology 622. What are the requirements for the formation of a thunderstorm? A. B. C. D.

An adequate supply of moisture, conditional instability and a lifting action Water vapour and high pressure A stratocumulus cloud with sufficient moisture A cumulus cloud with sufficient moisture associated with an inversion

623. In which of the following areas is the highest frequency of thunderstorms encountered ? A. B. C. D.

Temperate Tropical Subtropical Polar

624. A microburst with its damaging winds at the surface A. B. C. D.

occurs only in tropical areas is always associated with thunderstorms has a life time of more than 30 minutes has a diameter up to 4 km

625. Thunderstorms can occur on a warm front if the A. B. C. D.

cold air is moist and the environmental lapse rate is less than the dry adiabatic lapse rate cold air is moist and the environmental lapse rate exceeds the saturated adiabatic lapse rate warm air is moist and the environmental lapse rate is less than the saturated adiabatic lapse rate warm air is moist and the environmental lapse rate exceeds the saturated adiabatic lapse rate

626. With which type of cloud are tornadoes associated ? A. B. C. D.

Stratus Cumulonimbus Nimbostratus Cumulus mediocris

627. The diameter of a typical tornado is A. B. C. D.

in the order of 10 km only a few metres about 2 to 6 km 100 to 150 metres

628. In which stage of the life cycle of a single thunderstorm cell occur both up- and downdrafts simultaneously? A. B. C. D.

In all stages Dissipating stage Cumulus stage Mature stage

629. Fallstreaks or virga are A. B. C. D.

water or ice particles falling out of a cloud that evaporate before reaching the ground strong downdraughts in the polar jet stream, associated with jet streaks gusts associated with a well developed Bora strong katabatic winds in mountainous areas and accompanied by heavy precipitation

630. Supercooled droplets can occur in A. B. C. D.

clouds but not in precipitation clouds, fog and precipitation precipitation but not in clouds clouds but not in fog

631. During the formation of rime ice in flight, water droplets freeze A. B. C. D.

rapidly and spread out slowly and spread out slowly and do not spread out rapidly and do not spread out

632. Supercooled droplets are always A. B. C. D.

large and at a temperature below freezing at a temperature below freezing small and at a temperature below freezing at a temperature below -60°C

Page 89 of 213

ATP Online - Meteorology

Page 90 of 213

633. Supercooled droplets can be encountered A. B. C. D.

in winter only in high clouds only in winter at high altitude only in winter above 10000 FT at any time of the year

634. Clear ice is formed when supercooled droplets are A. B. C. D.

of any size at temperatures below -35°C. small and at a temperature just below freezing small and freeze rapidly large and at a temperature just below freezing

635. During an adiabatic process heat is A. B. C. D.

added lost neither added nor lost added but the result is an overall loss

636. The decrease in temperature, per 100 metres, in an unsaturated rising parcel of air is A. B. C. D.

0.5°C 0.65°C 2°C 1°C

637. The decrease in temperature, per 100 metres, in a saturated rising parcel of air at lower level of the atmosphere is approximately A. B. C. D.

0.6°C 0.35°C 1°C 1.5°C

638. The following temperatures have been observed over a station at 1200 UTC. Assume the station is at MSL. Height in feet. Temperature in degrees C. 20000.-12 18000.-11 16000.-10 14000.-10 12000.-6 10000.-2 8000. +2 6000. +6 4000. +12 2000. +15 surface+15. Which of the following statements is correct ? A. B. C. D.

Assuming that the MSL pressure is 1013.25 hPa the true altitude of an aircraft would actually be higher than the indicated altitude. The height of the freezing level over the station is approximately 12000 FT. The layer between 16000 and 18000 FT is absolutely unstable The temperature at 10000 FT is in agreement with the temperature in the International Standard Atmosphere.

639. In unstable air, surface visibility is most likely to be restricted by A. B. C. D.

drizzle showers of rain or snow haze low stratus

640. A vertical temperature profile indicates the possibility of severe icing when the temperature profile A. B. C. D.

intersects the 0°C isotherm twice indicates temperatures above 3°C coincides with a dry adiabatic lapse rate indicates temperatures below -40°C

641. A pilot is warned of severe icing at certain flight levels by information supplied in A. B. C. D.

TAF and METAR SWC and SIGMET METAR and SIGMET TAF and SIGMET

642. Freezing precipitation occurs A. B. C. D.

mainly in the form of freezing rain or freezing drizzle only in the precipitation of a cold front mainly in the form of freezing hail or freezing snow only in the precipitation of a warm front

643. In the vicinity of industrial areas, smoke is most likely to affect surface visibility when

ATP Online - Meteorology A. B. C. D.

Page 91 of 213

there is a low level inversion the surface wind is strong and gusty cumulus clouds have developed in the afternoon a rapid moving cold front has just passed the area

644. The rate of cooling of ascending saturated air is less than the rate of cooling of ascending unsaturated air because: A. B. C. D.

moist air is heavier than dry air heat is released during the condensation process water vapour doesn't cool as rapidly as dry air water vapour absorbs the incoming heat from the sun

645. If the surface temperature is 15°C , then the temperature at 10000 FT in a current of ascending unsaturated air is: A. B. C. D.

-15°C 0°C -5°C 5°C

646. An inversion is A. B. C. D.

an increase of pressure with height a decrease of pressure with height an increase of temperature with height a decrease of temperature with height

647. In still air the temperature decreases at an average of 1.2°C per 100 m increase in altitude. This temperature change is called: A. B. C. D.

Environmental lapse rate normal lapse rate saturated adiabatic lapse rate dry adiabatic lapse rate

648. Freezing fog consists of A. B. C. D.

ice crystals frozen water droplets frozen minute snow flakes supercooled water droplets

649. Clear ice is dangerous because it A. B. C. D.

spreads out and contains many air particles is heavy and is difficult to remove from the aircraft surfaces is not translucent and forms at the leading edges is translucent and only forms at the leading edges

650. Hoar frost forms as a result of A. B. C. D.

water vapour turning directly into ice crystals on the aircraft surface droplets forming on the aircraft and then freezing small super-cooled droplets striking the aircraft freezing rain striking the aircraft

651. An air mass is unstable when A. B. C. D.

temperature increases with height pressure shows a marked variation over a given horizontal area an ascending parcel of air continues to rise to a considerable height. temperature and humidity are not constant

652. An air mass is stable when A. B. C. D.

temperature in a given area drops off very rapidly with height lifted air returns to its original level the lapse rate is 1°C per 100 m pressure is constant

653. On a clear summer day, turbulence caused by solar heating is most pronounced A. B. C. D.

during the early afternoon about midmorning during early morning hours before sunrise immediately after sunset

ATP Online - Meteorology

Page 92 of 213

654. A surface based inversion is a characteristic of A. B. C. D.

nocturnal radiation during clear nights hill fog the passage of cold front cumulus clouds

655. Clear ice forms as a result of A. B. C. D.

ice pellets splattering on the aircraft water vapour freezing to the aircraft supercooled droplets freezing on impact supercooled water droplets spreading during the freezing process

656. From which of the following pieces of information can the stability of the atmosphere be derived? A. B. C. D.

Environmental lapse rate Surface temperature Dry adiabatic lapse rate Pressure at the surface

657. The type of icing that occurs in dense clouds with large supercooled drops that have a temperature of -5°C is most likely to be A. B. C. D.

rime ice cloudy ice hoar frost clear ice

658. In order to reduce QFE to QNH, which of the following item(s) must be known ? A. B. C. D.

Elevation of the airfield and the temperature at MSL Elevation of the airfield Temperature at the airfield Elevation of the airfield and the temperature at the airfield

659. Which statement is true for hurricanes? A. B. C. D.

They intensify rapidly after landfall From the earth's surface up to the tropopause the core is warmer than its surroundings Their greatest frequency of occurrence is in winter The diameter is 50-500 m

660. Assume that an aircraft is flying in the northern hemisphere at the 500 hPa pressure surface on a heading of 270 degrees. Which of the following statements is correct? A. B. C. D.

If in this pressure surface the wind comes from the direction 360 degrees, then true altitude is increasing If in this pressure surface the wind comes from the direction 090 degrees, then true altitude is increasing If in this pressure surface the wind comes from the direction 180 degrees, then true altitude is increasing If in this pressure surface the wind comes from the direction 270 degrees, then true altitude is increasing

661. Which of the following statements concerning jet streams is correct? A. B. C. D.

In the northern hemisphere both westerly and easterly jet streams occur In the southern hemisphere only easterly jet streams occur In the southern hemisphere no jet streams occur In the northern hemisphere only westerly jet streams occur

662. Under which of the following conditions is the most severe CAT likely to be experienced ? A. B. C. D.

A westerly jet stream at low latitudes in the summer A jet stream, with great spacing between the isotherms A curved jet stream near a deep trough A straight jet stream near a low pressure area

663. An aircraft is flying at FL 180 in the northern hemisphere with a crosswind from the left. Which of the following is correct concerning its true altitude ? A. B. C. D.

It remains constant Without knowing temperatures at FL 180 this question can not be answered. It increases It decreases

664. A layer is absolutely unstable if the temperature decrease with height is A. 0.65°C per 100m B. less than 0.65°C per 100m

ATP Online - Meteorology

Page 93 of 213

C. between 1°C per 100m and 0.65°C per 100m D. more than 1°C per 100m

665. A layer in which the temperature remains constant with height is A. B. C. D.

neutral conditionally unstable unstable absolutely stable

666. A layer in which the temperature increases with height is A. B. C. D.

conditionally unstable absolutely stable absolutely unstable neutral

667. A layer in which the temperature decreases with 1°C per 100m increasing altitude is A. B. C. D.

neutral for dry air absolutely unstable absolutely stable conditionally unstable

668. If in a 100 m thick layer the temperature at the bottom of the layer is 10°C and at the top of the layer is 8°C then this layer is A. B. C. D.

conditionally unstable absolutely unstable neutral absolutely stable

669. An inversion is A. B. C. D.

an increase of pressure with height a decrease of temperature with height an increase of temperature with height a decrease of pressure with height

670. Below a low level inversion visibility is often A. B. C. D.

moderate or poor because there is no vertical exchange very good at night very good in the early morning moderate or poor due to heavy snow showers.

671. Above and below a low level inversion the wind is likely to A. B. C. D.

change in speed but not in direction experience little or no change in speed and direction change significantly in speed and direction change in direction but not in speed

672. Which of the following is true concerning an aircraft that is flying at FL180 in the northern hemisphere, where wind is geostrophic and the true altitude remains constant ? A. B. C. D.

Without knowing temperature at FL 180 this question can not be answered There is a cross wind from the left There is no cross wind There is a cross wind from the right

673. Which of the following statements concerning the core of a polar front jet stream is correct ? A. B. C. D.

It lies in the warm air; its pressure surfaces are horizontal at the height of the core It lies at a height where there is no horizontal temperature gradient; the slope of the pressure surfaces at the height of the core is at its maximum It lies in the cold air; the wind reverses direction at the height of the core It and its surface projection lie in the warm air

674. On a particular day part of a polar front jet stream runs from north to south in the northern hemisphere. This means that A. B. C. D.

the polar air is below and to the east of the core of the jet above the core of the jet the horizontal temperature gradient runs from north to south the polar air is on the eastern side and above the core of the jet below the core of the jet the horizontal temperature gradient runs from north to south

ATP Online - Meteorology

Page 94 of 213

675. An isohypse of the 500 hPa pressure surface is labelled with the number 552. This means that for all points on the isohypse the A. B. C. D.

pressure altimeter will overread by 552 FT topography is 552 decameters above MSL pressure is 552 hPa topography is 552 meters above MSL

676. Which of the following statements concerning trade winds is correct? A. B. C. D.

They occur only in the lower part of the troposphere and more pronounced over the oceans They reach up to the tropopause and are more pronounced over the oceans They reach up to the tropopause and are more pronounced over the continents They occur only in the lower part of the troposphere and are more pronounced over the continents

677. In the central part of the Atlantic Ocean between 10°N and 20°N the prevailing winds are A. B. C. D.

NE monsoon in winter and SW monsoon in summer SE trade winds NE trade winds SW winds throughout the whole year

678. Along the West coast of India the prevailing winds are the A. B. C. D.

SW monsoon in July and a NE monsoon in January SE monsoon in July and a SW monsoon in January SW monsoon in July and a SE monsoon in January NE monsoon in July and a SW monsoon in January

679. The validity of a routine TAF is A. B. C. D.

9 hours from the time of issue 2 hours stated in the TAF between 6 and 9 hours

680. ATIS information contains A. B. C. D.

operational information and if necessary meteorological information only meteorological information meteorological and operational information only operational information

681. If CAVOK is reported then A. B. C. D.

any CB's have a base above 5000 FT no low drifting snow is present low level wind shear has not been reported no clouds are present

682. Runway visual range can be reported in A. B. C. D.

both a TAF and a METAR a METAR a TAF a SIGMET

683. SIGMET information is issued as a warning for significant weather to A. B. C. D.

heavy aircraft only VFR operations only light aircraft only all aircraft

684. The wind direction in a METAR is measured relative to A. B. C. D.

magnetic north the 0-meridian grid north true north

685. A SPECI is A. B. C. D.

a warning for special weather phenomena an aviation routine weather report an aviation special weather report a forecast for special weather phenomena

ATP Online - Meteorology

Page 95 of 213

686. On the European continent METARs of main airports are compiled and distributed with intervals of A. B. C. D.

1 hour 0.5 hour 2 hours 3 hours

687. The RVR, as reported in a METAR, is always the A. B. C. D.

value representative of the touchdown zone average value of the A-, B- and C-position lowest value of the A-, B- and C-position highest value of the A-, B- and C-position

688. The cloud base, reported in the METAR, is the height above A. B. C. D.

the pressure altitude of the observation station at the time of observation airfield level mean sea level the highest terrain within a radius of 8 km from the observation station

689. The QNH is equal to the QFE if A. B. C. D.

T actual < T standard T actual = T standard the elevation = 0 T actual > T standard

690. Which of the following conditions gives the highest value of the QNH? A. B. C. D.

QFE = 995 hPa, elevation = 1600 FT (488m) QFE = 1000 hPa, elevation = 1200 FT (366m) QFE = 995 hPa, elevation = 1200 FT (366m) QFE = 1003 hPa, elevation = 1200 FT (366m)

691. The pressure altitude is equal to the true altitude if A. B. C. D.

the indicated altitude is equal to the pressure altitude the outside air temperature is standard for that height standard atmospheric conditions occur the air pressure is 1013.25 hPa at the surface

692. You must make an emergency landing at sea. The QNH of a field on a nearby island with an elevation of 4000 FT is 1025 hPa and the temperature is -20°C. What is your pressure altimeter reading when landing if 1025 hPa is set in the subscale? A. B. C. D.

Less than 0 FT 0 FT 4000 FT More than 0 FT, but less than 4000 FT

693. The troposphere A. B. C. D.

reaches the same height at all latitudes contains all oxygen of the stratosphere is the separation layer between the stratosphere and atmosphere has a greater vertical extent above the equator than above the poles

694. Which units are used for the forecasted wind at higher levels? A. B. C. D.

Direction in degrees relative to magnetic north and speed in km/h Direction in degrees relative to magnetic north and speed in knots Direction in degrees relative to true north and speed in knots Direction in degrees relative to grid north and speed in km/h

695. A wide body takes off on a clear night in Dhahran, Saudi Arabia. Shortly after take off the aircraft's rate of climb drops to zero. This can be due to A. B. C. D.

low relative humidity sand/dust in the engines very pronounced downdrafts a very strong temperature inversion

696. Given the following METAR: EDDM 250850Z 33005KT 2000 R26R/P1500N R26L/1500N BR SCT002 OVC003 05/05 Q1025 NOSIG Which of the following statements is correct ?

ATP Online - Meteorology A. B. C. D.

Page 96 of 213

Visibility is reduced by water droplets There is a distinct change in RVR observed Runway 26R and runway 26L have the same RVR RVR on runway 26R is increasing

697. Flight visibility from the cockpit during approach in a tropical downpour can decrease to minimal A. B. C. D.

about 500 metres about 1000 metres tens of metres about 200 metres

698. In what hPa range is an upper weather chart for FL 340 situated? A. B. C. D.

400 - 300 hPa 500 - 400 hPa 300 - 200 hPa 600 - 500 hPa

699. Isobars on a surface chart are lines of equal A. B. C. D.

QNE QNH QFE QFF

700. The Bora is a A. B. C. D.

squally warm catabatic wind which occurs mainly in summer cold catabatic wind always associated with clouds and heavy showers cold catabatic wind with the possibility of violent gusts cold catabatic wind with gusts associated with a maritime air mass

701. The range of wind speed in which radiation fog is most likely to form is: A. B. C. D.

above 15 kt below 5 kt between 5 and 10 kt between 10 and 15 kt

702. For a given airfield the QFE is 980 hPa and the QNH is 1000 hPa. The approximate elevation of the airfield is A. B. C. D.

540 metres 120 metres 600 metres 160 metres

703. Which of the following factors have the greatest effect on the formation of the various types of ice on an aircraft ? A. B. C. D.

Relative humidity inside the cloud Aircraft speed and curvature of the airfoil Aircraft speed and size of cloud droplets Cloud temperature and droplet size

704. Hoar frost is most likely to form when A. B. C. D.

taking off from an airfield with a significant ground inversion (sky clear). flying in supercooled drizzle. flying inside convective clouds. flying inside stratiform clouds.

705. Two aircraft, one with a sharp wing profile (S), and the other with a thick profile (T), are flying through the same cloud with same true airspeed. The cloud consists of small supercooled droplets. Which of the following statements is most correct concerning ice accretion ? A. B. C. D.

Aircraft S experiences more icing than T. Aircraft T experiences more icing than S. Neither of the aircraft accumulate ice due to the small size of droplets. Aircraft S and T experience the same amount of icing

706. While descending through a cloud cover at high level, a small amount of a white and rough powderlike contamination is detected along the leading edge of the wing. This contamination is called: A. B. C. D.

Frost. Rime ice. Mixed ice. Clear ice.

ATP Online - Meteorology

Page 97 of 213

707. A small supercooled cloud droplet that collides with an airfoil will most likely A. B. C. D.

travel back over the wing, creating rime ice. travel back over the wing, creating clear ice. freeze immediately and create rime ice. freeze immediately and create clear ice.

708. What is the name of the northerly, cold and strong wind, that sometimes blows over a certain part of Europe? A. B. C. D.

Typhoon. Foehn. Sirocco Mistral.

709. What are the characteristics of the Bora ? A. B. C. D.

It is a dry and hot southerly wind experienced in the Sahara desert, that often carries dust. It is a cold and very strong wind that blows mainly in winter from a tableland downwards to the Adriatic It is a warm and moist, south-westerly wind experienced in the eastern Mediterranean, that usually carries precipitation. It is a very cold wind that blows mainly in winter from a north-westerly direction in the Mediterranean

710. The reason for the fact, that the atmospheric pressure of a polar front depression is normally lower in winter than in summer is that A. B. C. D.

the temperature contrasts between arctic and equatorial areas are much greater in winter. converging air currents are of greater intensity in winter. the low pressure activity of the sea east of Canada is higher in winter. the strong winds of the north Atlantic in winter are favourable for the development of lows.

711. Which one of the following statements is correct concerning the movement of the ITCZ in the region of West Africa? A. B. C. D.

It reaches its maximum southerly position of 5° S in January It reaches its maximum northerly position of 15° - 20° N in July It oscillates during the year between 10 degrees North and 10 degrees South. It oscillates during the year between the Equator and 10 degrees North.

712. What is the name of the wind or air mass which gives to the main part of India its greatest proportion of precipitation? A. B. C. D.

South-west monsoon. Winter monsoon. Indian, maritime tropical air mass. South-east trade wind.

713. What is the type, intensity and seasonal variation of precipitation in the equatorial region ? A. B. C. D.

Warm fronts are common with continuous rain. The frequency is the same throughout the year Showers of rain or hail occur throughout the year; the frequency is highest in January. Rainshowers, hail showers and thunderstorms occur the whole year, but frequency is highest during two periods: April-May and October-November. Precipitation is generally in the form of showers but continuous rain occurs also. The greatest intensity is in July.

714. A flight is to depart from an airport with runways 09 and 27. Surface wind is 270/05; an inversion is reported at 300 feet with turbulence and wind shear. The wind just above the inversion is 090/30. What is the safest departure procedure ? A. B. C. D.

Depart on runway 09 with a tailwind. Depart runway 27 with as steep an ascent as possible. Depart runway 27 with maximum throttle during the passage through the inversion. Take-off is not possible under these conditions.

715. The turbulence which occurs at high flight levels (above FL 250) is mainly of the type Clear Air Turbulence. In what way can moderate to severe Clear Air Turbulence affect an aircraft, the flight and the passengers? A. The turbulence can be resembled with the roughness of a washing-board (small scale) and will not have influence on the aircraft and its solidity, but will make flight a little more difficult. The passengers will seldom notice anything of this turbulence. B. The turbulence is wave like which makes the flight unpleasant for the passengers but the manoeuvring will not be affected essentially. C. The turbulence is a small scale one and can cause damage. The manoeuvring of the aircraft will be made more difficult or even impossible. For the passengers the flight will be unpleasant. D. The turbulence is a large scale one (waving) so that the aircraft will be difficult to manoeuvre. The passengers will feel some discomfort.

716. Which statement regarding aircraft and lightning is correct? A. Aircraft made by composite material can't conduct a lightning and will therefore very seldom be struck. B. An aircraft made by metal has a certain capacity to attract a lightning, but the lightning will follow the surface and therefore no damage will be caused. C. An aircraft has in the atmosphere the same qualities as a "Faradays cage", which means that struck of lightning seldom occurs. But if it happens, the result will be an occasional engine failure. The crew may get a shock. D. Aircraft made by composite material may get severe damage, the crew may be blinded and temporarily lose the hearing.

ATP Online - Meteorology

Page 98 of 213

717. Which is true of the temperature at the tropopause? A. B. C. D.

It is highest in mid-latitudes There is no significant difference with change of latitude It is higher in polar regions than in equatorial regions It is higher in equatorial regions than in polar regions

718. The value of the saturated adiabatic lapse rate is closest to that of the dry adiabatic lapse rate in A. B. C. D.

freezing fog stratus cumulus cirrus

719. For a similar pressure gradient, the geostrophic wind speed will be A. B. C. D.

equivalent to gradient wind ± thermal component greater at 30°N than at 60°N greater at 60°N than at 30°N the same at all latitudes north or south of 15°

720. For the same pressure gradient at 60°N, 50°N and 40°N the speed of the geostrophic wind will be A. B. C. D.

greatest at 60°N the same at all latitudes greatest at 40°N least at 50°N

721. Under anticyclone conditions in the northern hemisphere, with curved isobars the speed of the gradient wind is A. B. C. D.

proportional only to the Coriolis force less than the geostrophic wind greater than the geostrophic wind the same as the thermal component

722. In which of the following areas do surface high pressure systems usually predominate over the North Atlantic region between 30°N and 65°N and the adjoining land areas during the northern summer? A. B. C. D.

Azores, SE USA, SW Europe Iceland, SW USA, Azores Greenland, Azores, NE Canada Greenland, SW Europe, NE Canada

723. Assuming a generalised zonal system of world wind circulation, the SE trade winds are applicable to zone A. B. C. D.

u t w v

724. Assuming a generalised zonal system of world climatic and wind circulation, zone "t" is an area of A. B. C. D.

subtropical high pressure systems travelling low pressure systems NE trade winds SE trade winds

ATP Online - Meteorology

725. Considering Melbourne (C) in July, the weather is predominantly influenced by the zone of A. B. C. D.

subtropical high pressure, with the occasional passage of fronts originating in the adjacent zone of westerly waves equatorial low pressure due to the proximity of the intertropical convergence zone over central Australia disturbed temperate low pressure, bringing an almost continuous succession of fronts resulting in strong winds, low cloud and rain Antarctic high pressure due to the absence of any protective land mass between south Australia and Antarctica

726. Assuming a generalised zonal system of world climatic and wind circulation, zone "y" is an area of A. B. C. D.

NE trade winds subtropical high pressure systems SE trade winds travelling low pressure systems

Page 99 of 213

ATP Online - Meteorology

Page 100 of 213

727. Assuming a generalised zonal system of world wind circulation, the NE trade winds are applicable to zone A. B. C. D.

u w t v

728. Assuming a generalised zonal system of world climatic and wind circulation, zone "u" is in area of A. B. C. D.

subtropical high pressure SW trade winds travelling depressions NE trade winds

729. In the northern hemisphere the gradient wind of a cyclonic pressure distribution is 350/24, over the sea the surface wind would approximate A. B. C. D.

030/20 030/28 340/20 340/28

ATP Online - Meteorology

Page 101 of 213

730. The following statements deal with precipitation, turbulence and icing. Select the list containing the most likely alternatives for NS cloud: A. B. C. D.

Precipitation may be snow, sleet or rain. Icing and turbulence are frequently severe. Precipitation is frequently in the form of hail. Icing and turbulence are frequently severe. Precipitation and icing are usually nil. Turbulence is rarely more than moderate. Precipitation may be snow, sleet or rain. Icing is probable and may range between light and severe. Turbulence is rarely more than moderate.

731. Select the answer which you consider will complete correctly the following statement in relation to the main pressure systems affecting the North Atlantic region between 30°N and 65°N. During winter the predominant mean low pressure system at the surface A. B. C. D.

Azores Siberia USA Iceland / Greenland

732. The pressure system at position "D" is a A. B. C. D.

trough of low pressure col secondary low ridge of high pressure

733. Considering the North Atlantic region between 30°N and 65°N and the adjacent land areas during mid-summer, the predominant pressure systems are A. B. C. D.

Scandinavian high and Azores high weak low over NE Canada and Scandinavian high Azores high and weak low over NE Canada Azores low and Icelandic high

734. Considering the North Atlantic region between 30°N and 65°N together with the adjacent land areas during winter, the normal disposition of the main anticyclones at the surface is A. B. C. D.

Azores, Siberia NE Canada, Iceland Siberia, Iceland, Canaries Greenland, Iberian peninsula

735. The arrows labelled "r" represent the mean tracks of tropical revolving storms which occur mainly from A. B. C. D.

December to April and are called cyclones June to October and are called hurricanes June to October and are called typhoons December to April and are called tornadoes

ATP Online - Meteorology

736. Which is true of a secondary depression in the northern hemisphere? A. B. C. D.

It rapidly closes on, and merges with the primary It tends to move round the primary in an anticyclonic sense It tends to maintain its position relative to the primary It tends to move round the primary in a cyclonic sense

737. Why do tropical revolving storms tend to develop mostly in the western parts of the tropical oceans? A. B. C. D.

Because there is a maximum of humidity as a result of the trade winds long sea passage Because they are areas in which there is a strong progressive wind shear with increase of height Because the gulf formation of the coastlines triggers a strong rotary circulation Because there is a maximal temperature difference between land mass and sea

738. The arrows labelled "s" represent the mean tracks of tropical revolving storms which occur mainly from A. B. C. D.

December to April and are called cyclones December to April and are called typhoons May to November and are called hurricanes May to November and are called cyclones

Page 102 of 213

ATP Online - Meteorology

Page 103 of 213

739. The arrows labelled "t" represents the mean track of tropical revolving storms which occur mainly from A. B. C. D.

June to October and are called tornadoes June to October and are called cyclones December to April and are called hurricanes June to October and are called hurricanes

740. The arrows labelled "u" represent the tracks of tropical revolving storms which occur mainly from A. B. C. D.

May to July and are called cyclones December to April and are called tornadoes January to March and are called willy-willies July to October and are called typhoons

741. When, if at all, is a tropical revolving storm most likely to affect Darwin, on the central north coast of Australia? A. B. C. D.

Not experienced at Darwin December to April August to October May to July

742. Considering that portion of the route indicated from 30°E to 50°E, the upper winds in January above FL 300 are most likely to be

ATP Online - Meteorology A. B. C. D.

light easterlies variable in direction and less than 30 kt a westerly polar front jet stream, maximum speed exceeding 90 kt a subtropical westerly jet stream, maximum speed exceeding 90 kt

743. The dotted line designated "Z" represents the A. B. C. D.

northerly limit of the sub tropical jet stream during July; mean position of the intertropical front (ITCZ) during July; mean position of the intertropical front (ITCZ) during January; northerly limit of the SE trade winds during January;

744. The dotted line labelled "Y" represents the

Page 104 of 213

ATP Online - Meteorology A. B. C. D.

axis of the equatorial jet stream during July mean position of the intertropical convergence zone (ITCZ) during January axis of the subtropical jet stream during January mean position of the intertropical convergence zone (ITCZ) during July

745. What weather conditions are most likely to affect an approach to Dakar during July? A. B. C. D.

Reduced visibility due to the rising sand of the Harmattan Wet and thundery due to the proximity of the intertropical convergence zone (ITCZ) Generally clear skies - NW trade winds Dry and clear due to the influence of the Azores high pressure system

746. Weather conditions at Mumbai during early July are mainly influenced by the A. B. C. D.

NE monsoon and the proximity of the ITCZ passage of frontal system generated in the south Indian ocean high incidence of tropical revolving storms originating in the Persian gulf SW monsoon

Page 105 of 213

ATP Online - Meteorology

747. Weather conditions at Mumbai during January are mainly influenced by the A. B. C. D.

SW monsoon NE monsoon SE monsoon NW monsoon

748. The weather most likely to be experienced at position "R" is A. B. C. D.

early morning fog lifting to low stratus fine and warm at first - AC Castellanus and CB in late afternoon with thunderstorms overcast with drizzle and hill fog increasing amounts of AS and NS - heavy rain

Page 106 of 213

ATP Online - Meteorology

749. Which one of the following local winds is a Foehn wind? A. B. C. D.

Chinook Scirocco Harmattan Bora

750. Dew point is defined as A. B. C. D.

the lowest temperature at which evaporation will occur for a given pressure the temperature below which the change of state in a given volume of air will result in the absorption of latent heat the lowest temperature to which air must be cooled in order to reduce the relative humidity the temperature to which moist air must be cooled to become saturated at a given pressure

751. In meteorology the process by which water vapour is transformed directly into ice is known as A. B. C. D.

sublimation radiation cooling supercooling supersaturation

752. A trough near the surface is an area of A. B. C. D.

convergence and widespread ascent convergence and subsidence divergence and subsidence divergence and widespread ascent

753. Which is true of a typical thermal depression? A. B. C. D.

It forms over land in winter It forms over land in summer It forms over the ocean in winter It forms over the ocean in summer

754. A Foehn wind occurs on the A. windward side of a mountain range and is caused by surface heating B. windward side of a mountain range and is caused by surface cooling and reverse air flow C. leeward side of a mountain range and is caused by the condensation level being lower on the leeward side than on the windward side

Page 107 of 213

ATP Online - Meteorology

Page 108 of 213

D. leeward side of a mountain range and is caused by significant moisture loss by precipitation from cloud

755. Around Paris on January 3rd at 1800 UTC, the surface temperature, under shelter, is 3°C. The sky is covered by 8 oktas of stratus. QNH is 1033 hPa. If the sky is covered all night, the minimum temperature of the night of January 3rd to January 4th should be A. B. C. D.

slightly below +3°C. significantly below 0°C. slightly above +3°C. significantly above +3°C.

756. In summer, when wind is calm and sky is clear, the minimum temperature over land is reached approximately A. B. C. D.

at the moment the sun rises one hour before sunrise half an hour before sunrise half an hour after sunrise

757. An isohypse (contour) A. B. C. D.

is the limit between two air masses of different temperature indicates the altitude of the zero degree isotherm indicates the true altitude of a pressure level is the longest slope line of a frontal surface

758. Before landing, an altimeter set to QFE indicates A. B. C. D.

in standard atmosphere, the height of the aircraft above the official airport elevation. the flight level. the height of the aircraft's wheels above the runway. the aircraft's altitude above the mean sea level.

759. Relative humidity at a given temperature is the relation between A. B. C. D.

actual water vapour content and saturated water vapour content dew point and air temperature water vapour weight and dry air weight water vapour weight and humid air volume

760. From what type of cloud does drizzle fall ? A. B. C. D.

Stratus. Cirrostratus. Cumulus Altostratus

761. What type of clouds are associated with rain showers ? A. B. C. D.

Towering cumulus and cumulonimbus. Nimbostratus. Towering cumulus and altostratus. Altostratus and stratus.

762. What type of clouds are associated with snow showers ? A. B. C. D.

Cumulus and altostratus Nimbostratus Cumulus and cumulonimbus Altostratus and stratus

763. In the mid-latitudes the stratosphere extends on an average from A. B. C. D.

50 to 85 km 85 to more than 200 km 0 to 11 km 11 to 50 km

764. At the top of orographic waves, in mountainous regions, the cloud most likely to be encountered is A. B. C. D.

cumulus mediocris. altocumulus lenticularis. cirrus. cirrostratus.

765. The core of the polar front jet stream is usually located in the

ATP Online - Meteorology A. B. C. D.

Page 109 of 213

tropical air below the tropopause polar air below the tropopause polar air above the tropopause tropical air above the tropical tropopause

766. The most dangerous icing conditions are encountered in A. B. C. D.

unstable clouds at medium levels. zones where the air temperature is below -15°C. icy clouds at high levels. supercooled precipitation.

767. A cumulus congestus is A. B. C. D.

a cumulus that is of great vertical extent a cumulus that only occurs in association with the ITCZ a cumulus with little vertical development a remnant of a CB

768. A METAR message is valid A. B. C. D.

for the hour following the observation at the time of observation for 9 hours for 2 hours

769. A parcel of unsaturated air is forced to rise through an isothermal layer. So long as it remains unsaturated, the temperature of the parcel A. B. C. D.

remains constant decreases 0.65°C per 100 m becomes equal to the temperature of the isothermal layer decreases 1°C per 100 m

770. A wind of 20 knots corresponds to an approximate speed of A. B. C. D.

10 km/h 40 m/sec 10 m/sec 50 km/h

771. A wind sounding in the region of a polar front jet stream gives the following windprofile (Northern hemisphere).; 900hPa 220/20kt ; 800hPa 220/25kt ; 700hPa 230/35kt ; 500hPa 260/60kt ; 400hPa 280/85kt; 300hPa 300/100kt; 250hPa 310/120kt ; 200hPa 310/80kt; Which system is the jet stream associated with? A. B. C. D.

With a warm front. With an easterly wave. With a ITCZ. With a cold front.

772. Absolute instability in the atmosphere will occur when the environmental lapse rate is A. B. C. D.

greater than both saturated adiabatic lapse rate and dry adiabatic lapse rate less than both saturated adiabatic lapse rate and dry adiabatic lapse rate greater than saturated adiabatic lapse rate but less than dry adiabatic lapse rate less than saturated adiabatic lapse rate

773. Advection fog is most likely to form when A. B. C. D.

cold maritime air flows over a warmer surface and the wind speed is greater than 15 kt a mild moist airstream flows over snow covered ground and the wind speed is less than 10 kt cold air is forced over higher ground and further adiabatic cooling occurs warm maritime air flows over a relatively warmer surface and the wind speed is greater than 15 kt

774. After a prolonged VMC descent in very cold air, you penetrate a humid air mass. What type of icing will you encounter? A. B. C. D.

Rime ice Hoar frost Clear ice Smooth icing

775. After a sunny day, followed by a long clear night, you take-off from an airfield at mid-latitudes an hour before sunrise. The field is not situated close to the coast. Though the pressure gradient is rather large, the easterly surface wind is weak and makes A. a squally wind up to great heights B. little increase in wind speed and little veering of the wind up to an height of 5000 FT

ATP Online - Meteorology

Page 110 of 213

C. large but gradual increase in wind speed and large but gradual veering of the wind up to a height of 5000 FT D. a sudden strong increase in wind speed and strong veering of the wind a short time after take-off

776. Air masses which are being cooled from below are often characterized by: A. B. C. D.

fog, poor visibility and layered clouds. strong winds, cumulus clouds, good visibility. uniform temperature, good visibility. continuous rain and freezing temperatures.

777. An aircraft flies into freezing rain in an area with a temperature below 0°C. The type of icing it will most likely encounter is A. B. C. D.

granular frost hoar frost rime ice clear ice

778. An aircraft flying at FL 45 (OAT 6°C) obtains a reading of 1860 ft on its radio altimeter (ground elevation 3090 ft). What is the value of the QNH, to the nearest hPa, at that point? A. B. C. D.

996 1013 1030 1042

779. An aircraft maintains a constant indicated altitude of 4500 FT from A (360 FT/AMSL - QNH 986 hPa) to B (690 FT/AMSL - QNH 1011 hPa). Assuming that the altimeter sub-scale setting remains unchanged at 986 hPa, the actual height of the aircraft above the surface at B will be A. B. C. D.

4815 FT 4485 FT 5175 FT 3135 FT

780. An aircraft maintains a constant indicated altitude of 5500 FT from A (1050 FT/AMSL - QNH 968 hPa) to B (650 FT/AMSL - QNH 1016 hPa). Assuming that the altimeter subscale setting remains unchanged at 968 hPa, the actual height of the aircraft above the surface at B will be A. B. C. D.

6146 FT 4854 FT 7446 FT 6796 FT

781. An aircraft maintains a constant indicated altitude of 6500 FT from A (600 FT/AMSL - QNH 1012 hPa) to B (930 FT/AMSL - QNH 977 hPa). Assuming that the altimeter subscale setting remains unchanged at 1012 hPa, the actual height of the aircraft above the surface at B will be A. B. C. D.

5225 FT 4625 FT 5555 FT 6515 FT

782. An aircraft maintains a constant indicated altitude of 7500 FT from A (270 FT/AMSL - QNH 1021 hPa) to B (1650 FT/AMSL - QNH 983 hPa). Assuming that the altimeter sub-scale setting remains unchanged at 1021 hPa, the actual height of the aircraft above the surface at B will be A. B. C. D.

6876 FT 6204 FT 4824 FT 6474 FT

783. An occlusion has the characteristics of a warm front when: A. B. C. D.

the cold air behind is colder than the cold air ahead the cold air behind is lifted by the warm air the cold air behind is warmer than the cold air ahead the cold air ahead is lifted

784. An unstable air mass will normally be characterised by A. B. C. D.

stratiform cloud poor visibility due to haze at the lower levels continuous light rain from medium level layer cloud cumuliform cloud and good visibility except in precipitation

ATP Online - Meteorology

Page 111 of 213

785. Assuming that an initial "trigger" force is present, the conditions most likely to result in the formation of thunderstorms are A. B. C. D.

high relative humidity and instability throughout a deep layer intense surface heating, anticyclonic pressure system and relatively high freezing level rapid orographic cooling of cloud containing ice crystals high surface temperature, low dew point and high dry adiabatic lapse rate

786. At about what geographical latitude as average is assumed for the zone of prevailing westerlies? A. B. C. D.

50°N. 10°N. 30°N. 80°N.

787. At which time, if any, are polar front jet streams over the South Pacific usually strongest? A. B. C. D.

July January There is no annual variation October

788. Between mean sea level and a height of 20 km, the lowest temperature in the ICAO Standard Atmosphere (ISA) is: A. B. C. D.

-56.5°C -273°C -44.7°C -100°C

789. Clouds in patches, sheets or grey or whitish layers made up of elements resembling large pebbles or rollers, together or not, and always clear of the ground are: A. B. C. D.

Stratus. Altostratus. Nimbostratus. Stratocumulus.

790. Cold air pools A. B. C. D.

are most evident in the temperature and wind fields of the upper levels only occur at mid-latitudes only occur in winter can easily be recognized on synoptic surface charts

791. Considering the North Atlantic area north of 60°N during winter, the mean height of the tropopause is approximately A. B. C. D.

37 000 FT 56 000 FT 29 000 FT 20 000 FT

792. Considering the North Atlantic at latitude 50°N during winter, the mean height of the tropopause is approximately A. B. C. D.

43 000 FT 54 000 FT 31 000 FT 23 000 FT

793. Considering the North Atlantic between 30°N and 65°N, the mean position of the polar front during winter extends from A. B. C. D.

NE Canada to Portugal Newfoundland to Iceland Iceland to Norway Florida to SW England

794. Considering the North Atlantic region between 30°N and 65°N, the mean position of the polar front during summer extends from A. B. C. D.

NE Canada to Iceland Greenland to Spain Newfoundland to N Scotland Florida to SW England

795. Convective weather phenomena include A. thunderstorms, hail, tornadoes, wind gusts, heavy showers, lightning strikes B. hail, lightning strikes, wind lulls, squalls, stratocumulus, low level wind maximum C. thunderstorms, tornadoes, hail, haze, wind gusts, advection fog

ATP Online - Meteorology

Page 112 of 213

D. heavy showers, lightning strikes, hail, mist, squalls, light rain over a large area

796. During periods of undisturbed radiation weather, overland, the A. B. C. D.

angle between isobars and surface wind direction tends to be greatest in the mid afternoon surface wind speed tends to be highest during the mid afternoon wind tends to back from early morning until early afternoon surface wind speed tends to be highest at night

797. During summer an anticyclone covers the British Isles giving mainly clear skies. At 0600 UTC a south coast airfield in Southern England reports a surface wind of 350/06. The coastline at the airfield perimeter is aligned in an east/west direction. During the next twelve hours the surface wind is likely to A. B. C. D.

increase from the north by mid-morning becoming calm towards evening veer to easterly before becoming calm by the afternoon back to NW and strengthen by mid-afternoon become southerly to southwesterly and increase in velocity by afternoon

798. During summer, a weak pressure gradient covers a coastal airfield in Eastern England resulting in mainly clear skies. The surface wind at dawn was calm. If the alignment of the coastline in the vicinity of the airfield is predominantly north/south, the surface wind direction is likely to become A. B. C. D.

easterly and then westerly by afternoon westerly and then southerly by evening easterly to southeasterly and increase in velocitiy by afternoon westerly and increase in velocity by afternoon

799. During the approach to Mumbai ( 19°N - 73°E ) on the west coast of India, you are listening to the ATIS on 15 July at 0700 LT. Which of the following reports is most likely ? A. B. C. D.

30012KT 9999 SCT030 SCT200 20/16 Q1025 TEMPO 4000= 02005KT CAVOK 24/09 Q1030 NOSIG= 25014KT 4500 SHRA SCT015 BKN025CB 25/24 Q1006 NOSIG= 05013KT 3500 MIFG SCT003 BKN005 19/14 Q1012 BECMG 8000=

800. For a VFR aircraft, the conditions in which it could encounter severe airframe icing are: A. B. C. D.

flight into freezing rain, resulting in clear ice formation flight between two cloud layers, without precipitation, resulting in clear ice formation flight into supercooled rain, resulting in rime ice formation flight into an area outside of clouds where the temperature is below 0°C, resulting in rime ice formation

801. For an airfield located in the British Isles, the passage of a warm front will usually be indicated by A. B. C. D.

rapid improvement in visibility, pressure falling rapidly, wind veering and increasing rise in temperature, rapid rise in pressure, wind backs and becomes gusty rise in temperature, rise in dew point temperature, wind veers and decreases a fall in temperature, rise in dew point temperature, wind backing and decreasing

802. For both saturated and unsaturated air instability will occur when the A. B. C. D.

environmental lapse rate is greater than both dry adiabatic lapse rate and saturated adiabatic lapse rate environmental lapse rate is less than both dry adiabatic lapse rate and saturated adiabatic lapse rate environmental lapse rate is greater than saturated adiabatic lapse rate but less than dry adiabatic lapse rate dry adiabatic lapse rate is less than saturated adiabatic lapse rate but greater than environmental lapse rate

803. For the same horizontal distance between adjacent isobars the gradient wind speed will be least at: A. B. C. D.

50°N with a cyclonic circulation 30°N with a cyclonic circulation 50°N in an anticyclone 30°N in an anticyclone

804. From summer to winter the average geographic position of the polar front jet stream over the North Atlantic moves A. B. C. D.

towards the south and the speed increases towards the north and the speed increases towards the north and the speed decreases towards the south and the speed decreases

805. Hazardous hailstones, reaching the ground, are most likely to be experienced below CB clouds situated A. in continental interiors near the equator B. in continental interiors in middle latitudes C. over the sea in middle latitudes

ATP Online - Meteorology

Page 113 of 213

D. over the sea near the equator

806. Ice accretion to the airframe is likely to be most hazardous at temperatures A. B. C. D.

between 0°C and -17°C in ST between 0°C and -17°C in AS below -40°C in CB between 0°C and -23°C in large CU

807. If the QFE, QNH and QFF of an airport have the same value, A. B. C. D.

the 1013.25 hPa level must be at MSL the airport must be at MSL and the conditions must be as in the ISA the airport must be at MSL the conditions must be as in the ISA

808. In a METAR message, BR and HZ mean respectively: A. B. C. D.

BR = mist HZ = widespread dust BR = mist HZ = haze BR = fog HZ = haze BR = mist HZ = smoke

809. In a mountainous area, the most hazardous flying conditions associated with standing waves are likely to be experienced A. B. C. D.

between the troughs of the waves particularly 30 NM to 50 NM downwind and close to the tropopause on the windward side of the mountain just above the "cap" cloud in the vicinity of the "roll" cloud or rotor zone beneath the first wave on the leeward side just above the lenticular cloud on the windward side

810. In addition to a stable layer of air over a substantial mountain range, the conditions most favourable to the development of standing waves are A. B. C. D.

steep dry adiabatic lapse rate, wind speed increasing and changing direction rapidly with increase in height wind speed excess of 20 kt at the surface and increasing with height, wind direction perpendicular to the general direction of the range conditional instability, wind speed constant from a direction parallel to the mountain range significant moisture loss due to precipitation, rapid lowering of the tropopause in the area of the mountain range

811. In an intense trough of low pressure over Iceland during wintertime the weather likely to be experienced is: A. B. C. D.

light wind, good visibility and a high cloud ceiling strong wind with subsidence at low levels strong wind shear, convection and snow showers strong wind associated with an almost clear sky

812. In mid-latitudes, the tops of Cumulus are often limited by A. B. C. D.

a temperature inversion a layer of unstable air the tropopause a radiation inversion

813. In relation to the polar front jet stream, the greatest rate of wind shear is most likely to occur A. B. C. D.

well below the core close to the core on the polar side 5000 FT or more above the core on the tropical side of the core

814. In relation to the total weight of the atmosphere, the weight of the atmosphere between mean sea level and a height of 5500 m is A. B. C. D.

99% 50% 1% 25%

815. In summer in the northern hemisphere the maximum wind speeds associated with sub-tropical jet streams are usually located A. B. C. D.

above the tropopause at about 250 hPa below the tropopause at about 200 hPa below the tropopause at about 300 hPa above the tropopause at about 100 hPa

816. In the lower levels of the atmosphere when the environmental lapse rate is greater than saturated adiabatic lapse rate but less than dry

ATP Online - Meteorology

Page 114 of 213

adiabatic lapse rate - the air mass is described as being A. B. C. D.

stable conditionally unstable unstable absolutely unstable

817. In the weather pattern behind a cold front, the visibility outside precipitation is A. B. C. D.

low and the precipitation is showers. good and the precipitation is showers. good and the precipitation is steady rain. low and the precipitation is steady rain.

818. Isotachs are lines joining equal A. B. C. D.

wind speed lapse rates sea-level pressures horizontal wind speed gradients wind speeds

819. Maximum wind speeds associated with subtropical jet streams are usually located in the A. B. C. D.

tropical air below the tropopause tropical air above the tropopause polar air below the tropopause polar air above the tropopause

820. Mountain waves should be expected A. B. C. D.

on the downwind side of the mountain chain directly over the mountain chain on the upwind side of the mountain chain when instability is high

821. Of what does lenticular cloud provide evidence? A. B. C. D.

Mountain waves Stratospheric inversions Areas of high level clear air turbulence Jet streams

822. In a METAR message, the wind group is 23010MPS. This means: A. B. C. D.

Wind from 230° magnetic at 10 miles per hour Wind from 230° true at 10 miles per hour Wind from 230° true at 20 knots Wind from 230° magnetic at 20 knots

823. On a Significant Weather Chart, we see the following for the area where a VFR-flight will take place: BKN CU SC 100 / 023 If the flight is planned at FL 85, we can estimate that A. B. C. D.

we will fly just below the cloud base where the cloud cover is between 5 and 7 oktas. we will probably have to divert around some cumulus tops. we will fly above a solid cloud cover. we will be in solid clouds and consequently a VFR flight is not possible.

824. On the Significant Weather Chart, we see the following indications:; SCT CU100/060; The forecast charts for FL 050 and FL 100 both indicate a wind from 320°/15 kt. In flight at FL 055 over an area of plains (flat country) during the validity period of the Significant Weather Chart, we observe moderate turbulence. This turbulence is caused by A. B. C. D.

a wind shear. system waves. the instability of the air mass between the surface and the top of Cumulus clouds. the surface radiation.

825. On the West coast of India it can be said in general that the wind blows A. B. C. D.

for six month from the North East and for six month from the South West the whole year from the South East the whole year from the North East for six month from the North West and for six month from the South East

826. Over which areas can tropical revolving storms occur? A. Caribbean Sea, Gulf of Bengal, Indian Ocean east of Madagascar

ATP Online - Meteorology

Page 115 of 213

B. Caribbean Sea, Indian Ocean at 20°S, Pacific Ocean at 2°N C. Australia, Gulf of Bengal, Atlantic Ocean at 20°S D. India, Arabic Sea, Atlantic Ocean at 2°S

827. Refer to the following TAF message. LFxx 180800Z 180918 22020KT 6000 SCT015 SCT080 BECMG 1214 24025KT 2000 RA BKN009 OVC070= At 1400 UTC, the lowest cloud base will be A. B. C. D.

at 1500 feet AGL at 900 feet AGL between 900 and 1500 feet AGL between 900 and 1500 feet AMSL

828. Sea breezes are most likely to occur when A. B. C. D.

a strong pressure gradient, relatively high sea temperatures and overcast conditions persist slack pressure gradient and clear skies result in relatively high land temperatures a slack pressure gradient, relatively high sea temperatures and overcast conditions persist a strong pressure gradient, relatively high sea temperatures and clear skies at night exist

829. Stratus formed by turbulence will occur when A. B. C. D.

in the friction layer mixing occurs by turbulence and the mixing condensation level is situated below the top of the turbulent layer the wind speed is greater than 10 kt and the mixing condensation level is situated just above the turbulent layer absolute instability exists at low level the wind speed is less than 10 kt and the air is heated by the earth''s surface

830. Surface temperature inversions are frequently generated by A. B. C. D.

compression causing the release of latent heat in a layer of stratiform cloud terrestrial radiation on a calm clear night gusting winds increasing surface friction during the day with consequent mixing at the lower levels an unstable air mass causing convection currents and mixing of the atmosphere at lower levels

831. Surface-based temperature inversions are common during A. B. C. D.

cloudy days in summer when the ground is wet cloud-free days in summer when the ground is dry cloud-free nights in winter when the ground is dry cloudy days in winter when the ground is wet

832. The conditions most favourable to the formation of mountain waves are A. wind direction approximately at right angles to the mountain range - wind speed 30 kt and steadily increasing with height - an inversion just above the crest level with less stable air above and below B. wind direction parallel to the general alignment of the mountain range - wind speed increasing with height - intense surface heating C. wind direction parallel to the mountain range - wind speed increasing with height - extensive isothermal layer between mountain crests and the tropopause D. wind speed less than 15 kt and wind direction at right angles to mountains - intense radiation cooling at night particularly at the higher levels

833. The formation of clear ice on the leading edges of an aircraft is most likely to be caused by the A. B. C. D.

instantaneous freezing of large supercooled water droplets and snow crystals relatively slow freezing of small supercooled water droplets and ice crystals relatively slow freezing of large supercooled water droplets instantaneous freezing of small supercooled water droplets

834. The formation of morning fog before sunrise is possible if A. B. C. D.

the wind is strong the sky is overcast air temperature and dew point are equal or close to one another the turbulence in the lower layers is moderate

835. The Hurricane season is mainly from A. B. C. D.

April until July. October until January. July until November. January until April.

836. The ICAO Standard Atmosphere (ISA) assumes that temperature will reduce at the rate of A. B. C. D.

1,98°C per 1000 FT up to 36090 FT after which it remains constant to 65617 FT 2°C per 1000 FT up to 65617 FT after which it will remain constant to 104987 FT 2°C per 1000 FT up to 36090 FT and will then increase at 0,3°C per 1000 FT up to 65617 FT 1,98°C per 1000 FT up to 36090 FT and will then rise at 0,3°C per 1000 FT up to 65617 FT when it will remain constant

ATP Online - Meteorology

Page 116 of 213

837. The length, width and depth of a typical mid-latitude jet stream are respectively A. B. C. D.

1000 nautical miles, 150 nautical miles, 30000 feet 200 nautical miles, 5 nautical miles, 10000 feet 1000 nautical miles, 5000 to 8000 feet, 30000 feet 1000 nautical miles, 150 nautical miles, 10000 feet

838. The maximum vapour pressure over a flat supercooled water surface and the maximum vapour pressure over a flat ice surface of the same temperature are compared. The maximum vapour pressure is A. B. C. D.

equal over both surfaces the same over both surfaces if the air pressure is the same smaller over the water surface greater over the water surface

839. The most likely place to encounter clear air turbulence associated with a jet stream is A. B. C. D.

5000 feet or more above the core well below the core on the tropical side of the core close to the core on the side facing the polar air

840. The passage of a warm front can be associated with areas of fog. The types of fog just in advance and just after the passage are respectively A. B. C. D.

arctic smoke and frontal fog advection fog and radiation fog advection fog and steaming fog frontal fog and advection fog

841. The pressure system indicated in a vertical cross section by pressure surfaces at lower heights bulging upward and pressure surfaces at greater heights bulging downward, is a A. B. C. D.

warm low pressure area warm high pressure area cold high pressure area cold low pressure area

842. The lowest cloud type observed is Stratus fractus, and there is moderate continuous rain. The area in which you are at this moment is most likely situated A. B. C. D.

in the main body of a warm or cold front in a high pressure area in a warm sector well behind a cold front

843. The stability in a layer increases by advection of A. B. C. D.

dry air in the upper part moist air in the lower part cold air in the lower part warm air in the lower part

844. The TAF weather messages are A. B. C. D.

landing forecasts of the "trend" type hourly or semi-hourly weather observations special weather observations airport forecasts

845. The temperature at the surface is given as +15°C and at 4000 FT it is +9°C, the state of this layer is said to be: A. B. C. D.

absolutely unstable unstable conditionally unstable stable

846. The term PROB as used in a TAF message, indicates the probability in percentage, of phenomena described during a specific period. The numerical values immediately following the term PROB, are: A. B. C. D.

35 or 50 20 or 30 25 or 35 30 or 40

ATP Online - Meteorology

Page 117 of 213

847. The thickness of the troposphere varies with A. B. C. D.

latitude longitude the wind rotation of the earth

848. Thunderstorms are often preceded by: A. B. C. D.

Nimbostratus Altostratus Altocumulus Lenticularis Altocumulus Castellanus

849. Thunderstorms reach their greatest intensity during the A. B. C. D.

Mature stage. dissipating stage. period in which precipitation is not falling. cumulus stage.

850. What name is given to the low level wind system between the subtropical high pressure belt and the equatorial trough of low pressure (ITCZ) ? A. B. C. D.

Westerly winds. Monsoon. Doldrums. Trade winds.

851. What type of precipitation might occur at 1700 UTC? MKJP 160430Z 160606 36010KT 9999 FEW025 BECMG 1315 14020G34KT FEW015CB SCT025 PROB30 TEMPO 1720 6000 +SHRA SCT010 BKN015CB BECMG 2224 34010KT FEW025= A. B. C. D.

Light drizzle Heavy rain showers Continuous moderate rain Intermittent light rain

852. What intensity and type of airframe icing is most likely to occur when aircraft descend rapidly from FL 320 (temp. -45°C) to FL 60 in warm, moist clear air? A. B. C. D.

Moderate opaque rime Nil ice Light or moderate hoar frost Light opaque rime and light clear ice

853. What intensity and type of airframe icing is most likely to occur when flying at FL 170 in AS with the outside air temperature at -20°C? A. B. C. D.

Moderate - hoar frost Light - rime Moderate - clear Severe - clear

854. What is a downburst? A. B. C. D.

An extremely strong wind gust in a tropical revolving storm. A small low pressure system where the wind circulates with very high speeds. A concentrated downdraft with high speeds and a higher temperature than the surrounding air. A concentrated downdraft with high speeds and a lower temperature than the surrounding air.

855. What is a favourable synoptic situation for the development of a Scirocco? A. B. C. D.

Low pressure area in the western part of the mediterranean sea High pressure area over Italy Extension of the Azores high pressure area over the Alps High pressure area in the western part of the Mediterranean sea

856. What is signified if an occlusion is described as "cold"? A. B. C. D.

It derives from a polar depression The air ahead of the associated warm front is less cold than the air behind the associated cold front The air ahead of the associated warm front is colder than the air behind the associated cold front On meeting the warm front, the cold front moves up the warm frontal surface

857. What is the difference between radiation fog and advection fog ? A. Radiation fog is formed by surface cooling in a calm wind. Advection fog is formed by evaporation over the sea.

ATP Online - Meteorology

Page 118 of 213

B. Radiation fog forms due to surface cooling at night in a light wind. Advection fog forms when warm humid air flows over a cold surface. C. Radiation fog forms only on the ground, advection fog only on the sea. D. Radiation fog forms due to night cooling and advection fog due to daytime cooling.

858. What is the lowest possible cloudbase forecast for 2300 UTC?; EDDF 272200Z 280624 VRB05KT4000 BR SCT005 OVC013 BECMG 1314 9000 SHRA OVC015 PROB40 TEMPO 1416 VRB15G25KT 1600 TSRA OVC010CB BECMG 1618 26010KT BKN030 BECMG 2122 CAVOK A. B. C. D.

1500 feet 500 feet 3000 feet 5000 feet

859. What is the meaning of the expression "FEW"? A. B. C. D.

1 - 2 oktas. 8 oktas. 5 - 7 oktas. 3 - 4 oktas.

860. What surface wind is forecast for 2200 UTC ?; EDDF 272200Z 280624 VRB05KT4000 BR SCT005 OVC013 BECMG 1314 9000 SHRA OVC015 PROB40 TEMPO 1416 VRB15G25KT 1600 TSRA OVC010CB BECMG 1618 26010KT BKN030 BECMG 2122 CAVOK A. B. C. D.

Variable/05 kt Variable/15 to 25 kt Calm 260°/10 kt

861. What surface wind is forecast for ETA 1700 UTC at Kingston? MKJP 160430Z 160606 36010KT 9999 FEW025CB BECMG1315 14020G34KT FEW015CB SCT025 PROB30 TEMPO 1720 6000 +SHRA SCT010 BKN015CB BECMG 2301 34010KT FEW025= A. B. C. D.

140° / 27 kt 140° / 20 kt gusts 34 kt 340° / 10 kt 360° / 10 kt

862. What visibility is most likely to be experienced at 1400 UTC ?; EDDF 272200Z 280624 VRB05KT4000 BR SCT005 OVC013 BECMG 1314 9000 SHRA OVC015 PROB40 TEMPO 1416 VRB15G25KT 1600 TSRA OVC010CB BECMG 1618 26010KT BKN030 BECMG 2122 CAVOK A. B. C. D.

4000 metres 9000 metres 1000 metres 1600 metres

863. What weather conditions are indications of the summer monsoon in India? A. B. C. D.

Sandstorms. Stratus clouds and drizzle. Thunderstorms, showers of heavy rain. Fog.

864. When a front has to cross a chain of mountains, its activity A. B. C. D.

decreases when it reaches the mountains. is not disturbed by the mountains. ceases immediately. strengthens "upwind" of the mountains.

865. When and where is an easterly jet stream likely to be encountered ? A. B. C. D.

In summer from the Middle East extending over the southern part of the Mediterranean to southern Spain. Throughout the year to the south of the Azorian high. In summer from south-east Asia extending over southern India to central Africa. In winter along the Russian coast facing the Arctic ocean.

866. When compared to the geostrophic wind in the northern hemisphere, surface friction will cause the surface wind to A. B. C. D.

veer and decrease back and increase back and decrease veer and increase

867. When flying at 5000 feet in the northern hemisphere over plains (flat country) with an anticyclone on the left and a depression on the right, the wind will be

ATP Online - Meteorology A. B. C. D.

Page 119 of 213

a tail wind. a head wind. from the right. from the left.

868. When fog is reported, the visibility is below A. B. C. D.

1 km 0.8 km 1.5 km 3 km

869. When visibility is reduced by water droplets to less than 1000 metres it is classified as A. B. C. D.

haze mist fog dust fog

870. Which area of a polar front jet stream in the northern hemisphere has the highest probability of turbulence? A. B. C. D.

Looking downstream, the area to the left of the core. In the core of the jet stream. Above the core in the boundary between warm and cold air. Looking downstream, the area to the right of the core.

871. Which cloud type may indicate the presence of severe turbulence ? A. B. C. D.

Altocumulus lenticularis Stratocumulus Cirrocumulus Nimbostratus

872. Which is true of advection fog? A. B. C. D.

It develops slowly and clears fast It usually forms by night and clears by day It forms when unstable air is adiabatically cooled It can appear suddenly by day or by night

873. Which of the following phenomena is least likely to lead to the formation of a Cumulonimbus with thunderstorm ? A. B. C. D.

Orographic lift Ground radiation Convergence Convection

874. Which of the following sets of conditions are most favourable to the development of thunderstorms? A. B. C. D.

Environmental lapse rate less than dry adiabatic lapse rate with freezing level below the cloud base, high relative humidity and strong surface winds Extensive isothermal layer, ice particles and water droplets must exist just below the freezing level and orographic lifting Environmental lapse rate less than saturated adiabatic lapse rate with dew point below 0°C and considerable surface heating Environmental lapse rate greater than saturated adiabatic lapse rate through a great vertical extent, high relative humidity and an initial lifting process

875. Which of the following sets of conditions are most likely to lead to the formation of advection fog? A. B. C. D.

A mild moist airstream flowing over colder surfaces with a wind in excess of 30 kt Cold maritime air flowing over a warmer land surface at a speed greater than 15 kt A mild moist airstream flowing over colder surfaces with the wind speed less than 15 kt Clear skies at night over an inland marshy area

876. Which of the following statements concerning the variation in wind speed between summer and winter on the North Atlantic between FL 300 and FL 400 is most correct? A. The average westerly component is greater in the summer than in the winter. The latitude of the axis of greatest wind speed is also further north in summer than in winter B. The average westerly component remains the same but the altitude of greatest wind speed reduces in winter C. The average westerly component is greater in the winter than in the summer. The latitude of the axis of greatest seasonal wind speed is further south in winter than in summer D. The average westerly component remains the same but the axis of greatest speeds moves south in winter

877. Which of the following statements is correct? A. The Bergeron-Findeisen process is mainly based on the difference of maximum vapour pressure over water and over ice of the same temperature B. At mid-latitudes the coalescense process is usually the initial process in the formation of precipitation C. The principle of the Bergeron-Findeisen process is mainly based on the difference in size of the cloud elements

ATP Online - Meteorology D. In the tropics stratiform precipitation does not occur

878. Which of the following statements is true of the dew point of an air mass? A. B. C. D.

It can be used to estimate the air mass's relative humidity even if the air temperature is unknown It can be used together with the air pressure to estimate the air mass's relative humidity It can be higher than the temperature of the air mass It can only be equal to, or lower, than the temperature of the air mass

879. Which of the following statements is true with regard to mountain waves? A. B. C. D.

Mountain waves are not experienced beyond 100 miles downwind from initiating high ground, regardless of the height of the ground Flight with headwind toward high ground is likely to be more hazardous than flight with tailwind toward high ground The absence of cloud over high ground indicates the absence of mountain waves Flight with tailwind toward high ground is likely to be more hazardous than flight with headwind toward high ground

880. Which of the following types of cloud can extend over the low, medium and high cloud levels ? A. B. C. D.

CB ST CI AC

881. Which of the following types of clouds are evidence of unstable air conditions? A. B. C. D.

SC, NS. ST, CS. CI, SC. CU, CB.

882. Which of the following types of jet streams can be observed all year round? A. B. C. D.

Equatorial jet stream / polar front jet stream. Equatorial jet stream / arctic jet stream. Subtropical jet stream / polar front jet stream. Arctic jet stream / subtropical jet stream.

883. Which of the following phenomena can provide the initial lifting leading to air mass thunderstorms? A. B. C. D.

Advection of warm air over a cold sea Low level wind shear Mountain waves Advection of cold air over a warm sea

884. Which statement about hurricanes is correct? A. B. C. D.

Their eye can be well observed by weather satellites They intensify after landfall They move with a speed of at least 60 kt They move towards the equator

885. Which statement is correct concerning a mountain ridge where a marked mountain wave has been reported ? A. B. C. D.

There are always rotor clouds The atmosphere is unstable at the level of the mountain tops The axis of a rotor is horizontal and perpendicular to the mountains Ragged altocumulus lenticularis is an indication for the presence of moderate/severe turbulence at the level of these clouds

886. Which thunderstorms move forward the fastest? A. B. C. D.

Thermal thunderstorms. Orographic thunderstorms. Thunderstorms formed by lifting processes. Frontal thunderstorms.

887. Which type of fog can NOT be formed over water? A. B. C. D.

Radiation fog Arctic smoke Frontal fog Advection fog

888. Which weather chart gives information about icing ?

Page 120 of 213

ATP Online - Meteorology A. B. C. D.

Page 121 of 213

700 hPa chart Surface chart 500 hPa chart Significant weather chart

889. You are flying at FL 340 on the northern hemisphere. The wind is geostrophic and there is a cross wind from the right all the time. Your true altitude will A. B. C. D.

decrease decrease, only if the pressure at the surface is decreasing on your route increase, only if the temperature at your flight level is rising on your route increase

890. You are flying in the northern hemisphere at 2000 FT over a flat country area. An anticyclone is ahead of you and a depression is behind you. The wind affecting you, will be A. B. C. D.

from your left from behind from ahead from your right

891. Lenticular clouds in mountainous areas indicate: A. B. C. D.

turbulence. unstable air. an inversion. light variable winds.

892. Which of the following layers of fog above land is coded as MIFG? A. B. C. D.

A layer of 5 feet deep. A layer of 15 feet deep. A layer of 20 feet deep. A layer of 10 feet deep.

893. Which of the following statements about lightnings and lightning strikes is correct? A. B. C. D.

Compasses and electronics are always affected The aircraft is temporarily part of the lightning trajectory Spherical lightnings often penetrate into aircraft Lightning strikes always cause heavy damage

894. Which of the following situations favours the formation of heavy thunderstorms? A. B. C. D.

The passage of a warm front in the morning. A cold front on the leeward side of a mountain range. A cold front approaching a mountain range in the evening. A warm sector moving over a snow-covered ground.

895. Which of the following quantities remains unchanged if unsaturated air is lifted until it reaches the lifting condensation level? A. B. C. D.

Maximum mixing ratio Mixing ratio Difference between temperature and dewpoint temperature Maximum vapour pressure

896. Which of the following processes will increase the stability of an air mass? A. B. C. D.

Cooling by the underlying surface. Advection of colder air aloft. Addition of water vapour in the lower layer. Warming of the air mass from below.

897. Which of the following phenomena should be described as precipitation at the time they are observed? A. B. C. D.

SA DZ TS SQ

898. Which of the following phenomena should be described as precipitation at the time they are observed? A. B. C. D.

+SHSN VA MIFG BR

ATP Online - Meteorology

Page 122 of 213

899. With all other quantities being constant, the density of the atmosphere increases with increasing A. B. C. D.

relative humidity stability air pressure temperature

900. Which of the following phenomena has to be mentioned in a SIGMET? A. B. C. D.

Thick fog. Strong inversion. Heavy duststorm. Snow and ice on the runway.

901. With the development of a thunderstorm, at what stage will there be only updraughts of air? A. B. C. D.

Anvil stage Mature stage Dissipating stage Initial stage

902. Where, as a general rule, is the core of the polar front jet stream to be found? A. B. C. D.

Just below the cold-air tropopause. In the tropical air mass. In the polar air mass. Just above the warm-air tropopause.

903. Where is the projection of the polar front jet stream on the surface most likely to be found in relation to the cold and warm fronts of a depression? A. B. C. D.

300 to 450 NM behind the cold front and 50 to 200 NM ahead of the warm front Up to 100 NM either side of the cold front and up to 200 NM either side of the warm front 50 to 200 NM behind the cold front and 300 to 450 NM ahead of the warm front Up to 200 NM either side of the cold front and up to 200 NM either side of the warm front

904. Where does wind shear occur? A. B. C. D.

Wind shear occurs primarily at lower altitudes in the vicinity of mountain waves. Wind shear of any significance occurs only in connection with jetstreams. Wind shear occurs only when there is a strong temperature inversion, or when the jetstream is associated with a strong depression. At any level in the atmosphere if associated with either a change of wind direction and/or wind speed.

905. Where do the trade winds blow? A. B. C. D.

At the equator. Between the "horse latitudes" and the doldrums. Between the "horse latitudes" and the equatorial highs. Between the "horse latitudes" and the mid latitudes.

906. Where are easterly waves found? A. B. C. D.

Between subtropical high pressure cells and ITCZ. In the region of the subtropical jet stream. In the temperate latitudes. On the poleward side of the subtropical high pressure belt.

907. When severe mountain waves are present, where would the area of most severe turbulence be located? A. B. C. D.

On the windward side of the mountain range Just above the cap cloud In the rotor zone Just below the tropopause

908. Which of the following phenomena has to be mentioned in a SIGMET? A. B. C. D.

Snow and ice on the runway. Strong inversion. Thick fog. Volcanic ash.

909. Why do tropical revolving storms tend to develop mostly in the western parts of the tropical oceans? A. Because there is a maximum of humidity as a result of the trade winds long sea passage

ATP Online - Meteorology

Page 123 of 213

B. Because there is a maximal temperature difference between land mass and sea C. Because they are areas in which there is a strong progressive wind shear with increase of height D. Because the gulf formation of the coastlines triggers a strong rotary circulation

910. What type of weather can usually be expected in a polar maritime air mass over central Europe in the daytime during summer? A. B. C. D.

Drizzle and low stratus. Showers and good visibility. Continuous rain and poor visibility. Sky clear.

911. You are flying over the sea at FL 250 and measure an outside temperature of -50°C. The pressure at sea level is 1023 hPa. What is your approximate true altitude calculated using normal vertical change in temperature with increase in height? A. B. C. D.

23230 FT/AMSL. 23770 FT/AMSL. 26230 FT/AMSL. 26770 FT/AMSL.

912. You are flying at FL 200. Outside air temperature is -40°C, and the pressure at sea level is 1033 hPa. What is the true altitude? A. B. C. D.

21770 ft 18290 ft 19310 ft 20630 ft

913. You are flying at FL 160. Outside air temperature is -27°C, and the pressure at sea level is 1003 hPa. What is the true altitude? A. B. C. D.

15100 ft 16920 ft 16360 ft 15620 ft

914. TAF LSZH 250600Z 250716 00000KT 0100 FG VV001 BECMG 0810 0800 VV002 BECMG 1012 23005KT 2500 BR BKN005 TEMPO 1316 6000 SCT007 = Which of these statements best describes the weather that can be expected at 1200 UTC? A. B. C. D.

Visibility 6 kilometres, cloud base 500 feet, wind speed 5 knots Visibility 800 metres, fog, wind from 230°, cloud base 500 feet Visibility 800 metres, fog, vertical visibility 200 feet, calm Visibility 2,5 kilometres, mist, cloud base 500 feet, wind speed 5 knots

915. Which statement is correct for the southern hemisphere? A. B. C. D.

The wind veers at the passage of a cold front The jet streams are easterly If the wind veers with increasing height then warm air is advected In the friction layer the wind backs with increasing height

916. Which statement is correct for microbursts ? A. B. C. D.

Their downdraft is warmer than the surroundings The diameter of the affected area on the surface does not exceed 4 km They occur in the tropics only They only develop below convective clouds with heavy rain

917. Which statement is correct for an absolutely unstable atmosphere? A. B. C. D.

the dry adiabatic lapse rate is more than 1°C / 100 m visibility is good between showers clouds are mainly of the stratiform type the environmental lapse rate is less than 1°C / 100 m

918. Which statement is correct for a warm occlusion? A. B. C. D.

The warm front overtakes the cold front. Both fronts become fronts aloft. The warm front becomes a front aloft. The cold front becomes a front aloft.

919. Which statement does correspond to the definition of severe turbulence? A. B. C. D.

Aircraft may be out of control for short periods, occupants are forced violently against seat belts, loose objects are tossed about. Difficulty in walking, occupants feel strain against seat belts, loose objects move about. Aircraft will be damaged and an emergency landing will be absolutely necessary. Aircraft gets out of control and crashes.

ATP Online - Meteorology

Page 124 of 213

920. Which statement concerning the tropopause is correct? A. B. C. D.

The temperature at the tropopause is approximately -80°C over the Poles and approximately -40°C over the equator The layer just above the tropopause is absolutely stable In the ICAO standard atmosphere the tropopause lies lower over the Poles than over the equator Above the tropopause no clear air turbulence occurs

921. Which statement concerning the Sirocco is correct? A. B. C. D.

It blows from southerly directions and can carry dust and sand which may reach Europe. The carried dust and sand does not reach great heights. This is caused by the presence of a trade wind inversion. It is formed by a strong increase of air pressure over North Africa. It is a northeasterly wind over the western part of North Africa with much dust and sand.

922. What winds and air mass characteristics are mainly associated with the winter monsoon in the monsoon regions of the Indian subcontinent? A. B. C. D.

South-westerly winds carrying warm and humid air. North-easterly winds bringing dry and hazy air. South-easterly winds carrying warm and humid air. North-westerly winds bringing dry and hazy air.

923. The air mass in the warm sector of a polar front is: A. B. C. D.

equatorial air. arctic air. tropical air. polar air.

924. Which two air masses are most likely to govern weather in western Europe? A. B. C. D.

Maritime polar warm and continental tropical warm. Maritime tropical warm and continental polar cold. Continental tropical warm and continental polar cold. Maritime tropical warm and maritime polar cold.

925. Stratus formed by turbulence will occur when A. B. C. D.

the wind speed is greater than 10 kt and the condensation level is situated just above the turbulent layer absolute instability exists at low level the wind speed is less than 10 kt and the air is heated by the earth's surface in the friction layer mixing occurs by turbulence and the condensation level is situated below the top of the turbulent layer

926. Runway Visual Range (RVR) is A. B. C. D.

reported when visibility is less than 2000m usually better than visibility reported in AIRMET and METAR measured with ceilometers alongside the runway

927. Relative humidity A. B. C. D.

is higher in cool air than in warm air is higher in warm air than in cool air increases if the air is cooled whilst maintaining the vapour pressure constant decreases if the air is cooled whilst maintaining the vapour pressure constant

928. Radiation fog most frequently occurs in: A. B. C. D.

high pressure systems over sea. low pressure systems over sea. low pressure systems over land. high pressure systems over land.

929. Pressure altitude is obtained by: A. B. C. D.

setting the altimeter to QFF pressure. setting the altimeter to standard sea level pressure. setting the altimeter to a station pressure which has been corrected to sea level. correcting the altimeter for temperature deviation from ISA.

930. Over the Indian Ocean and the Bay of Bengal tropical cyclones are A. never observed.

ATP Online - Meteorology

Page 125 of 213

B. occasionally observed, in the average 12 per year. C. frequently observed, in the average 15 per year over the Indian Ocean, but never over the Bay of Bengal. D. rare, in the average one every two years.

931. On an upper wind and temperature chart: A. B. C. D.

the pressure is constant all over the chart. the temperature is constant all over the chart. the wind increases from south to north. the height is constant all over the chart.

932. The air masses that are observed most frequently over western Europe are: A. B. C. D.

polar air and equatorial air. polar air and tropical air. arctic air and tropical air. arctic air and polar air.

933. What is the lowest visibility forecast for approach into Geneva (ETA 1200 UTC)? TAF LSGG 020900Z 021019 18007KT 9999 SCT020 BKN100 BECMG 1114 8000 RA SCT010 OVC020 PROB30 TEMPO 1018 VRB10G25KT TSRA SCT005 BKN015CB= A. B. C. D.

8 NM 8 km 10 km 6 NM

934. What is the most likely forecast lowest cloud base that can be expected during an approach to Madrid into 2300 UTC? TAF LEMD 281200Z 281812 13005KT CAVOK TEMPO 1821 8000 SCT020 SCT030 BECMG 2123 21005KT 9999 SCT015 BKN080 PROB40 TEMPO 2306 6000 SCT008 A. B. C. D.

1500 feet 2000 feet 800 feet 3000 feet

935. What kind of fog is often observed in the coastal region of Newfoundland in spring time? A. B. C. D.

Frontal fog. Steaming fog. Advection fog. Radiation fog.

936. What is, approximately, the temperature at 20000 FT in the ICAO Standard Atmosphere? A. B. C. D.

-15°C -25°C -30°C -20°C

937. What is true for the water vapour distribution in the layer between the surface and the 500 hPa pressure surface in the tradewind belt? A. B. C. D.

The lower part is relatively moist and the upper part is relatively dry. The whole layer is relatively dry. The lower part is relatively dry and the upper part is relatively moist. The whole layer is relatively moist.

938. What is the relationship between visibility and RVR in homogeneous fog? A. B. C. D.

There is no specific relationship between the two. The visibility generally is greater than the RVR. The visibility generally is less than the RVR. The visibility generally is the same as the RVR.

939. What is the most probable temperature at the tropical tropopause? A. B. C. D.

-75°C -35°C -25°C -55°C

940. Which statement is correct? A. B. C. D.

Mist and haze consist of water droplets. Fog can be supercooled and can also contain ice crystals. Fog and haze do not occur in the tropics. Mist and haze only differ by different values of visibility.

ATP Online - Meteorology

Page 126 of 213

941. What is the lowest visibility forecast for Bangkok at ETA 1400 UTC ? TAF VTBD 271800Z 280024 VRB05KT 1200 BR TEMPO 0002 4000 BECMG 0205 9999 SCT015 BKN100 TEMPO 1118 04010G20KT 5000 TSRA SCT009 BKN014 BKN018CB BECMG 2024 6000= A. B. C. D.

6 km 5 km 1200 metres 10 km or more

942. Which thunderstorms generally produce the most severe conditions, such as heavy hail and destructive winds? A. B. C. D.

Squall line thunderstorms. Warm front thunderstorms. Daytime air mass thunderstorms. Nocturnal air mass thunderstorms.

943. What is the lowest probable cloud base forecast for ETA 1700 UTC at Kingston ? TAF MKJP 160000Z 160606 36010KT 9999 FEW025 BECMG 1315 14020G34KT FEW015CB SCT025 PROB30 TEMPO 1520 6000 +SHRA SCT010 BKN015CB BECMG 2301 34010KT FEW025= A. B. C. D.

1500 FT 1000 FT 1000 m 1500 m

944. What is the lowest cloud base forecast for arrival at Geneva (ETA 1200 UTC)? TAF LSGG 020900Z 021019 18007KT 9999 SCT020 BKN100 BECMG 1114 8000 RA SCT010 OVC020 PROB30 TEMPO 1018 VRB10G25KT TSRA SCT005 BKN015CB= A. B. C. D.

1000 m 1000 FT 500 FT 500 m

945. What is the height of the lowest cloud forecast for Bangkok at ETA 1400 UTC ? TAF VTBD 271800Z 280024 VRB05KT 1200 BR TEMPO 0002 4000 BECMG 0205 9999 SCT015 BKN100 TEMPO 1118 04010G20KT 5000 TSRA SCT009 BKN014 BKN018CB BECMG 2024 6000= A. B. C. D.

1200 feet 1800 feet 1400 feet 900 feet

946. What is the dry adiabatic lapse rate ? A. B. C. D.

1.5°C/1000 FT 3.0°C/1000 FT 3.5°C/1000 FT 2.0°C/1000 FT

947. What is the best approximation for the wind speed at flight level 250? A. By simple interpolation of wind information available from the 500 and 300 hPa charts. B. By reading wind direction and speed from the 500 hPa chart. C. By interpolation of the wind information available from the 500 and 300 hPa charts, while also considering the maximum wind information found on the Significant Weather Chart. D. By reading wind direction and speed from the 300 hPa chart.

948. Why can the following METAR not be abbreviated to CAVOK? DLLO 121550Z 31018G30KT 9999 FEW060TCU BKN070 14/08 Q1016 TEMPO 4000 TS= (Aerodrome elevation 1000 ft, MSA for sector 000-190° 5800 ft, for sector 190-360° 7300 ft.) A. B. C. D.

Because towering cumulus are observed. Because the cloud base is below the highest minimum sector altitude. Because thunderstorms are forecast temporary for the next 2 hours. Because there are gusts reported.

949. Which type of air mass never occurs over central Europe ? A. B. C. D.

Tropical air Arctic air Polar air Equatorial air

950. What visibility is most likely to be experienced during an approach into Madrid at 2300 UTC ? TAF LEMD 281200Z 281812 13005KT CAVOK TEMPO 1821 8000 SCT020 SCT030 BECMG 2123 21005KT 9999 SCT015 BKN080 PROB40 TEMPO 2306 6000 SCT008 A. Greater than 10 kilometres

ATP Online - Meteorology

Page 127 of 213

B. 8000 metres C. 6000 metres D. 10 kilometres or more

951. What is the lowest visibility that may be expected during an approach into Dhahran at ETA 0600 UTC ? TAF OEDR 280000Z 280110 VRB08KT CAVOK BECMG 0103 7000 TEMPO 0410 28014G24KT 4000 SA= A. B. C. D.

4000 yards 4 km 7 km 10 km or more

952. In which zone of a polar front jet stream is the strongest CAT to be expected ? A. B. C. D.

Exactly in the centre of the core. About 12000 FT above the core. On the polar air side of the core. On the tropical air side of the core.

953. The weather most likely to be experienced at position "B" is A. B. C. D.

advection fog and drizzle frequent showers of rain or snow poor visibility in anticyclonic circulation early morning fog lifting to low stratus later

954. During summer, the cloud type most applicable to square 2A is A. B. C. D.

ST AC CS CB

ATP Online - Meteorology

955. In which squares are conditions most likely to cause the occurrence of low level wind shear? A. B. C. D.

3 B and 3 C 3 B and 3 D 3 A and 3 B 3 A and 3 C

956. The dotted line designated "Z" represents the A. B. C. D.

mean position of the intertropical convergence zone (ITCZ) during July northerly limit of the SE trade winds during January northerly limit of the sub tropical jet stream during July mean position of the intertropical convergence zone (ITCZ) during January

Page 128 of 213

ATP Online - Meteorology

957. Which is true of Nairobi (Kenya)? A. B. C. D.

There are two wet seasons There is one wet season In July it is frequently affected by clear skies and northerly winds from North Africa Apart from the wet season(s) it is cloud and rain free

958. The cloud type most applicable to square 2B is A. B. C. D.

CS CB SC ST

959. The cloud type most applicable to square 2C is A. B. C. D.

CS CU AS CB

Page 129 of 213

ATP Online - Meteorology

960. The cloud type most applicable to square 1E is A. B. C. D.

SC NS CS CB

961. The front at the bottom of the diagram, south of position C, is A. B. C. D.

a cold front an occlusion above the surface a warm front an occlusion on the surface

Page 130 of 213

ATP Online - Meteorology

962. The weather most likely to be experienced at position "B" is A. B. C. D.

clear skies, moderate wind, good visibility scattered stratocumulus with good visibility mainly overcast with stratus or stratocumulus, drizzle frequent showers of rain or snow, good visibility outside showers

Page 131 of 213

ATP Online - Meteorology

963. The air mass at position "X" is most likely to be A. B. C. D.

Continental tropical. Continental polar. Maritime polar. Maritime tropical.

Page 132 of 213

ATP Online - Meteorology

964. The weather most likely to be experienced near to position "A" is A. B. C. D.

clear skies - radiation fog at night frequent showers of rain and snow, good visibility outside precipitation advection fog and drizzle overcast layer cloud - rain later

Page 133 of 213

ATP Online - Meteorology

Page 134 of 213

965. For an aircraft making an approach to an airfield located in square 3B, away from the vicinity of the fronts, the most likely weather conditions in winter are: A. B. C. D.

prolonged periods of heavy rain and hail scattered SC and CU, good visibility generally overcast, moderate continuous rain and risk of low level wind shear poor visibility in mist and drizzle

966. For an aircraft making an approach to an airfield located in square 3B, the most likely weather conditions are A. B. C. D.

moderate continuous rain scattered AC base 2000 FT, good visibility low cloud, mist showers of rain and hail

ATP Online - Meteorology

967. What flight conditions are most likely to be experienced in square 2B by an aircraft at FL 120? A. B. C. D.

IMC in NS with risk of light icing IMC in layers of AS and isolated CB risk of severe turbulence and icing VMC below an overcast of AS and CS, generally smooth air VMC above layers of ST and SC, generally stable conditions

968. The cloud type most applicable to most of square 3B is A. B. C. D.

AS CS SC NS

969. The weather most likely to be experienced at position "S" is A. B. C. D.

small amounts of CU with generally good visibility scattered thunderstorms and good visibility increasing AS and NS with moderate to heavy rain overcast with drizzle and orographic fog

Page 135 of 213

ATP Online - Meteorology

970. The weather most likely to be experienced on the coast near position "Q" is A. B. C. D.

mainly overcast with poor visibility mainly clear skies with fog developing overnight showery with generally good visibility thundery showers particularly at night

971. What is the average temperature difference from ISA at FL 300 between Edinburgh and Madrid? A. B. C. D.

-2°C -12°C +12°C +2°C

Page 136 of 213

ATP Online - Meteorology

972. What is the average wind forecast for FL 300 between Edinburgh and Madrid? A. B. C. D.

240/25 280/30 180/20 300/45

Page 137 of 213

ATP Online - Meteorology

973. For 1300 UTC select a METAR which you consider to be most appropriate to position "T". A. B. C. D.

19010KT 6000 RA BKN016 OVC090 08/06 Q1004= 27030KT 8000 SCT020 07/03 Q1004= 18015KT 9999 SCT020 03/01 Q1000= 24020KT 5000 RA BKN100 11/10 Q1002=

Page 138 of 213

ATP Online - Meteorology

974. Considering the route between Valencia and Charleston at FL 340, the forecast mean temperature is A. B. C. D.

-55°C -50°C -40°C -45°C

Page 139 of 213

ATP Online - Meteorology

975. The front labelled "E" is a A. B. C. D.

cold occlusion cold front warm front warm occlusion

Page 140 of 213

ATP Online - Meteorology

976. The front located from 10°W to 10°E is most likely to be A. B. C. D.

an active warm front moving north an active occlusion moving south a cold front moving south a quasi-stationary front

977. At 40°N 20°W, the forecast wind at FL 390 is A. B. C. D.

070°/30 kt 270°/45 kt 090°/45 kt 060°/45 kt

Page 141 of 213

ATP Online - Meteorology

Page 142 of 213

978. For an aircraft making an approach to an airfield which is not situated near high ground, in the vicinity of the active front in square 3 B, a potential hazard exists in the form of A. B. C. D.

rotor streaming clear air turbulence (CAT) low level wind shear radiation fog

ATP Online - Meteorology

979. The temperature deviation from ISA (to the nearest °C) overhead Charleston at FL 340 is A. B. C. D.

-5 -7 +3 +5

980. The occurrence of freezing rain at FL 60 is most likely in square A. B. C. D.

2D 2A 3B 3C

Page 143 of 213

ATP Online - Meteorology

981. From indications shown on the chart, when front "S" passes position "V" the surface wind should A. B. C. D.

back and decrease back and remain more or less at the same speed veer and increase veer and remain more or less at the same speed

Page 144 of 213

ATP Online - Meteorology

982. The typical weather conditions affecting the Darwin area during July are A. B. C. D.

NW monsoon - very wet - proximity of the intertropical convergence zone causes widespread thunderstorm activity NE monsoon - continuous heavy rain but little thunderstorm activity dry season - mainly SE winds - visibility reduced by dust and haze mainly clear skies - advection fog drifting inland with the typical NE wind

983. When front "G" passes position "T" the surface wind should A. B. C. D.

back and increase back and decrease veer and decrease veer and increase

Page 145 of 213

ATP Online - Meteorology

Page 146 of 213

984. What conditions are most likely to prevail at an aerodrome located in square 3B? A. B. C. D.

6 - 8 oktas SC and ST, visibility moderate to poor in drizzle Broken CU base 2000 FT, visibility more than 5 km, occasional showers of rain or snow Mainly overcast at 8000 FT, visibility less than 5 km in continuous moderate rain Intermittent thunderstorms otherwise generally clear skies with good visibility

985. For an aircraft at FL 40, approaching the front (square 3C) from the direction of square 3D a potential hazard exists before reaching the front in the form of A. B. C. D.

severe downdrafts from subsiding air severe turbulence and wind shear clear ice accretion to the airframe severe turbulence associated with CB

ATP Online - Meteorology

986. For an aircraft at FL 80, ahead of the front in square 2D, the expected flight conditions are A. B. C. D.

high CI and CS type cloud, light turbulence and poor visibility IMC in cumuliform cloud, moderate turbulence with a risk of rime icing below AS type cloud, generally smooth air with light precipitation overcast skies, moderate to heavy turbulence with the possibility of thunderstorms

987. The air mass type indicated by arrow number 4 is designated A. B. C. D.

Maritime tropical. Continental polar. Continental tropical. Maritime polar.

988. At 40°N 70°W, the forecast wind is A. B. C. D.

240°/90 kt 280°/70 kt 270°/95 kt 320°/40 kt

Page 147 of 213

ATP Online - Meteorology

989. The weather most likely to be experienced at position A is A. B. C. D.

clear skies, good visibility in light winds cumulus, cumulonimbus clouds, heavy rain or snow showers, medium to strong winds radiation fog, low stratus, drizzle, no medium or upper cloud, light wind mainly overcast with stratus or stratocumulus and drizzle, medium to strong winds

990. What does the code TAF AMD mean? A. B. C. D.

No change compared with the previous TAF. Delayed issue of a TAF. Revised TAF. Weather conditions expected to affect the safety of normal operations.

991. What does the code R24R/P1500 mean? A. Snow clearance in progress on RWY 24 right, useable runway length 1500 metres.

Page 148 of 213

ATP Online - Meteorology

Page 149 of 213

B. RVR RWY 24 right 1500 metres. C. RVR RWY 24 right below 1500 metres. D. RVR RWY 24 right above 1500 metres.

992. What conditions are necessary for vertical visibility to be reported? A. B. C. D.

It is always reported. Whenever the lowest ceiling is below 1500 FT above ground. Whenever the lowest cloud base is below 1500 FT above ground. Whenever the sky is obscured by fog or heavy precipitation and the height of the cloud base cannot be measured.

993. What are the characteristics of rime ice, and what conditions are most favourable for its formation? A. Transparent appearance and tendency to take the shape of the surface on which it freezes. Stratiform clouds and temperatures only slightly below freezing promote its formation. B. Milky granular appearance, forming on leading edges and accumulating forward into the air stream. Stratiform clouds at temperatures of -10°C to -20°C are most conducive to its formation. C. Opaque rough appearance, tending to spread back over an aircraft surface. Most frequently encountered in cumuliform clouds at temperatures slightly below freezing. D. Smooth appearance and builds forward from leading surfaces into a sharp edge. Most common in cumuliform clouds at temperatures of -20°C to -25°C.

994. Tropical revolving storms are NOT formed in: A. B. C. D.

Gulf of Bengal. waters around the Philippines. South Indian Ocean. South Atlantic Ocean.

995. What intensity and type of aircraft icing is likely to occur at FL 100 in a thunderstorm, with the freezing level at 7000 FT? A. B. C. D.

Light icing due to clear ice. Moderate to severe icing due to clear ice. Light icing due to rime ice. Moderate to severe icing due to rime ice.

996. The widest precipitation zone occurs usually: A. B. C. D.

in rear of a cold front. ahead of a cold front. ahead of a warm front. in rear of a warm front.

997. What is a characteristic of stable air? A. B. C. D.

Temperature decreases rapidly. Stratiform clouds. Fair weather cumulus clouds. Unlimited visibility.

998. The term CAVOK is used when weather conditions are: A. B. C. D.

9000, SKC, NOSIG 8000, HAZARDOUS WX NIL, NOSIG 9999, NSC, NOSIG 9999, NSC, NSW

999. The temperature lapse rate of the ICAO standard atmosphere in the troposphere is: A. B. C. D.

6,5°C/1000 FT 3°C/1000 FT 2,5°C/1000 FT 2°C/1000 FT

1000. The SW monsoon starts in the month of A. B. C. D.

June in southern India to reach Pakistan in July. March in southern India to reach Pakistan end of April. December in southern India to reach Pakistan in May. September in Pakistan to reach southern India in November.

1001. The slope and speed of a warm front compared to the slope and speed of a cold front is in general: A. B. C. D.

greater and faster. smaller and slower. greater and slower. smaller and faster.

ATP Online - Meteorology

1002. A cumulonimbus cloud at mid-latitudes in summer contains A. B. C. D.

only ice crystals only water droplets ice crystals, water droplets and supercooled water droplets ice crystals and water droplets but never supercooled water droplets

1003. The radiosonde can directly measure: A. B. C. D.

air temperature, humidity, wind. atmospheric pressure, air temperature, humidity. humidity, wind, atmospheric pressure. wind, atmospheric pressure, air temperature.

1004. The presence of ice pellets at the surface is the evidence that: A. B. C. D.

after take-offyou can climb to a higher altitude without encountering more than light icing conditions. temperatures are above freezing at some higher altitudes. there are thunderstorms in the area. a cold front has passed.

1005. The air mass affecting position "A" is most likely to be A. B. C. D.

Maritime polar. Maritime tropical. Continental tropical. Continental polar.

1006. The cloud type most applicable to square 3C is A. B. C. D.

NS AS AC CB

Page 150 of 213

ATP Online - Meteorology

1007. The cloud type most applicable to square 2D is A. B. C. D.

AS CU CS CB

Page 151 of 213

ATP Online - Meteorology

1008. The cloud most likely to be experienced in square 1E is A. B. C. D.

CI AC AS CB

Page 152 of 213

ATP Online - Meteorology

1009. The weather most likely to be experienced at position "S" is A. B. C. D.

OVC SC little or no cloud - hazy frequent heavy showers of rain or snow morning fog lifting to low stratus

Page 153 of 213

ATP Online - Meteorology

1010. The air mass affecting position "R" is most likely to be A. B. C. D.

Continental tropical. Continental polar. Maritime polar. Maritime tropical.

Page 154 of 213

ATP Online - Meteorology

Page 155 of 213

1011. The air mass affecting position "P" is most likely to be A. B. C. D.

Maritime tropical. Maritime polar. Continental polar. Continental tropical.

1012. What characteristics will the surface winds have in an area where the isobars on the weather map are very close together? A. B. C. D.

Strong and flowing parallel to the isobars. Very weak but gusty and flowing somewhat across the isobars. Moderate and flowing parallel to the isobars. Strong and flowing somewhat across the isobars.

1013. The air mass affecting position "C" is most likely to be A. B. C. D.

Continental polar. Continental tropical. Maritime tropical. Maritime polar.

ATP Online - Meteorology

Page 156 of 213

1014. Flight from Lisbon (LPPT) to Kingston (MKJP) At 40°N 20°W, the temperature deviation from ISA at FL 390 is A. B. C. D.

+2°C 0°C +6°C -2°C

ATP Online - Meteorology

Page 157 of 213

1015. Considering the sector from 10°N to Nairobi of the route indicated, during January the upper winds at the 300 hPa level are most likely to be A. B. C. D.

westerlies at 40 kt easterly jet streams in excess of 70 kt westerlies in excess of 60 kt light easterlies

1016. Considering the sector Rome to 25°N on the route indicated, what mean upper winds may be expected at FL 300 during winter? A. B. C. D.

Variable easterlies Westerly jet streams in excess of 70 kt Light westerlies Easterlies at 40 kt

ATP Online - Meteorology

Page 158 of 213

1017. Assuming a generalised zonal system of world climatic and wind circulation, zone "x" is an area of A. B. C. D.

NE trade winds subtropical high pressure systems travelling low pressure systems the intertropical convergence zone (ITCZ)

1018. Considering the route indicated from Recife to Dakar the mean height of the tropopause during January is approximately A. B. C. D.

36 000 FT 56 000 FT 29 000 FT 43 000 FT

1019. Considering the North Atlantic route from the Azores to Bermuda, the mean height of the tropical tropopause during summer is approximately A. B. C. D.

39 000 FT 60 000 FT 33 000 FT 51 000 FT

ATP Online - Meteorology

Page 159 of 213

1020. What is necessary for the development of a polar front jet stream? A. B. C. D.

An unstable atmosphere up to great heights. Strong vertical temperature gradients. A uniform pressure pattern. Strong horizontal temperature gradients.

1021. What is a characteristic phenomenon for a stable atmosphere? A. B. C. D.

Stratified clouds. No precipitation. Weak wind. Many vertical currents.

1022. The air mass affecting position "S" is most likely to be A. B. C. D.

Maritime tropical and stable. Maritime polar and stable. Continental tropical and unstable. Maritime polar and unstable.

1023. If radiation fog forms on a clear night with light winds, the increase in wind speed from 5 kt to 13 kt will most likely A. B. C. D.

change the radiation fog to advection fog. have no effect. disperse the fog immediately. cause the fog to lift and become low stratus.

1024. The average forecast wind for the leg from Madrid to Dhahran at FL 390 is A. B. C. D.

270/30 310/50 270/50 320/70

ATP Online - Meteorology

Page 160 of 213

1025. At which pressure and temperature conditions may you safely assume that the minimum usable flight level at least lies at the same height, as the minimum safe altitude? A. B. C. D.

In a cold low pressure region At a temperature less than or equal to that of the ISA and where the QNH is less than 1013.25 hPa At a temperature greater than or equal to that of the ISA and where the QNH is greater than or equal to 1013.25 hPa In a very cold area with a QNH of 1015 hPa

1026. Between which latitudes are you most likely to find the region of travelling low pressure systems during summer? A. B. C. D.

25° - 45° 45° - 70° 15° - 25° 10° - 15°

1027. Clouds will mainly consist of supercooled water droplets when the temperature is: A. B. C. D.

below -40° C. between -5° C and -30° C. between -30° C and -40° C. between 0° C and -15° C.

1028. During the passage of a front in the northern hemisphere the wind veers. This statement is: A. B. C. D.

only true for the passage of a warm front. only true for the passage of a cold front. not true. true.

1029. Frontal fog can be formed by A. B. C. D.

condensation of air saturated by adiabatic cooling cooling at night evaporation of moisture at the surface condensation of air saturated by evaporation of precipitation

1030. Given: Altimeter setting: 1013.2 hPa Altimeter reading: 5000 ft Outside air temperature at 5000 ft: +5°C QFE: 958 hPa QNH: 983 hPa What is the true height of the aeroplane above the aerodrome?

ATP Online - Meteorology A. B. C. D.

Page 161 of 213

4325 FT 4865 FT 4190 FT 3515 FT

1031. At the surface the lifetime of a typical microburst and the diameter of the area affected by damaging winds are in the order of A. B. C. D.

5-10 minutes and 12 km 30-40 minutes and 4 km 1-5 minutes and 4 km 5-15 minutes and 8 km

1032. How is a ceiling defined? A. B. C. D.

Height above ground or water of the highest layer of cloud or obscuring phenomena aloft that covers 4 oktas or more of the sky. Height above ground or water of the lowest layer of cloud below 20000 ft covering more than half of the sky. Height above ground or water of the lowest layer of cloud independent on the amount of clouds. Height above ground or water of the lowest layer of cloud that contributes to the overall overcast.

1033. At FL 180, the air temperature is -35°C.The air density at this level is: A. B. C. D.

Equal to the density of the ISA at FL 180. Greater than the density of the ISA at FL 180. Unable to be determined without knowing the QNH. Less than the density of the ISA at FL 180.

1034. In an unsaturated layer in the friction layer the air is well mixed by turbulence. The layer remains unsaturated. After some time the vertical temperature profile will A. B. C. D.

coincide with the average wet adiabat in the layer. at first coincide with the average dry adiabat and then coincide with the average wet adiabat. correspond to the dry adiabatic lapse rate. always change toward a more stable position.

1035. In mature Cb's the probability of severe icing, according to meteorological rules, is greatest in the following temperature range: A. B. C. D.

-23°C to -40°C +5°C to 0°C 0°C to -23°C -40°C to -60°C

1036. In the METAR code the abbreviation VC indicates: A. B. C. D.

present weather within a range of 8 km, but not at the airport. present weather at the airport. volcanic ash. present weather within the approach area.

1037. In the METAR code the abbreviation VCBLDU means: A. B. C. D.

an active duststorm. an active sandstorm. blowing dust in the vicinity. an active dust- and sandstorm.

1038. In the mid-latitudes of the northern hemisphere the wind blows: A. B. C. D.

clockwise around anticyclones and cyclones. direct from high to low pressure areas. clockwise around anticyclones and anti-clockwise around cyclones. clockwise around cyclones and anti-clockwise around anticyclones.

1039. In the northern hemisphere advection of warm air aloft indicates: A. B. C. D.

the approach of a warm occlusion. increasing probability for showers. the formation of advection fog. backing winds with increasing heights.

1040. In the northern hemisphere with an anticyclonic pressure system the geostrophic wind at 2000 FT over the sea is 060/15. At the same position the surface wind is most likely to be A. B. C. D.

070/20 050/10 060/10 060/20

ATP Online - Meteorology

Page 162 of 213

1041. Good visibility in the lower levels may be expected when A. B. C. D.

warm air is moving over a cold surface. the atmospheric pressure is low. cool dry air is moving over a warmer surface. the air temperature is low.

1042. An air mass is: A. B. C. D.

an extensive body of air within which the temperature and humidity in horizontal planes are practically uniform. a large body of air with temperature and humidity constant in the vertical. a large body of air within which the temperature and humidity is uniform in horizontal and vertical planes. a body of air with a volume of not more than thousand cubic kilometres.

1043. A layer of stratus is most likely to be dispersed by A. B. C. D.

absorption of longwaved solar radiation in the stratus layer adiabatic cooling due to subsidence insolation resulting in the lifting of the condensation level the release of latent heat due to precipitation

1044. A stationary front is a front in which: A. B. C. D.

there are never frontal clouds. there is no difference in temperature between the two air masses. there is no wind on both sides of the front. there is no horizontal motion perpendicular to the front.

1045. A stationary observer in the northern hemisphere is situated in front of a depression. The centre of the depression passes from west to east and south of the observer. For this observer the wind A. B. C. D.

backs veers initially backs, then veers initially veers, then backs

1046. Absolute instability is said to exist whenever the environmental lapse rate A. B. C. D.

is less than the saturated adiabatic lapse rate is between the dry and saturated adiabatic lapse rate exceeds the dry adiabatic lapse rate is less than the dry adiabatic lapse rate

1047. Advection of very cold air over a warm sea current can cause: A. B. C. D.

no fog at all. advection fog. steaming fog. frontal fog.

1048. After a clear night cumuliform clouds are formed in the morning. Why can the base of these clouds become higher during the day? A. B. C. D.

Because the difference between the temperature and the dewpoint temperature at the initial condensation level becomes smaller. The wind speed is increasing, because the cold air mass changes into a warm air mass. Because the stability increases. Because the surface temperature increases.

1049. Air masses which are being cooled from below are often characterized by: A. B. C. D.

strong winds, cumulus clouds, good visibility. continuous rain and freezing temperatures. fog, poor visibility and layered clouds. uniform temperature, good visibility.

1050. At which height and at what time of the year can an aircraft be affected by the equatorial jet stream? A. B. C. D.

FL 400 during the winter in the southern hemisphere. FL 500 from November to February. FL 400 during the winter in the northern hemisphere. FL 500 from June to August.

1051. An air mass acquires its basic properties A. by widespread thunderstorms

ATP Online - Meteorology

Page 163 of 213

B. by stagnation of the air for a long period of time over areas having particular characteristics C. in the westerlies of the mid-latitudes D. by the influence of jet streams

1052. In which environment is aircraft structural ice most likely to have the highest accumulation rate? A. B. C. D.

Cirrus clouds. Cumulus clouds. Freezing rain. Stratus clouds.

1053. An aircraft descents in layered clouds; the freezing level is situated at FL 60. At what levels is the probability for airframe icing the highest? A. B. C. D.

At FL 140. Between FL 120 and FL 60. Between FL 60 and FL 20. Between FL 120 and FL 180.

1054. An inversion is A. B. C. D.

a layer that can be either stable or unstable a conditionally unstable layer an absolutely stable layer an unstable layer

1055. An occlusion is called a warm occlusion when the cold air A. B. C. D.

at the rear of the occlusion is colder than the cold air ahead, with the warm air at a higher altitude. ahead of the surface position of the occlusion is only at a higher altitude. at the rear of the occlusion is colder than the cold air ahead. at the rear of the occlusion is less cold than the cold air ahead, with the warm air at a higher altitude.

1056. An unstable air mass is forced to ascend a mountain slope. What type of clouds can be expected? A. B. C. D.

Layer-like clouds with little vertical development. Stratiform clouds with considerable turbulence. Clouds with considerable vertical development and associated turbulence. Stratiform clouds with a temperature inversion.

1057. As a result of diurnal variation radiation fog is lifted and a cloud cover is formed. Which statement is true? A. B. C. D.

Low stratus will develop caused by radiation in combination with low geostrophic wind speed. Low stratus will develop caused by increasing wind speed. Stratocumulus will develop caused by insolation. Layers of altocumulus will develop caused by increasing wind speed.

1058. At a cold front: A. B. C. D.

temperature rises owing to increased pressure. fog will form from the interaction of cold and warm air. warm air is lifted as cooler air pushes under it. warm air is compressed as cold air rises over it.

1059. At a station at the surface the significant weather with a warm front will come: A. B. C. D.

mostly before the front passes. after the front has passed. after the warm sector has passed. only at the same time as the front passes.

1060. An aeroplane flies at flight level 40. Elevation of the aerodrome: 990 ft; QNH 976 hPa. The tower clears the pilot to fly at 3000 ft QNH. Which of the following statements is correct? A. B. C. D.

Only a small change of altitude is necessary. The aeroplane has to climb about 1000 ft. The aeroplane has to descend about 1000 ft. The aeroplane has to descend about 2000 ft.

1061. The diagram of the system in annex represents a A. B. C. D.

cold occlusion. cold front. warm occlusion. warm front.

ATP Online - Meteorology

Page 164 of 213

1062. According to ICAO, which symbol indicates danger to an aircraft flying according to instrument flight rules (IFR)? A. B. C. D.

Symbol 1 Symbol 2 Symbol 4 Symbol 3

ATP Online - Meteorology

1063. The warm sector is indicated by: A. B. C. D.

number 3 number 4 number 1 number 2

1064. A convergence line is indicated by: A. B. C. D.

number 1 number 3 number 2 number 4

Page 165 of 213

ATP Online - Meteorology

Page 166 of 213

1065. In which air mass are extremely low temperatures encountered? A. B. C. D.

Continental polar air. Maritime polar air. Continental tropical air. Maritime arctic air.

1066. According to the extract of the surface isobar map, the surface wind direction over the sea is approximately A. B. C. D.

140° 110° 220° 310°

1067. In appendix are shown four sections of the 700 hPa wind chart. The diagram representing most accurately the wind direction and speed is

ATP Online - Meteorology A. B. C. D.

030°/30 kt (diagram A) 210°/30 kt (diagram D) 210°/30 kt (diagram B) 030°/30 kt (diagram C)

1068. At FL 300 between Geneva and Tunis, what mean wind would be most likely? A. B. C. D.

245/50 225/25 265/25 265/40

Page 167 of 213

ATP Online - Meteorology

Page 168 of 213

1069. The mean temperature that may be expected to affect that segment of the route from the coast of SE England to Geneva at FL 270 is A. B. C. D.

-34°C -42°C -38°C -30°C

ATP Online - Meteorology

Page 169 of 213

1070. The mean wind that may be expected to affect the route segment from the coast of SE England to Geneva at FL 270 is A. B. C. D.

270/70 245/55 220/70 240/90

ATP Online - Meteorology

Page 170 of 213

1071. Flight from Lisbon (LPPT) to Kingston (MKJP) Considering the route segment between 60°W and 70°W, at FL 390 the forecast mean temperature is A. B. C. D.

-52°C -55°C -58°C -61°C

ATP Online - Meteorology

Page 171 of 213

1072. At Lyon (LFLY, N4545 E00500), at 1200 UTC, the sky is overcast with Stratocumulus and Altostratus and it is raining. Using the SWC in appendix, valid at 1200 UTC, we can estimate a weather improvement for Lyon A. B. C. D.

2100 UTC 0300 UTC the following day 1330 UTC 1215 UTC

ATP Online - Meteorology

Page 172 of 213

1073. 8/8 stratus base 200 FT/AGL is observed at sunrise at an aerodrome in the north of France; the QNH is 1028 hPa and there is a variable wind of 3 kt. What change in these clouds is likely at 1200 UTC in summer and winter? A. B. C. D.

Winter: BKN, base 2500 FT/AGL; summer: BKN, base 3500 FT/AGL. Winter: clear sky; summer: BKN CB, base 1500 FT/AGL. Winter: OVC, base 500 FT/AGL; summer: SCT, base 3000 FT/AGL. Winter: SCT, base 3000 FT/AGL; summer: OVC, base 500 FT/AGL.

1074. What is the average temperature difference from ISA at FL 390 between Madrid and Dhahran? A. B. C. D.

+5°C -5°C -1°C +2°C

ATP Online - Meteorology

Page 173 of 213

1075. In which of the following METAR reports is the probability of fog formation in the coming night the highest? A. B. C. D.

201850Z 21003KT 8000 SCT250 12/m08 Q1028 NOSIG= 201850Z 06018G30KT 5000 -RA OVC010 04/01 Q1024 NOSIG= 201850Z 15003KT 6000 SCT120 05/04 Q1032 BECMG 1600 BR= 201850Z 25010KT 4000 RA BKN012 OVC030 12/10 Q1006 TEMPO 1500=

1076. In which of the following situations is the probability for severe thunderstorms the highest? A. B. C. D.

Advection of maritime cold air over a warm sea surface. Advection of maritime warm air over a cold sea surface. Advection of continental cold air over a warm land surface. Advection of warm air over a cold land surface.

1077. In which of these cloud types can icing be virtually ruled out? A. B. C. D.

SC NS AS CS

1078. According to ICAO, which symbol indicates danger to an aircraft flying according to instrument flight rules (IFR) ? A. B. C. D.

Symbol 4 Symbol 2 Symbol 3 Symbol 1

ATP Online - Meteorology

1079. A blocking anticyclone in the northern hemisphere is A. B. C. D.

a cold anticyclone/steering depressions/situated over Scandinavia a warm anticyclone/quasi stationary/situated between 50°N and 70°N situated between 50°N and 70°N/a cold anticyclone/steering depressions quasi stationary/situated between 50°N and 70°N/a cold anticyclone

1080. Judging by the chart, what wind speeds can you expect at FL 310 above London ? A. B. C. D.

300 kt 110 km/h 90 kt 140 kt

Page 174 of 213

ATP Online - Meteorology

Page 175 of 213

1081. In which conditions would you most likely encounter clear icing, and how would it normally appear? A. B. C. D.

Stratiform clouds, small water droplets, temperatures between -10°C and -20°C. Appears granular and tends to accumulate forward into the air stream. Cumuliform clouds, large water droplets, temperatures between 0°C and -15°C. Appears smooth and tends to spread back over an aircraft wing. Stratiform clouds, large water droplets, temperatures well below freezing. Appears opaque and builds forward from leading surface into a sharp edge. Cumuliform clouds, small water droplets, temperatures between -20°C and 25°C. Appears transparent and tends to take the shape of the surface on which it freezes.

1082. A Cb with thunderstorm has reached the mature stage. Which statement is correct? A. B. C. D.

In temperatures lower than -23°C icing is still possible. The freezing level in the whole cloud lies lower than outside the cloud. Severe turbulence occurs in the cloud, but hardly ever below the cloud. If hail occurs, it only occurs in downdrafts.

1083. The ICAO Standard Atmosphere (ISA) assumes that temperature will reduce at the rate of A. B. C. D.

1,98°C per 1000 FT up to 36090 FT after which it remains constant to 65617 FT 2°C per 1000 FT up to 36090 FT and will then increase at 0,3°C per 1000 FT up to 65617 FT 2°C per 1000 FT up to 65617 FT after which it will remain constant to 104987 FT 1,98°C per 1000 FT up to 36090 FT and will then rise at 0,3°C per 1000 FT up to 65617 FT when it will remain constant

1084. The region of the globe where the greatest number of tropical revolving storms occur is A. B. C. D.

the south-western Indian ocean, affecting Madagascar, Mauritius and the island of Réunion. the northern Indian ocean, affecting India, Sri Lanka and Bangladesh. the Caribbean sea, affecting the West Indies, Mexico and the south-east coastline of the USA. the north-west Pacific, affecting Japan, Taiwan, Korea and the Chinese coastline.

1085. Which of the following statements is an interpretation of the METAR ? 00000KT 0200 R14/0800U R16/P1500U FZFG VV001 m03/m03 Q1022 BECMG 0800 = A. B. C. D.

Visibility 200 feet, RVR for runway 16 more than 1500 metres, vertical visibility 100 feet, fog with hoar frost RVR for runway 14 800 metres, vertical visibility 100 feet, calm, visibility improving to 800 metres in the next 2 hours Visibility for runway 14 800 metres, fog with hoar frost, RVR for runway 16 more than 1500 metres Visibility 200 metres, RVR for runway 16 1500 metres, temperature -3°C, vertical visibility 100 metres

1086. Which of the four answers is a correct interpretation of data from the following METAR ? LSZH 050820Z 16003KT 0400 R14/P1500 R16/1000 FZFG VV003 M02/M02 Q1026 BECMG 2000 BR =

ATP Online - Meteorology A. B. C. D.

Page 176 of 213

Visibility 400 m, RVR for runway 16 1000 m with no distinct tendency, dew point -2°C, freezing fog. RVR for runway 16 1000 m with no distinct tendency, visibility increasing in the next 2 hours to 2000 m, vertical visibility 300 m, temperature -2°C. Visibility 1000 m, RVR 400 m, freezing level at 300 m, variable winds, temperature 2°C. RVR for runway 14 1500 m, visibility 400 m, QNH 1026 hPa, wind 160° at 3 kt.

1087. The following sequence of clouds is observed at an airport: cirrus, cirrostratus, altostratus, nimbostratus. This is typical for: A. B. C. D.

the passage of a squall line. the passage of a warm front. anticyclonic weather. the passage of a cold front.

1088. The Foehn wind is a A. B. C. D.

warm katabatic wind cold katabatic wind cold anabatic wind warm anabatic wind

1089. The diurnal variation in temperature is largest when A. B. C. D.

the sky is clear and the wind is strong the sky is overcast and the wind is weak the sky is clear and the wind is weak the sky is overcast and the wind is strong

1090. The dewpoint temperature A. B. C. D.

can not be lower than the air temperature can not be equal to the air temperature can be reached by lowering the pressure whilst keeping temperature constant can be reached by cooling the air whilst keeping pressure constant

1091. The average slope of a cold front is in the order of: A. B. C. D.

1:800 1:10 1:80 1:500

1092. The average position of the polar front in the northern hemisphere is: A. B. C. D.

more southerly during the summer than during the winter. more southerly during the winter than during the summer. located near 65°N during the whole year. located near 55°N during the whole year.

1093. The arctic front is the boundary between: A. B. C. D.

polar air and tropical air. cold polar air and less cold polar air. arctic air and tropical air. polar air and arctic air.

1094. The amount of water vapour which air can hold largely depends on: A. B. C. D.

relative humidity. stability of air. air temperature. the dew point.

1095. Which of these four METAR reports suggests that a thunderstorm is likely in the next few hours? A. B. C. D.

201350Z 34003KT 0800 SN VV002 M02/M04 Q1014 NOSIG = 201350Z 21005KT 9999 SCT040CB SCT100 26/18 Q1016 TEMPO 24018G30KT TS = 201350Z 16004KT 8000 SCT110 OVC220 02/M02 Q1008 NOSIG = 201350Z 04012KT 3000 BR OVC012 04/03 Q1022 BECMG 6000 =

1096. Which of the following statements concerning the tropopause is correct? A. B. C. D.

The temperature remains constant above and below the tropopause. The temperature of the tropopause at the equator is higher than at the poles. The temperature of the tropopause at the equator and at the poles is equal. The temperature lapse rate changes abruptly at the tropopause.

ATP Online - Meteorology

Page 177 of 213

1097. Which of the following weather reports could be, in accordance with the regulations, abbreviated to "CAVOK"? (MSA minus airport elevation equals: LSZB 10000 FT, LSZH 8000 FT, LSGG 12000 FT, LFSB 6000 FT) A. B. C. D.

LSZH VRB02KT 9000 BKN080 21/14 Q1022 NOSIG= LSGG 22006KT 9999 BKN090 17/15 Q1008 RERA NOSIG= LFSB 24008KT 9999 SCT050 18/11 Q1017 RERA NOSIG= LSZB 28012KT 9999 OVC100 16/12 Q1012 BECMG 5000 -RA=

1098. Which of the following weather reports could be, in accordance with the regulations, abbreviated to "CAVOK"? A. B. C. D.

26012KT 8000 SHRA BKN025 16/12 Q1018 NOSIG = 34004KT 7000 MIFG SCT260 09/08 Q1029 BECMG 1600 = 27019G37KT 9999 BKN050 18/14 Q1016 NOSIG = 00000KT 0100 FG VV001 11/11 Q1025 BECMG 0500 =

1099. Which of the following statements is true with regard to mountain waves? A. B. C. D.

The absence of cloud over high ground indicates the absence of mountain waves Flight with tailwind toward high ground is likely to be more hazardous than flight with headwind toward high ground Flight with headwind toward high ground is likely to be more hazardous than flight with tailwind toward high ground Mountain waves are not experienced beyond 100 miles downwind from initiating high ground, regardless of the height of the ground

1100. Which of the following statements is correct? A. B. C. D.

The Coriolis force is inversely proportional to the sine of latitude The Coriolis force is directed in the same sense at all latitudes At the equator horizontal motions do not induce any horizontal Coriolis force In gradient wind the Coriolis force is in balance with the horizontal pressure gradient force and the frictional force

1101. Which of the following statements is correct? A. B. C. D.

At warm fronts thunderstorms are often observed. Normally atmospheric pressure stops falling rapidly behind a warm front, the air temperature rises. Cumulus clouds and a good visibility are normally observed in a warm sector in winter. Cumulus clouds and a good visibility are normally observed in a warm sector in autumn.

1102. Which of the following statements is correct? A. B. C. D.

HZ is reported only when visibility is reduced to 2000 m or less VC denotes not at the aerodrome but not within 8 nm FG is reported only when visibility is reduced by water droplets or ice crystals to less than 1000 m When BR is reported then the visibility is more than 1000 m but less than 8000 m

1103. Which of the following statements is correct? A. B. C. D.

Mountain waves are always accompanied by rotor clouds Above the tropopause no turbulence occurs Haze is a reduction of visibility due to the presence of water vapour Airframe icing can occur in clear air

1104. Which statement concerning the cold front and warm front of a frontal depression in the northern hemisphere is correct? A. B. C. D.

While occluding the warm front always becomes a front aloft. The wind backs more at the warm front than at the cold front. The precipitation zone of the cold front is in general wider than the precipitation zone of the warm front. The risk of fog is greater ahead of and behind the warm front than ahead of and behind the cold front.

1105. In which situation is advection fog most likely to form? A. B. C. D.

An air mass moving inland from the coast in winter. A light breeze blowing colder air out to sea. A warm moist air mass on the windward side of the mountains. Warm moist air settling over a warmer surface under no-wind conditions.

1106. About ten identical clouds are in the sky, well isolated from one another, dense, with well defined contours, developping vertically in a cauliflower shape. The side of these clouds lit by the sun is bright white. Their base, relatively dark, is essentially horizontal and at FL 30, and their tops at FL 150. These clouds are A. B. C. D.

towering Cumulus; Altocumulus castellanus; broken Cumulus; Stratocumulus;

1107. An aircraft is approaching under visual flight rules an airfield (northern hemisphere) whose runway is parallel to the coast. When downwind over the sea, the airfield is on the left. What wind effect should be anticipated on final approach and landing during a sunny

ATP Online - Meteorology

Page 178 of 213

afternoon ? A. B. C. D.

Tailwind Crosswind from the left Crosswind from the right Headwind

1108. An aircraft is flying through the Alps on a very cold winter's day. The regional QNH is 1013 hPa. During the flight, you circle around a mountain at an altitude of its summit. What reading will the aneroid altimeter give, compared to the elevation of the summit? A. B. C. D.

The same altitude as the elevation of the summit; A lower altitude than the elevation of the summit; A higher altitude than the elevation of the summit; There is insufficient information to come to a conclusion;

1109. An aircraft is flying through the Alps on a warm summer''s day. The weather is fine, and there is a high pressure system in the area. During the flight, a mountain is passed at an altitude of its summit. What reading will the aneroid altimeter give, compared to the summit''s elevation? A. B. C. D.

A lower altitude than the elevation of the summit; There is insufficient information to come to a conclusion; A higher altitude than the elevation of the summit; The same altitude as the elevation of the summit;

1110. Flight from Bordeaux to Amsterdam, ETA 2100 UTC. What is the minimum visibility forecast for ETA Amsterdam ? TAF EHAM 281500Z 281601 14010KT 6000 -RA SCT025 BECMG 1618 12015G25KT SCT008 BKN013 TEMPO 1823 3000 RA BKN005 OVC010 BECMG 2301 25020KT 8000 NSW BKN020 = A. B. C. D.

5 NM 6 km 3 km 5 km

1111. Flight from Bordeaux to Amsterdam, ETA 2100 UTC. What lowest cloud base is forecast for arrival at Amsterdam? TAF EHAM 281500Z 281601 14010KT 6000 -RA SCT025 BECMG 1618 12015G25KT SCT008 BKN013 TEMPO 1823 3000 RA BKN005 OVC010 BECMG 2301 25020KT 8000 NSW BKN020 = A. B. C. D.

800 FT 500 FT 250 FT 500 m

1112. TAF EHAM 142300Z 150009 33005KT 9999 SCT025 BKN100 BECMG 0002 27015KT 4500 -SN SCT008 OVC015 TEMPO 0206 0400 +SN VV002 BECMG 0406 01008KT 9999 NSW SCT030 TEMPO 0709 03015G25KT 1200 SNSH SCT006 SCT015CB= What is the expected visibility at 0300 UTC? A. B. C. D.

Between 4500 m and 10 km or more Between 400 m and 10 km or more 4500 m Between 400 m and 4500 m

1113. The first clouds are thin, wispy cirrus, followed by sheets of cirrus and cirrostratus, and altostratus. The sun is obscured as the altostratus thickens and drizzle or rain begins to fall. The cloud base is lowering as Nimbostratus arrives. These phenomena indicate: A. B. C. D.

trade wind front sea-breeze front cold front warm front

1114. What is the lowest cloud base that can be expected from the forecast for 1500 UTC ? KCHS 280430Z 280606 VRB05KT 4000 BR SCT005 OVC013 PROB40 SHRA BECMG 1314 9000 SHRA OVC015 PROB40 TEMPO 1416 VRB15G25KT 1600 TSRA OVC010CB BECMG 1618 26010KT BKN030 BECMG 2122 CAVOK. A. B. C. D.

1500 feet 1300 feet 3000 feet 1000 feet

1115. What type of meteorological hazard to safe flight is most likely to be experienced during the final approach to Geneva (ETA 1200 UTC) ? TAF LSGG 020900Z 021019 18007KT 9999 SCT020 BKN100 BECMG 1114 8000 RA SCT010 OVC020 PROB30 TEMPO 1018 VRB10G25KT TSRA A. B. C. D.

Low level wind shear Radiation fog Anabatic winds Roll cloud associated with standing waves

ATP Online - Meteorology

Page 179 of 213

1116. In the middle of the eye of a hurricane the wind speed is; A. B. C. D.

lower than 63 knots and the air is ascending; higher than 63 knots and the air is ascending; higher than 63 knots and the air is descending; lower than 63 knots and the air is descending;

1117. When in the northern hemisphere at the same latitude the distance between isobars is equal then; A. B. C. D.

the coriolis force in a high pressure area is smaller than the gradient force in a high pressure area; the geostrophic wind speed is less than the gradient wind speed in a low pressure area; the wind around a low pressure area is stronger than around a high pressure area; the gradient wind in a low pressure area is weaker than in a high pressure area;

1118. What are the conditions required for the formation of hoar frost?; A. B. C. D.

Falling droplets on a surface with a temperature below freezing.; The temperature of the surface is higher than the dew point of the air and the dew point is lower than 0°C.; The temperature of the surface is lower than the dew point of the air and the dew point is lower than 0°C.; Falling droplets on a surface while the air temperature is below freezing.;

1119. Solid precipitation which is transparent or translucent and has a diameter of 5mm or less formed by the freezing of raindrops or refreezing of melted snow, and usually bounces off hard surfaces, is a description of; A. B. C. D.

ice pellets; hail; snow grains; small hail;

1120. Which of the following statements is correct concerning the geostrophic wind?; A. B. C. D.

A horizontal temperature difference of at least 5 degrees Celsius per 100 km is required to have geostrophic wind conditions.; It is present at latitudes higher than about 15 degrees north/south. It is only present in the friction layer.; It is present in a pressure system consisting of curved and non-parallel isobars.;

1121. In the trade wind belt between the surface and the 500 hPa level, which of the following is true? A. B. C. D.

the air is relatively moist the air is relatively dry in the lower part and moist in the upper part the air is relatively dry the air is relatively moist in the lower part and dry in the upper part

1122. What is the significance to aviation of breaks or steps in the tropopause ? A. B. C. D.

they indicate light winds at the surface (the ‘‘horse latitudes’’) they indicate the position and movement of anticyclones they indicate the position of strong upper winds they indicate the position and movement of tropical revolving storms

1123. What is the symbol for hail on a significant weather chart? A. B. C. D.

Symbol 2 Symbol 3 Symbol 4 Symbol 1

ATP Online - Meteorology

Page 180 of 213

1124. What is the ICAO qualifying term for the described intensity of turbulence:; "Conditions in which abrupt changes in aircraft attitude and/or altitude occur; aircraft may be out of control for short periods"? A. B. C. D.

Light Severe Noticeable Moderate

1125. A special air-report A. B. C. D.

may be the reason for closing a part of an airway due to meteorological conditions may be the reason for closing an airport due to meteorological conditions is used to produce better aerodrome forecasts (TAF) may trigger a SIGMET message

1126. What kind of turbulence is dependent on the sun's radiation and therefore follows a pronounced diurnal pattern? A. B. C. D.

Mechanical turbulence Orographic turbulence Convective turbulence Turbulence induced by wind speed

1127. Which of the following groups of conditions requires a crew to transmit a special air-report (subsonic air transport)? A. B. C. D.

Volcanic ash cloud; isolated thunderstorms; moderate or severe sandstorms Heavy wind shears; moderate or severe aircraft icing; moderate or severe turbulence Severe turbulence; widespread thunderstorms; severe mountain waves Moderate aircraft icing; severe turbulence; embedded thunderstorms

1128. Which of the following statements is correct concerning the typical weather in a warm sector in mid- and high latitudes over continental areas during summer? A. B. C. D.

Generally moderate to good visibility, haze, sometimes few or scattered cumulus Overcast by medium and high clouds, stratus with drizzle, poor visibility Generally good visibility outside the frequent showers of CBs Moderate to poor visibility, fog or stratus, stratocumulus and drizzle

1129. The average position of the polar front in the northern hemisphere is: A. B. C. D.

more southerly during the winter than during the summer. located near 55°N during the whole year. located near 65°N during the whole year. more southerly during the summer than during the winter.

1130. Which interpretation of the following message is correct? LFLD AD WRNG 1 VALID 050430/050900 FROST FCST= A. It is forecast for the period between 0430 and 0900 on the 5th of that month that especially an aircraft in the final approach phase will be covered by hoar frost. B. It is forecast for the period between 0430 and 0900 on the 5th of that month that a departing or arriving aircraft will pass a layer of ground fog in which the

ATP Online - Meteorology

Page 181 of 213

aircraft will be contaminated by hoar frost. C. It is forecast that some parts of the surface, vehicles, and aircraft parked outside will be contaminated by hoar frost between 0430 and 0900 on the 5th of that month. D. The aerodrome warning for runway 05 due to frost issued at 0430 UTC has been cancelled at 0900 UTC.

1131. Which of the following statements is correct concerning the geostrophic wind? A. B. C. D.

It is present in a pressure system consisting of curved and non-parallel isobars. It is only present in the friction layer. It is present at latitudes higher than about 15 degrees north/south. A horizontal temperature difference of at least 5 degrees Celsius per 100 km is required to have geostrophic wind conditions.

1132. Which of the following statements concerning the use of airborne weather radar in the vicinity of thunderstorms is true? A. B. C. D.

Accurate assessment of the weather ahead of the aircraft might be hampered due to the attenuation of the radar echoes by heavy rain. The radar reflectivity echoes are much more determined by the number of reflecting particles than by the particle size. The antenna tilt shall be adjusted in order to receive the most significant echoes (generally received from an altitude of approximately 30000 ft). The most significant echoes will be received due to lightning discharges near the freezing level.

1133. On an upper wind and temperature chart: A. B. C. D.

the temperature is constant all over the chart. the wind increases from south to north. the pressure is constant all over the chart. the height is constant all over the chart.

1134. Which of the following statements concerning the tropopause is correct? A. B. C. D.

The temperature lapse rate changes abruptly at the tropopause. The temperature of the tropopause at the equator is higher than at the poles. The temperature of the tropopause at the equator and at the poles is equal. The temperature remains constant above and below the tropopause.

1135. You are flying in the southern hemisphere at FL 180 from east to west. If the wind at FL 180 is southerly, which of the following statements is correct? A. B. C. D.

The true altitude is increasing. The true altitude is decreasing. The true altitude remains constant. A statement on altitude changes cannot be given without information on temperature.

1136. An aircraft is flying from Point A to Point B at the flight level corresponding to the pressure surface of the constant pressure chart. Which of the following statements is correct? A. B. C. D.

Wind speed over B is higher than over A Wind speed over A is higher than over B The true altitude will be higher over A than over B The true altitude will be higher over A than over Madrid

1137. An aircraft is flying from Point A to Point B at the flight level corresponding to the pressure surface of the constant pressure chart. Which of the following statements is correct? A. B. C. D.

The true altitude will be higher over A than over B Wind speed over A is higher than over B Wind speed over B is higher than over A The true altitude will be higher over B than over Melbourne

ATP Online - Meteorology

Page 182 of 213

1138. An aircraft is flying from Point A to Point B at the flight level corresponding to the pressure surface of the constant pressure chart. Which of the following statements is correct? A. B. C. D.

The true altitude will be higher over B than over London Wind speed over A is higher than over B Wind speed over B is higher than over A The true altitude will be higher over B than over A

1139. An aircraft is flying from Point A to Point B at the flight level corresponding to the pressure surface of the constant pressure chart. Which of the following statements is correct? A. B. C. D.

The true altitude will be higher over B than over Paris The true altitude will be higher over B than over A Wind speed over A is higher than over B Wind speed over B is higher than over Paris

1140. What is meant by the term "altitude"? A. B. C. D.

The vertical distance of a level or a point measured from mean sea level. Altimeter indication when altimeter subscale is set to present QFE. The vertical distance of a level or a point measured from the earth's surface. The vertical distance of a level or a point measured from the aerodrome reference point.

1141. In a warm occlusion

ATP Online - Meteorology A. B. C. D.

Page 183 of 213

the warm front becomes a front aloft the warm air is lifted the warm front overtakes the cold front the coldest air is lifted

1142. Which of the following statements is correct regarding the variation of wind with height? A. B. C. D.

The wind strength normally increases by a factor of 2 per 1000 feet up to the 850 hPa level. The wind strength normally increases by a factor of 2 per 500 feet up to the 700 hPa level. The wind normally veers during climb out through the first 2000-3000 feet in the northern hemisphere. The wind normally backs during climb out through the first 2000-3000 feet in the northern hemisphere.

1143. Which of the following statements is correct concerning the savannah climate? A. B. C. D.

There are no significant yearly variations in rainfall but there are constant daily variations There are significant yearly variations in rainfall with a wet and a dry period The annual mean temperature is above 25°C Little or no rainfall occurs

1144. What is the meaning of "RERA" in a METAR? A. B. C. D.

It is presently not raining, but there has been rain 2 hours ago. There is showery precipitation in the vicinity. The intensity of rain is decreasing (Retrograding Rain). There has been moderate or heavy rain since the last issue of METAR.

1145. In a westerly situation, the mean time interval between polar frontal waves in Western Europe is A. B. C. D.

three to four days six to twelve hours one to two days twelve to eighteen hours

1146. Which of the following is typical for the snow (tundra) climate? A. B. C. D.

Travelling depressions during winter time. High pressure weather dominates in summer. High pressure weather dominates in winter, with the sub-soil being frozen. Temperatures generally between +5 and +18°C throughout the whole year.

1147. Which of the following messages should be cancelled when the conditions are no longer occurring or no longer expected to occur? A. B. C. D.

Aerodrome warning Trend forecast GAMET TAF

1148. What name is given to the jet stream lying over Scandinavia (C)? A. B. C. D.

Tropical jet stream Subtropical jet stream Low level jet stream Polar front jet stream

ATP Online - Meteorology

Page 184 of 213

1149. Which climatic type is to be expected between 10°N and 10°S? A. B. C. D.

Dry climate Warm temperate rain climate Tropical transitional climate Tropical rain climate

1150. Lenticular clouds in mountainous areas indicate: A. B. C. D.

an inversion. unstable air. turbulence. light variable winds.

1151. Which of the following statements regarding the development of valley inversions is correct? A. The predominant factor in the development process of valley inversions is the non-uniformity of incident solar radiation due to orographic conditions in combination with density variations within the air. B. Usually a valley inversion is initiated by radiation cooling and anabatic winds which are due to orographic conditions. C. Valley inversions often are a result of radiation cooling in combination with gravity, both affecting the air at the surface of a mountain slope. D. The most important prerequisite for the development of valley inversions is warm air ascending along the mountain slope, thus leaving cooler air behind in the bottom of the valley.

1152. Two aircraft are flying simultaneously in level flight at different flight levels on opposite tracks overhead a given position.; Aircraft A flying at FL 070 experiences a 30 kt headwind.; Aircraft B flying at FL 060 experiences a 20 kt headwind.; None of the aircraft experiences cross wind. This wind condition will be referred to as A. B. C. D.

horizontal wind shear only vertical wind shear only vertical and horizontal wind shear no wind shear since the absolute value of the difference in wind velocity per height unit is too small to fit the definition of wind shear at all

1153. Which of the following processes contributes to the formation of frontal fog? A. B. C. D.

Maritime polar air flows over a warm ground Rain falls from the warm air into the cold air Very cold air flows over much warmer sea Moist air flows over land during a clear night

1154. Which degree of aircraft turbulence is determined by the following ICAO description? "Conditions in which abrupt changes in aircraft attitude and/or altitude occur; aircraft may be out of control for short periods. Usually, large variations in air speed. A. B. C. D.

Moderate Severe Violent Light

1155. What are the colours with which a stationary front is shown on a surface chart?

ATP Online - Meteorology A. B. C. D.

Page 185 of 213

Blue and violet Violet and red Red and blue Red and green

1156. A flight with a jet aeroplane from Munich to London is planned with a departure time of 2250 local time. Which type of satellite imagery available with observation time close to ETD is best to locate the clouds and the exact position of a cold front forecast for this night over Western Europe? A. B. C. D.

Visual (VIS), polar orbiting Visual (VIS), geo-stationary Water vapour (WV), geo-stationary or polar orbiting Infrared (IR), polar orbiting

1157. What are the normal and official sources of meteorological information for an aircraft in flight? A. B. C. D.

VOLMET broadcast; ATIS, and from the ATS-units the TWR only VOLMET broadcast; ATIS, and from the ATS-units the TWR, APP and FIC only VOLMET broadcast; ATIS, and from the ATS-units the FIC only VOLMET broadcast; ATIS; all ATS-units

1158. In this question the wind speed with straight isobars (vst), the wind speed around a high-pressure system (vhigh) and the wind speed around a low-pressure system (vlow) in the southern hemisphere are being compared while pressure gradient and latitude constant. Which statement is correct? A. B. C. D.

vst > vhigh and vst > vlow. vst > vhigh and vst < vlow. vst < vhigh and vst < vlow. vst < vhigh and vst > vlow.

1159. How would the term "vertical visibility is missing or not measurable" be indicated in a METAR? A. B. C. D.

VV000 VV/// VVmis VV999

1160. Which of the following thunderstorms produce the most severe conditions, such as heavy hail and destructive winds? A. B. C. D.

Warm front thunderstorms. Daytime air mass thunderstorms. Nocturnal air mass thunderstorms. Squall line thunderstorms.

1161. Which of the following front types is most known for gusty winds? A. B. C. D.

Cold front. Stationary front. Warm front. Occluded front.

1162. Occlusions are formed when A. B. C. D.

a cold front crosses a mountain ridge cold fronts overtake warm fronts warm fronts overtake cold fronts a warm front crosses a mountain ridge

1163. The main components of an altostratus are A. B. C. D.

water droplets and water vapour. water droplets and snow grains. ice crystals and water droplets. ice crystals and ice pellets.

1164. What is a VOLMET broadcast? A. B. C. D.

Weather information transmitted continuously via radio (special broadcast), containing weather information for one airport. Provision, as appropriate, of current METAR, SPECI, TAF and SIGMET by means of continuous and repetitive voice broadcast. That part of an ATIS which is concerned to meteorological information. Information issued by a meteorological watch office concerning the occurrence or expected occurrence of specified enroute weather phenomena which may affect the safety of aircraft operations.

1165. Imagine an aircraft during approach to an airfield which is located in a basin within mountainous area under the influence of a blocking anticyclone in winter. Mark the most reasonable statement given below.

ATP Online - Meteorology

Page 186 of 213

A. While approaching the airfield in the early morning hours, these conditions apply for the possibility of a marked valley inversion which has to be considered as a potential flight hazard due to a decreased lift after breaking through the inversion layer. B. While approaching the airfield in the late evening hours, without the pilot's intervention the engines of the aircraft are most likely to provide less thrust after breaking through the inversion layer. C. While approaching the airfield shortly before sunrise, the pilot has to consider turbulence because at first lift will be decreased due to katabatic winds and, after breaking through a valley inversion layer, lift is likely to improve due to anabatic wind D. While approaching the airfield in the early morning, after a short period of bumpiness the engines of the aircraft will deliver more thrust after breaking through the inversion layer if the pilot does not intervene.

1166. Consider a parcel of air being forced upwards in the atmosphere. The lapse rate of the surrounding air is less than the saturated adiabatic lapse rate. After the initial force is no longer effective, the parcel of air A. B. C. D.

will become conditionally unstable. will tend to descend to its original altitude. will tend to ascend further. will tend to remain at the new altitude.

1167. On which of the following aviation weather charts can a pilot most easily find a jet stream? A. B. C. D.

Upper air chart. Significant weather chart. Surface chart. Wind / temperature chart.

1168. Where does wind shear occur? A. B. C. D.

At any level in the atmosphere if associated with either a change of wind direction and/or wind speed. Wind shear of any significance occurs only in connection with jetstreams. Wind shear occurs primarily at lower altitudes in the vicinity of mountain waves. Wind shear occurs only when there is a strong temperature inversion, or when the jetstream is associated with a strong depression.

1169. In the tropopause the position of so called "tropopause breaks" is important for aviation because A. B. C. D.

moderate to severe icing often occurs in these breaks winds will reverse direction in these breaks the cores of jet streams can be located near these breaks from this position the direction of movement of high pressure systems can be derived

1170. Which of the following is correct regarding the wind speed in a jet stream in the northern hemisphere? A. B. C. D.

With colder air behind when looking downwind, the windspeed increases with height, while with the colder air ahead the windspeed decreases with height. With colder air to the left when looking upwind, the wind speed increases with height, while with the colder air to the right the wind speed decreases with height. With colder air ahead when looking downwind, the wind speed increases with height, while with the colder air behind the wind speed decreases with height. With colder air to the left when looking downwind, the wind speed increases with height, while with the colder air to the right the wind speed decreases with height.

1171. During an ILS approach on RWY 33, a northwesterly wind is blowing parallel to the runway. Its speed is increasing rapidly with height while its change in direction is negligible. What has the pilot to be aware of with respect to wind shear and glide path (no autopilot engaged)? A. B. C. D.

Without the pilot's intervention, the aircraft is likely to fly below the designated glide path with increasing deviation from it. Without the pilot's intervention, the aircraft is likely to fly above the designated glide path with decreasing deviation from it. A deviation from the glide path will not have to be considered since there is no significant wind shear to be expected. Without the pilot's intervention, the aircraft is likely to fly above the designated glide path with increasing deviation from it.

1172. During an ILS approach on RWY 33, a northwesterly wind is blowing parallel to the runway. Its speed is decreasing rapidly with height while its change in direction is negligible. What has the pilot to be aware of with respect to wind shear and glide path (no autopilot engaged)? A. B. C. D.

A deviation from the glide path will not have to be considered since there is no significant wind shear to be expected. Without the pilot's intervention, the aircraft is likely to fly below the designated glide path with decreasing deviation from it. Without the pilot's intervention, the aircraft is likely to fly below the designated glide path with increasing deviation from it. Without the pilot's intervention, the aircraft is likely to fly above the designated glide path with increasing deviation from it.

1173. Most tornadoes have a speed of movement that usually ranges from A. B. C. D.

120 to 180 knots. 30 to 70 knots. 20 to 40 knots. 100 to 140 knots.

1174. In the ICAO standard atmosphere, which of the following alternatives indicates the correct vertical temperature distribution in the lowest 11 km? A. B. C. D.

13°C at mean sea level, decreasing at 2°C per 1000 feet. 15°C at mean sea level, decreasing at 0.65°C per 100 feet. 15°C at mean sea level, decreasing at 0.65°C per 100 metres. 15.2°C at mean sea level, decreasing at 0.65°C per 100 metres.

ATP Online - Meteorology

Page 187 of 213

1175. On a route segment from A to B the highest terrain elevation is approximately 200 feet. For an instrument flight the minimum obstacle clearance is 1000 feet. The subscale of the altimeter is set to the local QNH of 1013 hPa and the temperature on the ground is -5°C. Assume a standard temperature lapse rate. What is the approximate minimum acceptable altimeter indication during the flight on this route segment? A. B. C. D.

1000 feet. 1400 feet. 1100 feet. 1300 feet.

1176. The mean height of the tropical zero degree isotherm is A. B. C. D.

14000 ft 16000 ft 19000 ft 12000 ft

1177. What is the approximate vertical interval which is equal to a pressure change of 1 hPa at sea level? A. B. C. D.

15 m (50 ft) 32 m (105 ft) 64 m (210 ft) 8 m (27 ft)

1178. Which one of the displayed cloud forms is representative of a cumulus? A. B. C. D.

3 2 4 1

1179. Which constant pressure chart is standard for FL 450? A. B. C. D.

150 hPa 200 hPa 250 hPa 300 hPa

1180. Which of the following constant pressure charts would be most relevant for planning a flight at FL 430? A. B. C. D.

300 hPa 150 hPa 250 hPa 200 hPa

1181. What lowest cloud base is most likely to be experienced during an approach into Madrid at 2300 UTC? TAF LEMD 281200Z 281812 13005KT CAVOK TEMPO 1821 8000 SCT020 SCT030 BECMG 2123 21005KT 9999 SCT015 BKN080 PROB40 TEMPO 2306 6000 SCT008= A. B. C. D.

1500 feet 800 feet 2000 feet 3000 feet

1182. What is the lowest possible cloud base forecast for 2300 UTC? EDDF 272200Z 280624 VRB05KT 4000 BR SCT005 OVC013 BECMG 1314 9000 SHRA OVC015 PROB40 TEMPO 1416 VRB15G25KT 1600 TSRA OVC010CB BECMG 1618 26010KT BKN030 BECMG 2122 CAVOK= A. B. C. D.

1500 feet 5000 feet 3000 feet 500 feet

1183. You are flying at FL 300 where the outside air temperature is -57.5°C and the pressure at MSL is 1013.25 hPa. If you assume that the difference between the actual temperature and the temperature in the ISA is valid for the whole troposphere, then the true altitude is: A. 28500 ft

ATP Online - Meteorology

Page 188 of 213

B. 31500 ft C. 30000 ft D. 27000 ft

1184. Which constant pressure chart is standard for FL 340? A. B. C. D.

200 hPa 400 hPa 300 hPa 250 hPa

1185. Which of the following constant pressure charts would be most relevant for planning a flight at FL 330? A. B. C. D.

300 hPa 200 hPa 250 hPa 400 hPa

1186. Given: Altimeter setting: 1013.2 hPa Altimeter reading: 5000 ft Outside air temperature at 5000 ft: +5°C QFE: 958 hPa QNH: 983 hPa What is the true height of the aircraft above the aerodrome? A. B. C. D.

3515 ft 4865 ft 4325 ft 4190 ft

1187. The mean height of the tropical tropopause is A. B. C. D.

49000 ft 54000 ft 59000 ft 44000 ft

1188. What average geographical latitude is assumed for the zone of the subtropical high? A. B. C. D.

50°N 30°N 0° (equator) 10°N

1189. If a flight at FL 150 is planned, which of these upper wind and temperature charts would be nearest to that flight level? A. B. C. D.

500 hPa 600 hPa 700 hPa 850 hPa

1190. By volume, what percentage of the air in the lower troposphere consists of water vapour? A. B. C. D.

5-10 % 20-30 % 0-5 % 10-15 %

1191. Which constant pressure chart is standard for FL 140? A. B. C. D.

600 hPa 850 hPa 500 hPa 700 hPa

1192. Which constant pressure chart is standard for FL 240? A. B. C. D.

400 hPa 500 hPa 600 hPa 300 hPa

1193. Which of the following constant pressure charts would be most relevant for planning a flight at FL 260? A. B. C. D.

600 hPa 300 hPa 500 hPa 400 hPa

ATP Online - Meteorology

Page 189 of 213

1194. Which of the following combinations of pressure and temperature is correct in the ICAO standard atmosphere? A. B. C. D.

300 hPa -50.5°C 500 hPa -21.1°C 700 hPa -8.4°C 200 hPa -62.3°C

1195. Which cloud species is described by the following definition?; "Clouds which present, in at least some portion of their upper part, cumuliform protuberances in the form of turrets which generally give the clouds a crenellated appearance. The turrets, some of which are taller than they are wide, are connected by a common base and seem to be arranged in lines. The typical character is especially evident when the clouds are seen from the side." A. B. C. D.

Calvus Castellanus Congestus Fractus

1196. The term CAVOK is used when weather conditions are: A. B. C. D.

9000, SKC, NOSIG 9999, NSC, NOSIG 9999, NSC, NSW 8000, HAZARDOUS WX NIL, NOSIG

1197. The presence of ice pellets at the surface is the evidence that: A. B. C. D.

temperatures are above freezing at some higher altitudes. after take-off you can climb to a higher altitude without encountering more than light icing conditions. there are thunderstorms in the area. a cold front has passed.

1198. What change in temperature will occur at point B during the next hour? A. B. C. D.

Approximately constant temperature A drop in temperature Irregular fluctuations A rise in temperature

1199. Katabatic wind is A. B. C. D.

a flow of warm air blowing from land to sea. a flow of warm air up the slope of a hill, caused by surface heating. a flow of cold air blowing from sea to land. a flow of cold air down the slope of a mountain.

1200. Which of the following layers of fog above land is coded as MIFG? A. B. C. D.

A layer of 20 feet deep. A layer of 10 feet deep. A layer of 5 feet deep. A layer of 15 feet deep.

ATP Online - Meteorology

Page 190 of 213

1201. Which FL corresponds with the 150 hPa pressure level? A. B. C. D.

FL 300 FL 390 FL 340 FL 450

1202. What is an aerodrome warning ? A. A message prepared by a meteorological office concerning wind shear which could adversely affect aircraft on the approach path or take-off path or during circling approach between runway level and a specified upper level, and aircraft on the runway during B. A message issued by a meteorological office concerning the meteorological conditions which could adversely affect aircraft on ground, including parked aircraft, and the aerodrome facilities and services. C. A message issued by a meteorological watch office concerning the occurrence and/or expected occurrence of specified en-route weather phenomena, which may affect the safety of aircraft operations. D. The part of a METAR message that is called supplementary information.

1203. Imagine an airfield during summer with sunny days and clear nights and with a runway parallel to the shoreline of a large inland lake 5 nautical miles away from the field.; A small aircraft heading for the airfield in the afternoon experiences wind shear during approach and a cross wind component which makes landing difficult.; The origin of the wind shear and cross wind is most likely to be seen in A. B. C. D.

a strong land breeze blowing more or less perpendicular to the runway. a strong sea breeze blowing more or less perpendicular to the runway. a radiation inversion due to surface cooling during the clear night. a subsidence inversion typical for summer anticyclones causing variations in wind speed and direction near the surface.

1204. What change in pressure, will occur at point B, during the next hour? A. B. C. D.

A drop in pressure A rise in pressure Approximately constant pressure Irregular fluctuations

1205. What change in pressure, will occur at point F, during the next hour? A. B. C. D.

A drop in pressure. Irregular fluctuations. Constant pressure. A rise in pressure.

ATP Online - Meteorology

1206. What change in temperature will occur at point A during the next hour? A. B. C. D.

Approximately constant temperature A rise in temperature A drop in temperature Irregular fluctuations

1207. In which situation is advection fog most likely to form? A. B. C. D.

An air mass moving inland from the coast in winter. A warm moist air mass on the windward side of the mountains. A light breeze blowing colder air out to sea. Warm moist air settling over a warmer surface under no-wind conditions.

1208. Which of the following circumstances most favours the development of orographic fog? A. B. C. D.

High relative humidity Katabatic winds Absolutely unstable air Clear nights

1209. In an isothermal layer the state of the atmosphere is A. absolutely unstable. B. absolutely stable. C. conditionally unstable.

Page 191 of 213

ATP Online - Meteorology

Page 192 of 213

D. neutral.

1210. A temperature inversion indicates a state of the atmosphere which is A. B. C. D.

indifferent absolutely stable. conditionally unstable. absolutely unstable.

1211. Which of the following processes will increase the stability of an air mass? A. B. C. D.

Advection of colder air aloft. Warming of the air mass from below. Addition of water vapour in the lower layer. Cooling by the underlying surface.

1212. Which of the following circumstances most favours the development of frontal fog? A. B. C. D.

Absolutely unstable air Increase of the relative humidity by evaporation of rain drops Advection of very cold air over much warmer sea Nocturnal radiation

1213. In which of the following situations is the probability for severe thunderstorms the highest? A. B. C. D.

Advection of continental cold air over a warm land surface. Advection of warm air over a cold land surface. Advection of maritime cold air over a warm sea surface. Advection of maritime warm air over a cold sea surface.

1214. What is determined by a transmissometer? A. B. C. D.

Air density. Height of cloud base. Vertical visibility. Visibility, runway visual range.

1215. Which of the following statements concerning an isothermal layer in the atmosphere is correct? A. B. C. D.

The vertical temperature gradient lies between the dry and saturated adiabatic lapse rates. Air parcels that are forced to rise through an isothermal layer keep the same temperature. An isothermal layer is formed when the air in the layer is well mixed by turbulence. An isothermal layer is absolutely stable.

1216. When the subscale of a pressure altimeter is set to the QFE of the destination airfield it will indicate A. B. C. D.

airfield elevation at landing roll out more than the airfield elevation zero at landing roll out the aircraft's altitude above MSL

1217. Orographic thunderstorms may occur: A. B. C. D.

almost exclusively during the daylight hours. at any time during day and night. mostly between sunset and 2 a.m. mostly in the early morning.

1218. Which one of the following cloud types can be characterised by the optical phenomenon called halo? A. B. C. D.

Cirrostratus. Altostratus. Altocumulus. Cirrocumulus.

1219. Which cloud type (genus) is described by the following definition? ; "Grey cloud layer, often dark, the appearance of which is rendered diffuse by more or less continuously falling rain or snow, which in most cases reaches the ground. It is thick enough throughout to blot out the sun. Low, ragged clouds frequently occur below the layer, with which they may or may not merge." A. B. C. D.

Stratocumulus Stratus Nimbostratus Altostratus

ATP Online - Meteorology

Page 193 of 213

1220. What is a stormscope? A. B. C. D.

An instrument on board an aircraft to detect electrical discharges. The technical term (jargon) for a radar scope showing thunderstorms. An instrument to measure wind velocity. An instrument to detect wind shear by means of the Doppler-effect during flight.

1221. Which of the following is correct regarding a warm anticyclone? A. B. C. D.

It remains the same strength up to the tropopause. It increases in intensity with increasing altitude. It weakens with altitude. Apart from a surface inversion, the temperature increases with increasing altitude.

1222. Which of the following is correct regarding a cold high pressure area? A. B. C. D.

Both temperature and pressure will remain constant with increasing altitude. It will increase in intensity with increasing altitude. It will decrease in intensity with increasing altitude. Apart from a surface inversion, the temperature will increase with increasing altitude.

1223. Most tornadoes have a life span that lasts for A. B. C. D.

a few minutes up to 30minutes. approximately one hour. not more than one minute. several hours.

1224. In which of the following areas is the frequency of tropical revolving storms the highest? A. B. C. D.

South Atlantic Ocean Indonesia South China Sea and the Philippines area Arabian Sea (Sea area west of India)

1225. Snow grains A. B. C. D.

are typically formed inside a CB, where they are being tossed up and down while growing to such size that they could no longer be supported in the updraught. are mainly formed inside altostratus and nimbostratus. fall from stratus or supercooled fog. are normally transparent.

1226. In the North Atlantic, you can often see a series of depressions located in a row and travelling from west to east. These depressions A. B. C. D.

are mostly generated by convergence between the subtropical highs and the equatorial trough. are primarily generated by the vaporization process that takes place above the ocean. are normally generated at the polar front. are residuals of tropical cyclones in the Florida area.

1227. Which of the following cloud types is classified as low level cloud? A. B. C. D.

AS CS SC AC

1228. How is an easterly wave identifiable on a surface weather chart? A. B. C. D.

As a weak trough As a well developed depression As a secondary depression As a ridge

1229. The processes and/or effects which cause clouds to release precipitation are A. B. C. D.

convection effect and advection effect. Bergeron-Findeisen process and convection effect. advection effect and coalescence process. coalescence process and Bergeron-Findeisen process.

1230. Clouds will mainly consist of supercooled water droplets when the temperature is: A. B. C. D.

between -30° C and -40° C. between 0° C and -15° C. below -40° C. between -5° C and -30° C.

ATP Online - Meteorology

Page 194 of 213

1231. An aircraft descends in layered clouds; the freezing level is situated at FL 60. At what levels is the probability for airframe icing the highest? A. B. C. D.

Between FL 60 and FL 20. Between FL 120 and FL 180. At FL 140. Between FL 120 and FL 60.

1232. Which cloud species is described by the following definition?; "Cumulonimbus characterised by the presence, mostly in its upper portion, of distinct cirriform parts of clearly fibrous or striated structure, frequently having the form of an anvil, a plume or a vast, more or less disorderly mass of hair. This specie is usually accompanied by a shower, or by a thunderstorm, often with squalls and sometimes with hail; it frequently produces very well defined virga." A. B. C. D.

Fractus Capillatus Congestus Calvus

1233. Runway visual range is calculated from values derived from A. B. C. D.

ceilometers anemometers psychrometers transmissometers

1234. Which of the following clouds may extend into more than one étage? A. B. C. D.

Cirrocumulus Altocumulus Stratus Cumulus congestus

1235. Which cloud type (genus) is described by the following definition? ; "Greyish or bluish cloud sheet or layer of striated, fibrous or uniform appearance, totally or partly covering the sky, and having parts thin enough to reveal the sun at least vaguely, as through ground glass. This genus does not show halo phenomena." A. B. C. D.

Altostratus Cirrostratus Stratus Stratocumulus

1236. Which cloud type (genus) is described by the following definition? "Detached clouds in the form of white, delicate filaments or white or mostly white patches or narrow bands. These clouds have fibrous (hair-like) appearance, or a silky sheen, or both." A. B. C. D.

Stratus Cirrostratus Altostratus Cirrus

1237. The pressure distribution located mainly in square 2A is a A. B. C. D.

ridge of high pressure trough of low pressure depression col

ATP Online - Meteorology

Page 195 of 213

1238. Which of the following situations will most likely lead to a stable air mass? A. B. C. D.

Widespread convection. Decreasing wind and developing showers. Subsidence and/or cooling from below. Cold air at higher levels and advection of warm air at lower levels.

1239. Which of the following statements concerning a cold air pool is correct? A. B. C. D.

Cold air pools can only be active for several hours Cold air pools normally disappear at night Direction and speed of movements of cold air pools are difficult to forecast During summer cold air pools are more active over the sea than over land areas

1240. An air mass which originates over the North Atlantic between 50 and 70 degrees north and is moving in over Scandinavia is called A. B. C. D.

maritime arctic air. maritime polar air. continental arctic air. continental polar air.

1241. What type of weather can usually be expected in a polar maritime air mass over Central Europe in the daytime during summer? A. B. C. D.

Sky clear. Continuous rain and poor visibility. Drizzle and low stratus. Showers and good visibility.

1242. What kind of turbulence is caused by the friction of air flowing over the earth's surface? A. B. C. D.

Turbulence in downdrafts Convective turbulence Clear air turbulence Mechanical turbulence

1243. Which force causes the surface wind, in the northern hemisphere, to deflect to the left compared to the geostrophic wind? A. B. C. D.

Pressure gradient force. Coriolis force. Centripetal force. Frictional force.

1244. Pressure altitude is obtained by: A. B. C. D.

setting the altimeter to a station pressure which has been corrected to sea level. setting the altimeter to QFF pressure. correcting the altimeter for temperature deviation from ISA. setting the altimeter to standard sea level pressure.

ATP Online - Meteorology

Page 196 of 213

1245. Which cloud type (genus) is described by the following definition? "Cumulus clouds which are markedly sprouting and are often of great vertical extent; their bulging upper part frequently resembles a cauliflower." A. B. C. D.

Cumulus fractus Cumulus humilis Towering cumulus Cumulonimbus capillatus

1246. Which of the following statements is correct? A. B. C. D.

At warm fronts thunderstorms are often observed. Cumulus clouds and a good visibility are normally observed in a warm sector in winter. Cumulus clouds and a good visibility are normally observed in a warm sector in autumn. Normally atmospheric pressure stops falling rapidly behind a warm front, the air temperature rises.

1247. How is the Mediterranean climate best described? A. B. C. D.

Anticyclonic and hot in summer, frontal depressions in winter. Cyclonic and chilly in summer, frontal depressions in winter. Anticyclonic and warm the whole year. Depressions due to the polar front affect the weather throughout the whole year.

1248. The arrows labelled "v" represent the mean tracks of tropical revolving storms which occur mainly from A. B. C. D.

December to April and are called cyclones May to November and are called Willy Willys May to November and are called typhoons December to April and are called hurricanes

1249. When a front crosses a mountain range, its activity A. B. C. D.

ceases immediately. decreases when it reaches the mountains. is not disturbed by the mountains. strengthens "upwind" of the mountains.

1250. Frontal fog will most likely dissipate with A. B. C. D.

decreasing intensity of precipitation the passage of a warm front backing of the wind a decrease in wind speed

1251. One of the main reasons for radiation fog to dissipate or become low stratus is: A. B. C. D.

lifting against the slope of a hill. surface heating. surface cooling. decreasing wind speed.

ATP Online - Meteorology

Page 197 of 213

1252. What is the meaning of the different coloured areas on a plan position indicator of an airborne weather radar? A. B. C. D.

Different ranges of cloud thickness (vertical extent) Different horizontal dimensions of clouds Different ranges of intensities of precipitation Different intensities of turbulence (light, moderate, severe) within clouds

1253. Which statement does correspond to the definition of severe turbulence? A. B. C. D.

Aircraft gets out of control and crashes. Difficulty in walking, occupants feel strain against seat belts, loose objects move about. Aircraft may be out of control for short periods, occupants are forced violently against seat belts, loose objects are tossed about. Aircraft will be damaged and an emergency landing will be absolutely necessary.

1254. Which FL corresponds with the 400 hPa pressure level? A. B. C. D.

FL 140 FL 240 FL 300 FL 340

1255. Which FL corresponds with the 250 hPa pressure level? A. B. C. D.

FL 390 FL 340 FL 450 FL 300

1256. Central Europe is situated in the zone of A. B. C. D.

mid-latitude climate dry climate subarctic climate snow climate

1257. An aerodrome warning message gives information, among other things, about A. B. C. D.

PL; SS; RA TS; SN including the expected or observed snow accumulation; hoar frost DS; SN clearing; FZRA including the observed braking action GR; FG; FZRA including the observed friction coefficient

1258. Mark the correct statement concerning the flight through an inversion caused by subsidence. A. During climb, the increased air density above the inversion layer will cause a certain increase in the aircraft performance (i.e. lift and thrust) and the pilot will experience a gradual change of the cross wind component. B. During climb, the pilot of an aircraft probably will experience a slight deterioration of flight visibility when breaking through the top of the inversion layer and the general aircraft performance will decrease. C. The pilot of an aircraft probably will experience a gradual increase of the cross wind component and a gradual improvement of flight visibility when breaking through the base of the inversion layer during descent. D. During climb, the reduced air density above the inversion layer will cause a certain decrease in the aircraft performance (i.e. lift and thrust) and the pilot will experience a sudden improvement of visibility.

1259. The weather activity within a cold air pool is usually greatest A. B. C. D.

early in the morning at noon in the afternoon in the late evening

1260. What wind direction will occur at point B? A. B. C. D.

East Southeast Southwest Northwest

ATP Online - Meteorology

Page 198 of 213

1261. What is a radiosonde? A. Equipment used for determining upper winds (velocity and direction) by transmitting different radio waves and measuring the amount of the reflected waves by the moving air. These values correlate to the speed and direction of the wind. B. A meteorological station on ground or water at which measurements of meteorological variables are made automatically and a radio transmitter sends automatically this information to a meteorological centre. These data from such a station are indicated in a C. Equipment used for determining upper winds only (velocity and direction) by tracking a free balloon by electronic means at the observing station. D. Instrument intended to be carried by a balloon up through the atmosphere, equipped with devices to measure meteorological variables and provided with a radio transmitter for sending this information to the observing station.

1262. What are the typical weather conditions in an area with a flat pressure pattern in summer? A. B. C. D.

Only short term weather improvements Steady rainfall Generally fine weather, possibly thunderstorms in the afternoon or evening Fine weather with strong westerly winds

1263. Which FL corresponds with the 600 hPa pressure level? A. B. C. D.

FL 100 FL 140 FL 240 FL 180

1264. What is the best way in dealing with thunderstorms associated with a cold front? A. B. C. D.

Avoidance of embedded CBs by using airborne weather radar. Flying underneath the cloud base in all cases. Flying through the upper third of the clouds to prevent the aircraft from being struck by lightning. Flying through the front with maximum speed and perpendicular to the front line in order to minimize the time of hazard exposure.

1265. During the preparation for approach to Zurich airport (1416 ft/AMSL) a pilot hears the following message on the ATIS: "Inversion warning. Inversion of 12°C between 1900 ft and 2300 ft altitude." This message indicates that the pilot should be prepared: A. B. C. D.

for fog below the inversion for wind shear just above the inversion that the performance of the engine will be reduced from inversion level down to ground for freezing precipitation below the inversion

1266. The intertropical convergence zone A. B. C. D.

forms the boundary between tropical and equatorial air nearly always coincides with the geographical equator is characterized by different wind directions on both sides of the zone is always characterized by CB clouds with high tops

1267. On which latitudes can the coriolis force in practice be neglected for meteorological purposes? A. From 0 to about 10 degrees north and south. B. From about 75 to 90 degrees north and south. C. From about 30 to about 75 degrees north and south.

ATP Online - Meteorology

Page 199 of 213

D. From about 10 to about 30 degrees north and south.

1268. What kind of fog is often observed in the coastal region of Newfoundland in spring time? A. B. C. D.

Frontal fog. Radiation fog. Advection fog. Steam fog.

1269. Which of the following changes of state is known as condensation? A. B. C. D.

Liquid to solid Liquid to gas Gas to liquid Gas to solid

1270. Which of the following changes of state is known as freezing? A. B. C. D.

Gas to solid Liquid to solid Solid to liquid Liquid to gas

1271. What are the typical weather conditions in a cold air pool in summer? A. B. C. D.

Ground fog formation is common Generally fine weather, thunderstorms in the afternoon or evening Precipitation is predominant Fine weather with thunderstorms in the morning or forenoon

1272. Which of the following combinations is most characteristic of unstable air behind a cold front? A. B. C. D.

Good visibility between showers, showery precipitation, cumuliform clouds Poor visibility, continuous precipitation, stratiform clouds Good visibility between showers, persistent precipitation, stratiform clouds Poor visibility, intermittent precipitation, cumuliform clouds

1273. With which meteorological phenomena are wind shear conditions mostly associated? A. B. C. D.

Thunderstorms; drizzle; conditional instability Gust fronts; sea breeze fronts; high stratiform clouds Gust fronts; low level temperature inversions; frontal surfaces Strong surface winds coupled with local topography; anticyclones; light precipitation

1274. When water vapour condenses or water freezes A. B. C. D.

heat is released and heat is required respectively heat is released in both cases heat is required in both cases heat is required and heat is released respectively

1275. In the following TAF, what is the forecast weather at 0800 UTC?; TAF EHAM 152300Z 160009 33005KT 9999 SCT025 BKN100 BECMG 0002 27015KT 4500 -SN SCT008 OVC015 TEMPO 0206 0400 +SN VV002 BECMG 0406 01008KT 9999 NSW SCT030 TEMPO 0709 03015G25KT 1200 SHSN SCT006 SCT015CB= A. B. C. D.

Moderate snow showers or no significant weather Moderate snow showers or heavy snow Light snow or moderate snow showers Heavy snow or no significant weather

1276. How is a ceiling defined? A. B. C. D.

Height above ground or water of the lowest layer of cloud that contributes to the overall overcast. Height above ground or water of the lowest layer of cloud below 20000 ft covering more than half of the sky. Height above ground or water of the lowest layer of cloud independent on the amount of clouds. Height above ground or water of the highest layer of cloud or obscuring phenomena aloft that covers 4 oktas or more of the sky.

1277. Troughs are extensions of A. B. C. D.

low pressure areas and generally have small amounts of cloud high pressure areas and generally have large amounts of cloud low pressure areas and generally have large amounts of cloud high pressure areas and generally have small amounts of cloud

ATP Online - Meteorology

Page 200 of 213

1278. Which of the following situations will be typical for the intertropical convergence zone? A. B. C. D.

High pressure the whole year, accompanied by calm winds. Low pressure and instability. High pressure the whole year, accompanied by strong prevailing winds. Hot, mainly dry climate.

1279. By volume, which of the following elements makes up the largest part of the atmosphere? A. B. C. D.

Noble gases. Oxygen. Nitrogen. Hydrogen.

1280. What degree of aircraft icing is determined by the following ICAO description? "Conditions in which immediate change of heading and/or altitude is considered essential" A. B. C. D.

Light Severe Violent Moderate

1281. The main components of a cirrostratus are A. B. C. D.

water droplets. ice crystals. ice crystals and water droplets. water droplets and water vapour.

1282. Mark the statement most reasonable regarding a valley inversion. A. Small scale density variations due to turbulent mixing (mainly caused by the roughness of rocks) are a significant prerequisite for the formation of valley inversions. B. If the incident solar radiation is non-uniform because of orographic conditions, the development of a valley inversion is very likely during daytime. C. Over night, cool dense air as a result of radiation cooling will descend along the mountain slopes into the basin and lead to the development of a valley inversion. D. Provided that the lee side winds are able to displace the air at the bottom of the basin, the Foehn effect is the predominant factor in the development of a valley inversion.

1283. Where does one normally find the highest cloud tops? A. B. C. D.

In a tropical cyclone Near an occlusion point In a cold front In a polar low

1284. When flying in mountainous terrain and assuming ISA temperatures and strong winds, which of the following is correct? A. B. C. D.

In case of combined effect of surface friction and adiabatic compression of the air, the dynamic pressure is lower than indicated by the altimeter. In case of venturi effect, the indicated altitude of the aircraft is lower than the actual altitude. In case of adiabatic compression of the air, the temperature is lower than indicated. In case of venturi effect, the actual altitude of the aircraft is lower than the indicated altitude.

1285. Where is the wind speed of a polar front jet stream normally at its strongest? A. B. C. D.

In the area of a trough. In the area where the jet stream is curving around an anticyclone. In the area between a trough and a ridge. In the area where the jet stream is curving around a depression.

1286. Where are the westerlies to be expected? A. B. C. D.

In the mid-latitudes Between 65° and 80° north or south Between 10° and 30° north or south In the subtropical high pressure belt

1287. In which areas are easterly waves most likely to form? A. B. C. D.

In the region of the equator between 0° and 5° north or south Just north of the prevailing westerlies In the trade wind zone In the subtropical high pressure belt between 20° and 35° north or south

1288. Where are easterly waves found? A. Between subtropical high pressure cells and ITCZ.

ATP Online - Meteorology

Page 201 of 213

B. In the region of the subtropical jet stream. C. On the poleward side of the subtropical high pressure belt. D. In the temperate latitudes.

1289. Which of the following statements is correct concerning the mediterranean climate? A. B. C. D.

The annual rainfall is significantly below 700mm It is not influenced by the westerlies Hot summers alternate with cold winters In winter it is influenced only by the subtropical high pressure belt

1290. For the planning of a flight with an estimated time of departure 0930 local time from an European airport a satellite picture is used to locate areas of fog. What type of satellite imagery available with observation time close to ETD does provide most likely the best information to locate these areas of fog? A. B. C. D.

Infrared (IR) and water vapour (WV), polar orbiting system Visual (VIS) and infrared (IR), polar orbiting system Infrared (IR), geo-stationary system Water vapour (WV), geo-stationary system

1291. For a VFR flight severe airframe icing may occur when flying A. B. C. D.

into freezing rain, resulting in clear ice formation between two cloud layers, without precipitation, resulting in clear ice formation into supercooled rain, resulting in rime ice formation into an area outside of clouds where the temperature is below 0°C, resulting in rime ice formation

1292. The eye of a hurricane A. B. C. D.

can not be spotted by satellites is filled with air which is colder than the air in the surrounding regions extends from the surface to the top of the hurricane has a diameter of at least 100 nautical miles

1293. Which of the following is correct regarding geostrophic wind? A. It blows parallel to straight equidistant isobars. B. It blows across the isobars from high to low pressure. C. It is the wind resulting from the vector sum of gradient wind and surface wind, such that the geostrophic wind normally blows at an angle of 20-45 degrees relative to the isobars. D. It blows parallel to curved isobars.

1294. Which of the following statements concerning airborne weather radar is correct? A. B. C. D.

It can also be used for proximity warning and anti-collision protection. It shows the areas which are dangerous for aircraft icing. The intensity levels for aircraft icing as defined by ICAO are indicated by contour lines. The indication on the plan position indicator represents the amount (mm/h) of water vapour within clouds. It shows on a plan position indicator the areas of precipitation of rain, snow and/or hail.

1295. Which statement is true for advection fog? A. B. C. D.

It usually forms by night and clears by day It forms when unstable air is adiabatically cooled It can appear suddenly by day or by night It develops slowly and clears fast

1296. Which of the following statements is true concerning orographic fog? A. B. C. D.

It forms suddenly by day but not by night It may be formed by day as well as by night It develops slowly and dissipates rapidly It is formed when the air is unstable

1297. What is a Foehn wind? A. It is a warm and moist anabatic wind that usually carries precipitation. B. It is an adiabatically heated wind blowing down a mountain side. The temperature on the lee side is normally lower than on the windward side of the mountain at the same level. C. It is an adiabatically heated wind blowing down a mountain side. The temperature on the lee side is normally higher than on the windward side of the mountain at the same level. D. It is a flow of warm air up a slope of a hill caused by surface heating.

1298. Which statement concerning the Scirocco is correct? A. The carried dust and sand does not reach great heights. This is caused by the presence of a trade wind inversion. B. It is formed by a strong increase of air pressure over North Africa.

ATP Online - Meteorology

Page 202 of 213

C. It is a northeasterly wind over the western part of North Africa with much dust and sand. D. It blows from southerly directions and can carry dust and sand which may reach Europe.

1299. What conditions are necessary for vertical visibility to be reported? A. B. C. D.

It is always reported. Whenever the sky is obscured by fog or heavy precipitation and the height of the cloud base cannot be measured. Whenever the lowest cloud base is below 1500 ft above ground. Whenever the lowest ceiling is below 1500 ft above ground.

1300. Which of the following statements is true concerning the Bergeron-Findeisen process? A. B. C. D.

It only takes place in clouds with supercooled water droplets and ice crystals It takes place in clouds which are composed only of ice crystals It does not occur in the tropics It only occurs in the mid-latitudes

1301. Which of the following statements is true concerning the coalescence process? A. B. C. D.

It only occurs in the tropics It only takes place in convective clouds In the mid-latitudes this process produces only drizzle or very light rain It only occurs in the mid-latitudes

1302. Which of the following statements is true concerning orographic fog? A. B. C. D.

It usually forms by night It is formed when air is forced to rise up a slope It develops when air is descending down a slope It usually forms by day

1303. Which of the following statements is true concerning frontal fog? A. B. C. D.

It develops rapidly and dissipates slowly It usually forms by night It may be formed by day as well as by night It usually forms by day

1304. If the subscale of an altimeter is set to QNH, what will it indicate after landing? A. B. C. D.

Aerodrome elevation. Pressure altitude. Less than aerodrome elevation when it is colder than standard, more than aerodrome elevation when it is warmer than standard. Less than aerodrome elevation when pressure is lower than standard, more than aerodrome elevation when pressure is higher than standard.

1305. In winter, after breaking through a low level inversion during descent and assuming the pilot does not intervene, A. B. C. D.

thrust most likely will increase and visibility is likely to deteriorate. lift most likely will increase and visibility is likely to improve. thrust most likely will decrease and visibility is likely to improve. lift most likely will decrease and visibility is likely to deteriorate.

1306. What degree of aircraft icing is determined by the following ICAO description? "Conditions in which change of heading and/or altitude may be considered desirable" A. B. C. D.

Severe Violent Moderate Light

1307. What type of precipitation might occur at 1730 UTC? MKJP 160430Z 160606 36010KT 9999 FEW025 BECMG 1315 14020G34KT FEW015CB SCT025 PROB30 TEMPO 1720 2500 +SHRA SCT010 BKN015CB BECMG 2224 34010KT FEW025= A. B. C. D.

Heavy rain showers Continuous moderate rain Light drizzle Intermittent light rain

1308. In which of the following changes of state is latent heat absorbed? A. B. C. D.

Gas to liquid Liquid to gas Liquid to solid Gas to solid

ATP Online - Meteorology 1309. Which of the following changes of state is known as evaporation? A. B. C. D.

Liquid to solid Gas to solid Liquid to gas Solid to liquid

1310. In which of the following changes of state is latent heat absorbed? A. B. C. D.

Gas to liquid Liquid to solid Gas to solid Solid to gas

1311. In which of the following changes of state is latent heat absorbed? A. B. C. D.

Solid to liquid Liquid to solid Gas to solid Gas to liquid

1312. Radiation fog most frequently occurs in: A. B. C. D.

low pressure systems over land. high pressure systems over sea. low pressure systems over sea. high pressure systems over land.

1313. What are the meanings of the meteorological abbreviations SQ, BR, MIFG and SG respectively? A. B. C. D.

Sudden strong increase in wind speed lasting for at least one minute / haze / patches of fog / snowgrains Sudden increase in wind speed lasting for at least one minute / mist / shallow fog / snowgrains Short wind gust / mist / shallow fog / polar snow Short wind gust / fog / shallow fog / snowgrains

1314. The air mass type advected from a direction indicated by arrow number 6 is designated A. B. C. D.

maritime polar continental polar continental arctic maritime arctic

1315. During the winter, the air mass type advected from a direction indicated by arrow number 1 is designated A. B. C. D.

maritime polar maritime arctic continental arctic continental polar

Page 203 of 213

ATP Online - Meteorology

Page 204 of 213

1316. An aircraft encounters atmospheric conditions in which abrupt changes of attitude and altitude of the aircraft occur. Changes in accelerometer readings go up to 1.2 g measured at the aircraft's centre of gravity. Lose objects inside the aircraft are tossed about. ; Which report, if any, is required? A. B. C. D.

Flight safety report No air-report is necessary Special air-report Routine air-report

1317. What does the code TAF AMD mean? A. B. C. D.

Delayed issue of a TAF. Revised TAF. Weather conditions expected to affect the safety of normal operations. No change compared with the previous TAF.

1318. Which of the following statements is correct concerning the weather in a warm sector? A. B. C. D.

Isolated thunderstorms are sometimes possible over continental areas during the summer In mid- and high latitudes thunderstorms may occur only over open sea during the summer No thunderstorms can occur Isolated air mass thunderstorms are common during all seasons over continental areas

1319. During flight the headwind increases suddenly due to wind shear. What initial effect does that have on true airspeed? A. B. C. D.

None, since wind only affects ground speed and drift. There will be a sudden temporary decrease in true air speed. There will only be a gradual increase in true airspeed until equilibrium is established again. There will be a sudden temporary increase in true air speed.

1320. The trade winds on the northern hemisphere blow from A. B. C. D.

west. north. south-west. north-east.

1321. What wind direction will occur over the sea at point F? A. B. C. D.

Northwest Northeast South Southeast

ATP Online - Meteorology

1322. What wind direction will occur over the land at point E? A. B. C. D.

Northeast Southeast West Southwest

1323. Which of the following statements concerning the airflow during the summer monsoon is correct? A. B. C. D.

Southwesterly airflow over India Westerly airflow over Japan Northeasterly airflow over India Easterly airflow over Japan

1324. Strongest clear air turbulence is to be expected A. B. C. D.

in the jet stream core on the warm air side of the jet stream core about 10000 ft above the jet stream core on the cold air side of the jet stream core

1325. On which side of an easterly wave are thunderstorms most likely to develop? A. On the west side of the wave B. On the east side of the wave C. On the south side of the wave

Page 205 of 213

ATP Online - Meteorology

Page 206 of 213

D. On the north side of the wave

1326. Which pilots are obliged to transmit a special air-report if they encounter severe aircraft icing? A. B. C. D.

All categories of pilots Only pilots with ATPL or IR Only pilots with ATPL or IR or CPL Only pilots with CPL or PPL

1327. What are the typical weather conditions in a warm anticyclone over land? A. B. C. D.

Strong westerly winds Only short term weather improvements Fine weather dominates in summer In the winter ground fog formation is not common

1328. What are the characteristics of rime ice, and what conditions are most favourable for its formation? A. Transparent appearance and tendency to take the shape of the surface on which it freezes. Stratiform clouds and temperatures only slightly below freezing promote its formation. B. Opaque rough appearance, tending to spread back over an aircraft surface. Most frequently encountered in cumuliform clouds at temperatures slightly below freezing. C. Milky granular appearance, forming on leading edges and accumulating forward into the air stream. Stratiform clouds at temperatures of -10°C to -20°C are most conducive to its formation. D. Smooth appearance and builds forward from leading surfaces into a sharp edge. Most common in cumuliform clouds at temperatures of -20°C to -25°C.

1329. In wintertime stratus is often formed when warm maritime air moves over cold land. How can such clouds be classified? A. B. C. D.

Convective clouds. Frontal clouds. Turbulence clouds. Orographic clouds.

1330. Runway visual range is calculated from values derived from A. B. C. D.

forward-scatter meters ceilometers pluviometers psychrometers

1331. Over the Indian Ocean and the Bay of Bengal tropical cyclones are A. B. C. D.

never observed. rare, in the average one every two years. frequently observed, in the average 15 per year over the Indian Ocean, but never over the Bay of Bengal. occasionally observed, in the average 12 per year.

1332. Frontal thunderstorms are mainly associated with: A. B. C. D.

cols. ridges of high pressure. cold fronts. warm fronts.

1333. What does the code R24R/P1500 mean? A. B. C. D.

Snow clearance in progress on RWY 24 right, useable runway length 1500 metres. RVR RWY 24 right below 1500 metres. RVR RWY 24 right 1500 metres. RVR RWY 24 right above 1500 metres.

1334. Which of the following cloud types is found at high level? A. B. C. D.

ST SC CS AC

1335. The SW monsoon starts in the month of A. B. C. D.

March in Southern India to reach Pakistan end of April. September in Pakistan to reach Southern India in November. December in Southern India to reach Pakistan in May. June in Southern India to reach Pakistan in July.

ATP Online - Meteorology

Page 207 of 213

1336. The wind at 500ft above ground is 290/15kt and at the same position at 600ft above ground 300/22kt. How is this wind situation called? A. B. C. D.

Severe horizontal wind shear Moderate vertical wind shear Severe low level turbulence Gradual horizontal wind shear

1337. For subsonic cruising levels, a SIGMET is NOT issued in case of A. B. C. D.

volcanic ash. heavy sandstorm. severe turbulence. isolated CB.

1338. The position of the intertropical convergence zone A. B. C. D.

shows more variation over the oceans than over the continents depends on the activity of thunderstorm clouds shows more variation over the continents than over the oceans is more to the north in January than in July

1339. Considering the region between 20°N and 70°N, the normal position of the main anticyclones at the surface during winter is A. B. C. D.

Greenland, Iberian peninsula, Tibet Siberia, Iceland, Canaries Azores, Siberia, Canada NE Canada, Iceland, Bermuda

1340. To which type of message does the following text belong? ... VALID 211600/212000 HVY SN 25CM FCST= A. B. C. D.

SIGMET Aerodrome warning SNOWTAM Supplementary information of a METAR

1341. To which type of message does the following text belong? …… MBST APCH RWY26 REP AT 1510 B747 30KT ASPEEDL 2NM FNA= A. B. C. D.

SIGMET Wind shear warning AIRMET Aerodrome warning

1342. During flight in clouds, which of the following means is the best one to identify the position of embedded CBs? A. B. C. D.

The pilot requests the position of the CBs from the ATC controller Significant weather chart Aircraft observations Weather radar

1343. In which of the following changes of state is latent heat released? A. B. C. D.

Solid to liquid Solid to gas Liquid to gas Liquid to solid

1344. Which of the following changes of state is known as melting? A. B. C. D.

Solid to liquid Liquid to gas Solid to gas Liquid to solid

1345. In which of the following changes of state is latent heat released? A. B. C. D.

Gas to solid Solid to gas Liquid to gas Solid to liquid

1346. In January an extended high pressure area is normally observed over A. Central Europe B. Southern Africa C. Siberia

ATP Online - Meteorology

Page 208 of 213

D. South America

1347. Which of the following cloud types is found at high levels? A. B. C. D.

SC AC ST CC

1348. What are the typical weather conditions in a westerly situation? A. B. C. D.

Fine weather with strong westerly winds Frequent thunderstorms during winter Very changeable Steady rainfall

1349. Below is the forecast for the destination of a flight whose ETA is 2030 UTC.; TAF YUDO 120600Z 121206 17010KT 3000 SCT005 BECMG 1820 VRB03KT BCFG SKC BECMG 2022 0300 FG VV002=; What can be said about the destination airport at 2030 UTC if CAT II ILS minima 350 m RVR, 100 ft Decision Height are being considered? A. B. C. D.

Still open until 2200 UTC because the weather change is not completed before 2200 UTC. Has to be considered as closed. Open the whole night because 300 m of visibility corresponds to at least 900 m RVR. Still open because the decrease of the visibility below the CAT II minima will be a little bit later than 2030 UTC.

1350. A conditionally unstable air mass is forced to ascend a mountain slope. What type of clouds can be expected? A. B. C. D.

Clouds with considerable vertical development and associated turbulence. Layer-like clouds with little vertical development. Stratiform clouds with a temperature inversion. Stratiform clouds with considerable turbulence.

1351. In which conditions would you most likely encounter clear icing, and how would it normally appear? A. Cumuliform clouds, small water droplets, temperatures between -20°C and -25°C. Appears transparent and tends to take the shape of the surface on which it freezes. B. Stratiform clouds, large water droplets, temperatures well below freezing. Appears opaque and builds forward from leading surface into a sharp edge. C. Cumuliform clouds, large water droplets, temperatures between 0°C and -15°C. Appears smooth and tends to spread back over an aircraft wing. D. Stratiform clouds, small water droplets, temperatures between -10°C and -20°C. Appears granular and tends to accumulate forward into the air stream.

1352. Which of the following cloud types will most probably form when cold moist air moves over a warm surface? A. B. C. D.

Cumulus. Stratus. Stratocumulus. Nimbostratus.

1353. What cloud type can produce freezing rain? A. B. C. D.

Stratus. Cumulonimbus. Stratocumulus. Nimbostratus.

1354. Which of the following phenomena has to be mentioned in a SIGMET? A. B. C. D.

Thick fog. Strong inversion. Heavy duststorm. Snow and ice on the runway.

1355. Which of the following phenomena has to be mentioned in a SIGMET? A. B. C. D.

Snow and ice on the runway. Volcanic ash. Strong inversion. Thick fog.

1356. In towering cumulus clouds A. B. C. D.

weak vertical air motion exists and the temperature in the cloud is lower than in the surrounding air at the same height weak vertical air motion exists and the temperature in the cloud is higher than in the surrounding air at the same height strong updraughts occur and the temperature in the cloud is higher than in the surrounding air at the same height strong updraughts occur and the temperature in the cloud is lower than in the surrounding air at the same height

ATP Online - Meteorology

Page 209 of 213

1357. In stratiform clouds A. B. C. D.

strong vertical currents occur and the temperature is much lower than the temperature of the surrounding air at the same height strong vertical currents occur and the temperature is much higher than the temperature of the surrounding air at the same height weak vertical currents occur and the temperature is nearly equal to the temperature of the surrounding air at the same height weak vertical currents occur and the temperature is much higher than the temperature of the surrounding air at the same height

1358. During periods of cloudless weather over land in the northern hemisphere the A. B. C. D.

angle between isobars and surface wind direction tends to be greatest in the mid afternoon surface wind speed tends to be highest during the mid afternoon surface wind speed tends to be highest at night wind tends to back from early morning until early afternoon

1359. Which of the following symbols represents a quasi-stationary front? A. B. C. D.

Symbol 3 Symbol 1 Symbol 2 Symbol 4

1360. Which kind of meteorological messages are normally transmitted in an ATIS? A. B. C. D.

TAF / SPECI METAR / SPECI AIRMET / GAMET MET REPORT / SPECIAL

1361. Which kind of meteorological messages are normally transmitted in a VHF-VOLMET broadcast within Europe? A. B. C. D.

GAMET / AIRMET TAF / SPECIAL AIREP / SPECIAL METAR / SPECI

1362. A night in January has been cold and clear. In the forenoon of the next day an airfield is in IMC due to fog which extends to a height of 700 ft. A heavily laden aircraft departs and, when breaking through the top of the fog, experiences temporary bumpiness and a sudden decrease in climb rate.; Mark the answer that states the most probable reason for the altered aircraft performance. A. B. C. D.

The aircraft suffers reduced lift and thrust due to a temperature inversion at the top of the fog. The additional weight gained by water which has been gathered on wings and fuselage during flight through the fog reduces the lift. The water which has been gathered during flight through the fog modifies the aerodynamic properties of the aircraft, thus reducing lift. The aircraft is under influence of a mountain breeze (descending air from the surrounding hills due to the incident sunshine).

1363. An aircraft flies at flight level 40. Elevation of the aerodrome: 990 ft; QNH 976 hPa. The tower clears the pilot to fly at 3000 ft QNH. Which of the following statements is correct? A. B. C. D.

The aircraft has to climb about 1000 ft. Only a small change of altitude is necessary. The aircraft has to descend about 2000 ft. The aircraft has to descend about 1000 ft.

1364. A given mass of air is saturated with water vapour (no condensed water). If temperature increases A. B. C. D.

the amount of water vapour remains constant the amount of water vapour decreases relative humidity increases relative humidity remains constant

1365. What is meant by QFE? A. The present atmospheric pressure at an aerodrome converted to mean sea level in accordance with the actual conditions. B. The atmospheric pressure at the official aerodrome elevation.

ATP Online - Meteorology

Page 210 of 213

C. The atmospheric pressure at the official aerodrome elevation, reduced to mean sea level in standard atmospheric conditions. D. The pressure at MSL in standard atmospheric conditions.

1366. Which of the following statements is correct concerning rotors below the crest of a mountain wave? A. B. C. D.

The axis of these rotors is vertical. The wind direction at the top of the rotors is opposite to the prevailing wind direction. The rotors are always visible by the presence of rotor clouds. The wind direction at the lower side of the rotors is opposite to the prevailing wind direction.

1367. While using the airborne weather radar in order to circumnavigate thunderstorms, the radar scope is clear between heavy echoes. Which of the following interpretations of the scope is correct? A. The clear area indicates an area from which no echoes are received. However, this radar scope provides no assurance of being in VMC while flying in this area. B. The clear area indicates an area from which no echoes are received. An aircraft flying in this area will be in VMC due to the absence of precipitation drops. C. The clear area indicates an area from which no echoes are received because the number of cloud droplets is too small to generate a significant echo. Thus, the pilot will have a good visual sight of the thunderstorm clouds. D. This is a typical radar scope from which the region of clear air turbulence between thunderstorm clouds can be detected.

1368. The polar front is A. B. C. D.

the boundary between arctic and tropical air masses. the boundary between arctic and polar air masses. the constant frontal activity situated at about 80 degrees north and south. the boundary between polar and tropical air masses.

1369. Moderate to severe CAT is encountered especially in A. B. C. D.

a CB the laminar part of a mountain wave the core of a jet stream an area with strongly curved, closely packed isohypses

1370. Which of the following statements regarding an aircraft being struck by lightning is true? A. Though flying through an electric field, the aircraft itself is not a charge carrier and therefore it cannot initiate a lightning discharge. B. The dimension of the damage caused by the lightning stroke is reciprocally proportional to the strength of the electric field the aircraft intruded. C. The crew of an aircraft made of conductive material (e.g. metal) will generally suffer greater adverse effects due to lightning discharge than the crew of an aircraft made of non-conductive composite material. D. The flight crew might have temporary difficulties in determining the current attitude of flight.

1371. What is the meaning of the following weather message? …… CNL WS WRNG 1 211130/211330= A. B. C. D.

The weather warning with the number 121 concerning a cyclone at low levels has a validity time from 1130 until 1330 UTC. The first wind shear warning on the 21st of that month, issued at 1130 UTC has been cancelled at the same day at 1330 UTC. This message cancels all weather warnings issued on the 21st of January between 1130 and 1330 UTC. The first wind shear warning on the 21st of that month with an occasional intensity has a forecast validity from 1130 until 1330 UTC.

1372. A cold air pool can be best identified by means of A. B. C. D.

the CAT areas on a significant weather chart a GAMET report the isobars on a surface chart the isohypses on an upper air chart

1373. With regard to temperature, which of the following is correct for the polar climates? A. B. C. D.

The mean temperature of all months is above 0°C, but temperatures as high as plus 10°C will never be reached. The mean temperature of all months is below minus 10°C. The mean temperature of all months is below plus 10°C. The mean temperature of the warmest month is 10°C or above, and of the coldest month below minus 3°C.

1374. Consider a parcel of air which is moved upwards in the surrounding air by an external force. Which one of the following situations describes instability? A. B. C. D.

The parcel tends to descend when it is released. It will move to, and maintain an altitude below its original altitude. The parcel tends to return to its original altitude when it is released. The parcel tends to gain altitude after being released. The parcel tends to maintain its new altitude after being released.

1375. Which of the following situations favours the formation of heavy thunderstorms? A. B. C. D.

A warm sector moving over a snow-covered ground. The passage of a warm front in the morning. A cold front on the leeward side of a mountain range. A cold front approaching a mountain range in the evening.

ATP Online - Meteorology

Page 211 of 213

1376. What is "mixing ratio"? A. B. C. D.

The percentage of water drops per volume unit of dry air. The ratio between actual water vapour content and saturated water vapour content. The volume of water vapour which is mixed with each volume unit of dry air. The number of grammes of water vapour per kilogramme of dry air.

1377. What is meant by QFF? A. B. C. D.

The pressure at mean sea level in standard atmospheric conditions. The present atmospheric pressure at an aerodrome converted to mean sea level in accordance with the actual conditions. The present atmospheric pressure at an aerodrome converted to mean sea level in accordance with the ICAO standard atmosphere. The atmospheric pressure at the official aerodrome elevation.

1378. Mark the correct statement concerning lightning discharge. A. B. C. D.

While flying through air that is electrically charged the aircraft is likely to become a charge carrier itself and can initiate a lightning discharge. Because an aircraft acts as a "Faraday's cage", the magnetisation associated with a lightning discharge hardly has an effect on the magnetic compass. The set of possible effects of a lightning discharge does not include welding of aircraft parts such as joints or bearings. The probability of a lightning discharge is reciprocally proportional to the strength of the electric field the aircraft intruded.

ATP Online - Meteorology

Page 212 of 213

Meteorology - Exam Key 1 11 21 31 41 51 61 71 81 91 101 111 121 131 141 151 161 171 181 191 201 211 221 231 241 251 261 271 281 291 301 311 321 331 341 351 361 371 381 391 401 411 421 431 441 451 461 471 481 491 501 511 521 531 541 551 561 571 581 591 601 611 621 631 641 651 661 671 681 691 701

D A B B A B A C C D A B B C D C D C D C B D D A B D B A A A B D D A C A B C A A B D C D A D A B A C A B A A A A C A D A C A A D B C A C B C B

2 12 22 32 42 52 62 72 82 92 102 112 122 132 142 152 162 172 182 192 202 212 222 232 242 252 262 272 282 292 302 312 322 332 342 352 362 372 382 392 402 412 422 432 442 452 462 472 482 492 502 512 522 532 542 552 562 572 582 592 602 612 622 632 642 652 662 672 682 692 702

A D C D D B C A D B B A C D B C C A A C D A D B A A C D B D D C C C A D B D D D D D C C A A B B D A C C A D C A D A C C A B A B A B C C B A D

3 13 23 33 43 53 63 73 83 93 103 113 123 133 143 153 163 173 183 193 203 213 223 233 243 253 263 273 283 293 303 313 323 333 343 353 363 373 383 393 403 413 423 433 443 453 463 473 483 493 503 513 523 533 543 553 563 573 583 593 603 613 623 633 643 653 663 673 683 693 703

D D C C C C A C C C B A D A C A D B B A C C B D A B B A C C B A A D A B B C C A A A A B B D B A A C A C D B A D C D A B A A B D A A D B D D D

4 14 24 34 44 54 64 74 84 94 104 114 124 134 144 154 164 174 184 194 204 214 224 234 244 254 264 274 284 294 304 314 324 334 344 354 364 374 384 394 404 414 424 434 444 454 464 474 484 494 504 514 524 534 544 554 564 574 584 594 604 614 624 634 644 654 664 674 684 694 704

B D C D D A D C B B C A C B A A D B D B D A C A C D D D D B D D A A B B C A C A C D D B D C A D D D A D D B B C C C C B C B D D B A D A D C A

5 15 25 35 45 55 65 75 85 95 105 115 125 135 145 155 165 175 185 195 205 215 225 235 245 255 265 275 285 295 305 315 325 335 345 355 365 375 385 395 405 415 425 435 445 455 465 475 485 495 505 515 525 535 545 555 565 575 585 595 605 615 625 635 645 655 665 675 685 695 705

A D C B D A C C B B A B A D B B C A C D C C A A A D D C B C A A B C C C A A B D C D A A D C B A B B C D C D A D C C D B D D D C A D D B C D A

6 16 26 36 46 56 66 76 86 96 106 116 126 136 146 156 166 176 186 196 206 216 226 236 246 256 266 276 286 296 306 316 326 336 346 356 366 376 386 396 406 416 426 436 446 456 466 476 486 496 506 516 526 536 546 556 566 576 586 596 606 616 626 636 646 656 666 676 686 696 706

B C B B A C A A C B D D B C B A B B D B B A D B C C B B C B A C B B A B B A D C B D A C B C D A B A D A B C A C C A C D D A B D C A B A B A B

7 17 27 37 47 57 67 77 87 97 107 117 127 137 147 157 167 177 187 197 207 217 227 237 247 257 267 277 287 297 307 317 327 337 347 357 367 377 387 397 407 417 427 437 447 457 467 477 487 497 507 517 527 537 547 557 567 577 587 597 607 617 627 637 647 657 667 677 687 697 707

B C D D A A A B A B C B C C B B D C A C A D B D C B B C A B C B A A D C A A D B D A D D B A B B C A A B C B C C C A D D B B D A A D A C A C C

8 18 28 38 48 58 68 78 88 98 108 118 128 138 148 158 168 178 188 198 208 218 228 238 248 258 268 278 288 298 308 318 328 338 348 358 368 378 388 398 408 418 428 438 448 458 468 478 488 498 508 518 528 538 548 558 568 578 588 598 608 618 628 638 648 658 668 678 688 698 708

C A C A C A B D C B B C B B B D A D C B C A D B B B A D C C D A D A A A A B D D C C A A D A C C C A C D A D D C B B A C A A D A D B B A B C D

9 19 29 39 49 59 69 79 89 99 109 119 129 139 149 159 169 179 189 199 209 219 229 239 249 259 269 279 289 299 309 319 329 339 349 359 369 379 389 399 409 419 429 439 449 459 469 479 489 499 509 519 529 539 549 559 569 579 589 599 609 619 629 639 649 659 669 679 689 699 709

A B D C D A A A B D C C B D D B C D C C A B C B A B C C C A C D A B C C A C C C C C A B A D A A A A B A C D A D D A C C D D A B B B C C C D B

10 20 30 40 50 60 70 80 90 100 110 120 130 140 150 160 170 180 190 200 210 220 230 240 250 260 270 280 290 300 310 320 330 340 350 360 370 380 390 400 410 420 430 440 450 460 470 480 490 500 510 520 530 540 550 560 570 580 590 600 610 620 630 640 650 660 670 680 690 700 710

B A C A B B C D D B A A A D A B A D C A A A A A C D C A D A D C D A A A D A C D A C D B D A B B B B C B B C A B C A B D C B B A A A A C A C A

ATP Online - Meteorology 711 721 731 741 751 761 771 781 791 801 811 821 831 841 851 861 871 881 891 901 911 921 931 941 951 961 971 981 991 1001 1011 1021 1031 1041 1051 1061 1071 1081 1091 1101 1111 1121 1131 1141 1151 1161 1171 1181 1191 1201 1211 1221 1231 1241 1251 1261 1271 1281 1291 1301 1311 1321 1331 1341 1351 1361 1371

B C D B A A A B C C C A C C B B A D A D B A A B B D A D D B C A C C B C B B C B B D C B C A A A A D D B D D B D C B A C A C D B C D B

712 722 732 742 752 762 772 782 792 802 812 822 832 842 852 862 872 882 892 902 912 922 932 942 952 962 972 982 992 1002 1012 1022 1032 1042 1052 1062 1072 1082 1092 1102 1112 1122 1132 1142 1152 1162 1172 1182 1192 1202 1212 1222 1232 1242 1252 1262 1272 1282 1292 1302 1312 1322 1332 1342 1352 1362 1372

A A B D A C A C C A A C A A C B D C A B C B B A C D B C D C D A B A C B A A B C D C A C B B D B A B B C B D C C A C C B D C C D A A D

713 723 733 743 753 763 773 783 793 803 813 823 833 843 853 863 873 883 893 903 913 923 933 943 953 963 973 983 993 1003 1013 1023 1033 1043 1053 1063 1073 1083 1093 1103 1113 1123 1133 1143 1153 1163 1173 1183 1193 1203 1213 1223 1233 1243 1253 1263 1273 1283 1293 1303 1313 1323 1333 1343 1353 1363 1373

C C C B B D B C D A B B C C B C B D B C A C B B B D A D B B C D B C B A C A D D D A C B B C C A D B C A D D C B C A A C B A D D D B C

714 724 734 744 754 764 774 784 794 804 814 824 834 844 854 864 874 884 894 904 914 924 934 944 954 964 974 984 994 1004 1014 1024 1034 1044 1054 1064 1074 1084 1094 1104 1114 1124 1134 1144 1154 1164 1174 1184 1194 1204 1214 1224 1234 1244 1254 1264 1274 1284 1294 1304 1314 1324 1334 1344 1354 1364 1374

A A A B D B B D C A B C C D D D D A C D D D A C D B B A D B B C C D C A C D C D D B A D B B C D B C D C D D B A B D D A A D C A C A C

715 725 735 745 755 765 775 785 795 805 815 825 835 845 855 865 875 885 895 905 915 925 935 945 955 965 975 985 995 1005 1015 1025 1035 1045 1055 1065 1075 1085 1095 1105 1115 1125 1135 1145 1155 1165 1175 1185 1195 1205 1215 1225 1235 1245 1255 1265 1275 1285 1295 1305 1315 1325 1335 1345 1355 1365 1375

Page 213 of 213 C A A B A A D A A B B A C D A C C D B B D D C D D D B C B A D C C A D A C B B A A D A C C D D C B A D C A C B B A C C A B B D A B B D

716 726 736 746 756 766 776 786 796 806 816 826 836 846 856 866 876 886 896 906 916 926 936 946 956 966 976 986 996 1006 1016 1026 1036 1046 1056 1066 1076 1086 1096 1106 1116 1126 1136 1146 1156 1166 1176 1186 1196 1206 1216 1226 1236 1246 1256 1266 1276 1286 1296 1306 1316 1326 1336 1346 1356 1366 1376

D D D D D D A A B D B A A D B C C D A A B B B B A C D C C A B B A C C A A A D A D C B C D B B A C C C C D D A C B A B C C A B C C D D

717 727 737 747 757 767 777 787 797 807 817 827 837 847 857 867 877 887 897 907 917 927 937 947 957 967 977 987 997 1007 1017 1027 1037 1047 1057 1067 1077 1087 1097 1107 1117 1127 1137 1147 1157 1167 1177 1187 1197 1207 1217 1227 1237 1247 1257 1267 1277 1287 1297 1307 1317 1327 1337 1347 1357 1367 1377

C A A B C A D A D C B B D A B B A A B C B C A C A D C A B A B D C C B A D B D B D C B A D B D B A A B C A A B A C C C A B C D D C A B

718 728 738 748 758 768 778 788 798 808 818 828 838 848 858 868 878 888 898 908 918 928 938 948 958 968 978 988 998 1008 1018 1028 1038 1048 1058 1068 1078 1088 1098 1108 1118 1128 1138 1148 1158 1168 1178 1188 1198 1208 1218 1228 1238 1248 1258 1268 1278 1288 1298 1308 1318 1328 1338 1348 1358 1368 1378

D D D B A B C A C B D B D D D A D D A D D D C B B C C C D A B D C D C A C A C C C A C D D A D B A A A A C A D C B A D B A C C C B D A

719 729 739 749 759 769 779 789 799 809 819 829 839 849 859 869 879 889 899 909 919 929 939 949 959 969 979 989 999 1009 1019 1029 1039 1049 1059 1069 1079 1089 1099 1109 1119 1129 1139 1149 1159 1169 1179 1189 1199 1209 1219 1229 1239 1249 1259 1269 1279 1289 1299 1309 1319 1329 1339 1349 1359 1369

B C B A A D B D C C A A D A A C B D C A A B A D C D D D D C D D A C A C B C C A A A D D B C A B D B C D C D C C C A B C D C C B A D

720 730 740 750 760 770 780 790 800 810 820 830 840 850 860 870 880 890 900 910 920 930 940 950 960 970 980 990 1000 1010 1020 1030 1040 1050 1060 1070 1080 1090 1100 1110 1120 1130 1140 1150 1160 1170 1180 1190 1200 1210 1220 1230 1240 1250 1260 1270 1280 1290 1300 1310 1320 1330 1340 1350 1360 1370

C D D D A C A A A B A B D D D A A D C B B B B D C C D C A C D D B D A C C D C C B C A C D D B C C B A B B B C B B B A D D A B A D D

View more...

Comments

Copyright ©2017 KUPDF Inc.
SUPPORT KUPDF